Sunteți pe pagina 1din 285

1

ALCOHOL & SMOKING

Hazardous drinking (Social alcoholism)


Your next patient in general practice is a 47 year old businessman, John Ramm, who wants to
discuss his alcohol consumption with you because he got pulled over by the police in the
morning on his way to work and he was breathalised with a BAC of 0.04. The police officer
warned him that this was close to the legal limit and a sign that he must have had a lot to
drink the night before. That gave him a bit of a shock and he wants to find out how much
alcohol is safe to drink and general information about the effect of alcohol on a person.

Task: Brief Hx, Perform PE and Ix, explain findings, Dx and Mx to the patient

HOPC: John is a self employed business man who had a business dinner meeting a few days
ago with quite a bit of wine and he actually can’t remember when and how he got home, but
his car was in the garage and he woke up in the morning with a bit of a headache as it
happens after a heavy session. Driving to his office in the morning he got breathalysed and
blew 0.04 which surprised the police officer although it was not over the legal limit. The
police officer mentioned to him that he must have had a very high blood alcohol level the
night before which gave John a shock realizing that if he had been caught last night he would
have lost his license which is vital for his business.
John has been drinking regularly for many years. As a sales representative for stationery he
meets lots of clients and entertains them often for lunch or dinner and usually there is some
beer or wine consumed and he also drinks wine if he has dinner at home. Basically he would
average 5 to 8 standard drinks per day 7/7, often being quite tipsy. .
He has never lost his driving license and he does not feel that the alcohol has done harm to
his physical, mental or social health. Although he has put on 12-15 kg over the last two years
and he does not do much physical exercise anymore and he also noticed that his sexual
function had deteriorated over the last few years even though he still has the desire for it.
He quite often is not able to get up in the morning because of mild to moderate hang-overs
but because he is self employed it has not caused any problems.
He does not spend a lot of time with his children and does not feel fit enough to kick the ball
with them or to go surfing like he used to years ago.
PHx. + FHx.: unremarkable although his father was quite a solid drinker but died of old age
in his eighties.
SHx.: married with 3 children (17, 12 and 9 years old), self employed sales man, no financial
problems, non smoker, NKA, no medication.

O/E: moderately overweight (BMI 27) but otherwise well looking man, P 72/min +reg., BP
168/95, RR 18, afebrile.
The abdominal examination reveals a mildly enlarged, soft liver., otherwise there are no
pathological findings on PE.

Ix:
• FBE: possible anaemia, macrocytosis and raised mean corpuscular volume(MCV normal
10-96 fL)
• LFTs: elevated gamma glutamyl transaminase (GGT), but others can be elevated as well.
• Serum uric acid often raised
• Lipids high

! 1!
ALCOHOL & SMOKING

Dx: ”HAZARDOUS DRINKING” (SOCIAL ALCOHOLISM)!


6.5% of Australians have had an alcohol-use disorder in the last 12 months and up to 40% of
emergency department patients have alcohol problems. Alcoholism is a common and socially
destructive problem in Australia. The identification of alcohol misuse is often difficult and
requires a sensitive history taking. It is important to identify the difference between regular
drinking and binge drinking!
Certain professional groups are at risk of “drinking”:
Young and middle-aged bachelors
Divorced or separated individuals
Alcoholic beverage trade: bar trade, hotel staff
Professionals: politicians, doctors and others
Travelling professions, e.g. seamen, salesmen, truck drivers
Armed forces, especially returned servicemen
Authors, journalists and related workers
Social club patrons, e.g. sporting clubs
There are a couple of useful questionnaires available:
1. CAGE:
C: thoughts of cutting down drinking
A: annoyed at criticism of drinking habits
G: guilty about drinking
E: “eye opener” drink to get rid of a hangover

2. AUDIT: the alcohol use disorder identification test, a screening test for problem
drinking developed by WHO with 10 questions relating to alcohol use.

The risk has been identified by NHMRC :

NH&MRC guidelines: number of standard drinks each day

Low risk Hazardous Harmful


Men 0-4 5-6 >6
Women 0-2 3-4 >4

One standard drink contains 10 g of alcohol which equals a pot of standard beer, a small
glass of wine, 1 glass of sherry (60 mls) or 1 glass of spirits (30 mls).
This can be helpful to calculate the amount of drinks before one reaches the 0.05 level BAC,
considering that 1 standard drink raises the BAC by 0.01. That means a person reaches 0.05
after 5 standard drinks. On the other hand the liver metabolises about 1 standard drink of
alcohol per hour, which means that the same person can than drink 1 standard drink per hour
to stay below 0.05.

Alcohol dependence can be physical, psychological or social!:

! 2!
ALCOHOL & SMOKING

Physical dependence is characterised by increasing tolerance and withdrawal and relief of


withdrawal symptoms by further alcohol consumption. They can be expressed in a wide
variety of effects:

Psychological and social effects Physical effects


• brain damage (if severe)
• depression
• epilepsy
• Wernicke-Korsakoff syndrome
• insomnia—nightmares
• hypertension
• loss of self-esteem • heart disease
• irritability — arrhythmias
• devious behaviour — cardiomyopathy
• anxiety and phobias — beri-beri heart disease
• depression • liver disease
• paranoia • pancreatic disease
• stress • dyspepsia (indigestion)
• relationship breakdown • acute gastritis
• child abuse • stomach ulcers
• poor work performance • sexual dysfunction
• memory disturbances • hand tremor
• financial problems • peripheral neuropathy
• accidents • myopathy
• driving offences • gout
• crime-violence • obesity
• personal neglect • other metabolic/endocrine effects
• attempted suicide — hyperlipidaemia
• pathological jealousy — pseudo-Cushing's syndrome
— osteoporosis
— osteomalacia
• haemopoiesis
— macrocytosis
— leucopenia
— thrombocytopenia

We classify adults according to the amount of alcohol they drink as


1. abstainers
2. social drinkers
3. social alcoholics
4. alcoholics

MANAGEMENT OPTIONS:
• EARLY IDENTIFICATION!!!
• Period of abstinence
• Alcoholics anonymous
• Life style changes
• Pharmacological options (thiamine, naltrexone, disulfiram/antabuse)

! 3!
2 ALCOHOL & SMOKING

Smoking counselling
A 30 year-old man return to your practice after a recent chest infection. He smoke 20
cigarettes a day. Last visit you advised him to quit smoking and now you want to follow-up
his respond to the previous advice. Counsel about quitting smoking.

Task: to assess his motivation to stop smoking. Counsel his approach and to discuss any
therapeutic option.

Hx:
I’m very glad that you have decided to quit and I can help you
I’d like to ask you some Qs (To assess how addicted he is)
How many cigarettes a day?
How long have you been smoking?
What is the pattern of your smoking a day?
How soon you smoke after you wake up in the morning?
Is this one (start early morning) the most difficult to quit?
Do you still like to smoke even if you’re very ill and can’t get out of bed?
How do you feel when you’re in the area of No Smoking sign?
Do you feel you have to smoke in spite of the sign?
Have you tried to quit before---failed trial of quitting
How many times
Why did the attempts fail ---perhaps very bad withdrawal symptoms
Apart from smoking, do you drink alcohol---go together drink & smoking
Coffee & tea---combination of coffee & smoking

Explanation:
The most important thing to quit is the strong motivation, I’m glad that you have made your
mind/decided to quit
It’s not too late, however quitting may not be very easy
Because you’ve been smoking in this pattern, you may get some withdrawal symptoms after
quitting and some people may get some :
- depression
- poor sleep
- irritability
- anxiety
- reduced concentration
- sometimes you may feel craving for smoke and sugar
- you may also notice an increase in your appetite
- Most of the people who quit they put on some weight

These withdrawal symptoms are maximal in the first few weeks (1-3 weeks after quitting)
and then they will disappear. The increase appetite may last in 1-2 years. We can help you if
you develop anyone of these symptoms.

The psychogenic symptoms (nervousness, anxiety) treat with benzodiazepine short course.

Advise to quit smoking:


1. Set a date by which you will quit totally, it’s good to quit within 2 weeks from now (he
may change his mind later)

! 4!
ALCOHOL & SMOKING

2. Your aim is total quit, no gradual reduction


3. Tell patient to review his previous attempts to quit and why he smoke again, in order not
to repeat the same mistakes again
4. Inform your relatives & friends that you quit smoking
5. Try to reduce alcohol & coffee intake
6. If you develop any of these withdrawal signs, please feel free to visit me or contact me
7. You may put on some weight, you need to watch your diet and exercise more
8. You may need some nicotine replacement
9. Frequent follow-ups, I need to see you every few days, very critical within the first 2
weeks (phone visits)
10. Offer resource materials and some helpful lines like quitting support groups

The aim of nicotine replacement is to help with the withdrawal symptoms in chronic smoker
and addictive symptoms
To replace 40% of nicotine that the patient was getting from smoking
- Nicotine patches
- Nicotine gum
- Nicotine inhalers

Contraindication
- Pregnancy
- Ischaemic heart disease

Medication to help:
- Bupropion  contraindicated in pregnancy, epileptic, relatively contraindicated in
diabetics (given for a few weeks)

Assess motivation
How bad smoker
Explain withdrawal symptoms
Advise for quitting lines
Quit total not gradual
Support group
!

! 5!
3 ALCOHOL & SMOKING

Alcohol counselling
You’re working in a GP clinic and you see a 45 year-old man consulted you regarding harmful effect
of alcohol after watching a program about it on TV.

Task: take relevant history, arrange further investigation & management.

Hx:
Mr Smith I know that you’re concern about your drinking, it’s a very good start. I’d like to ask you a
few Qs, some of them might be personal, is it OK with you?
Drinking habit
- Please tell me for how many years have you been drinking?…usually a long time
- How much are you drinking per week
- What type of alcohol are you drinking…. spirit, wine or beer
- Where do you prefer to drink, with your family or friends.
- Continuously or binge
- Are you aware of the safe level of drinking
- Have you noticed any ill effect of alcohol on you
- Tolerance – do you think you drink heavily without feeling or appearing drunken
Withdrawal effect
- How long can you go without alcohol  not more than 1 day
- How do you feel in the morning
- Any symptoms of agitation, sweating, nausea, shaking
- How do you feel after a period of abstinence from alcohol
- Do these symptoms disappear if you drink
- Do you ever need to drink before going to sleep
Social effect
- Have you noticed any problem at your work
- Any problem at home
- How is your relationship with you partner and children
- Have you had any financial problems
Accidents
- Have you had any accidents or crimes due to over drinking
- Have you ever tried cutting down your drinking habits before
- Have you ever visited any detoxification centre
Health problems
- Have you noticed any heartburn, gastritis
- Hypertension
- Heart disease
- Liver disease
- Anemia
- Any problems regarding your memory
- Any recent mood change or depression
- Any change in your sexual performance? …pt say that he has problems related to this age
CAGE
- Have you ever thought of cutting down with your drinking habits
- Do you feel annoyed about people when they talk about your drinking
- Have you ever felt guilty due to your drinking habits
- Do you need drinking the first thing in the morning as an eye opener
SAD
- Do you smoke? ----- a lot
- Have you tried any illicit or recreational drug
- Is there any family history of alcohol dependence or liver disease

! 6!
ALCOHOL & SMOKING

Now Mr Smith I’d like to do some investigations to see the effect of alcohol in your body
1. FBE: to look for anemia & macrocytosis,
2. B12 (usually low), folic acid
3. LFT
4. Lipid profile
5. Serum lipase
6. BSL prone to DM
7. Liver US
8. ECG

Counselling:
FLAGS (Feedback, Listen to the patient, set Aims, Goals, Strategies)
Now Mr Smith, from the history & examination, it showed that you’re drinking alcohol more than
normal. You’re very good in coming here to discuss your condition. From the result of your
investigation, the results coincide with your alcohol intake.

Mr Smith, the high alcohol intake usually causes:


- high BP,
- increases your body weight,
- and sometimes cause harmful effect on your stomach such as heartburn,
- can effect your liver, heart and brain,
- can cause gout,
- blackouts,
- loss of memory,
- problems with your social life,
- sometimes causes problem in your sexual life, relationship
What do you think Mr Smith, is this affecting you?
I’m going to tell you the safe level of drinking, 2 standard glasses everyday for 5 days and 2 days off.
(1 SD = 10 grams of alcohol)

S – Strategies:
- To cut down, don’t drink daily, drink only with food
- Have a glass of water between drinks (to dilute, to quinch/satisfy the thirst)
- Switch to low alcohol drinks/beer, don’t drink on empty stomach, mix alcoholic drink with non-
alcoholic drink, avoid high risk situation
- Avoid or limit drinks with alcoholic friends
- Avoid going to pub after work
- If you’re under pressure, told them that my doctor told me to cut down
- When you’re in stress, take a walk, explore new interests, plan other activities or tasks at a time
when you usually have a drink
- Always check the level of SD before each drink.

Mr Smith, it will save your money and you will have less family problems. It will decrease your BP
and your LFT will come back to normal. A lot of support is available for you such as family, myself
and a lot of support groups.

Mr Smith, always aim for period of absence from alcohol.


If you develop any tremor, sweating, agitation please report straight away.

I will arrange for the follow-up with you and I can also arrange a family meeting.
Here is a flyer about alcoholism, please read it and any concern you can come and see me anytime.
You took a very good decision, a lot of support is available for you.

! 7!
4 CARDIOVASCULAR SYSTEM

Angina Pectoris (Abnormal Stress Test)


GP setting. A 62 yo male who presented a few days ago with chest discomfort. ECG done at that time
was normal. A stress test was done later on showed ST segment change in anterolateral leads. Pt had
chest tightness during the test. Pt is a smoker, no Hx of DM.

Task: explain the test result, short and long term management.

Hx:
Any chest pain or discomfort presented at the moment?
Pain Qs previous episodes
HTN? Cholesterol?
FHx of cardiac disease
Ask for the cardiac risk assessment

Explanation:
Mr X, the condition that you have is known as angina
Angina is the name given to chest pain or discomfort that comes from the heart when it’s short of
oxygen. The heart is supplied by vessels known as coronaries and in this condition, the coronaries is
narrowed and this is due to disposition of fat-like atheroma. A very common condition. Millions is
suffering from it  reassuring in an indirect way.
The symptoms is dull, heavy pain, discomfort usually in the centre of chest, pressure symptoms,
shortness of breath, usually associated with physical activity or effort

Mx:
1. Start you on some medications: aspirin, (beta blocker, ACE inhibitor), statins and GTN (carry
with you all the time)
2. Refer you to cardiologists ASAP for further assessment and Ix. It’s most likely they will do
angiogram. According to the angiogram result, next step will be decided. They may put stents to
keep the arteries opened after ballooning or may go for an open heart surgery if needed.

Angina action plan


1. If you have any severe or prolonged pain after trying to rest, take this sublingual tablet (or
nitroglycerin spray) that I prescribed and wait for 5 min, if not relieve.…
2. Notify a friend or relative that you have the pain
3. If pain persists, repeat the dose and wait for another 5 minutes, if not relieve call ambulance and
take a third dose, tell the ambulance that I’m having a heart attack.

Long term management: ABCCDDES


1. Alcohol  cut down alcohol to safe level
2. BP <130/80 as a target, BMI between 20-24
3. Cigarette, quit smoking and refer to quitline…counsellor, nicotine patches
4. Cholesterol and lipid profile…start on statins
5. Diet – ideal diet
6. Diabetes for BSL
7. Exercise, once your condition is stabilised have 30 minutes of brisk walking
8. Stress  remodel the stress
9. Follow up and reading material

Sexual life: Angina by itself is not affecting your ability to have intercourse, but sexual excitement
can lead to anginal pain. I’d advise you to avoid this until we stabilise the case.
Driving: carry your medication, for any pain while driving, – stop and don’t continue driving.

Will I have heart attack?  The angiogram is to assess the risk of having a heart attack.

! 1!
5 CARDIOVASCULAR SYSTEM

AMI with Cardiac arrest


You’re an intern in ED. A 55 yo man, who was brought to the ED by his wife with a severe
central chest pain, breathless & sweaty.

Task: further relevant Hx, Dx, discuss DDx with examiner, Mx and answer examiner’s Qs.
(Examiner will ask Qs after 3 minutes)

DDx:
Cardiac causes:
- Acute coronary syndrome
- Aortic dissection
- Pericarditis
- AAA
Respiratory causes:
- Pneumothorax
- Pneumonia
- PE
- Pleurisy
Less likely in this case is gastroesophageal causes such as
- Oesophagitis
- Gastroesophageal reflux
- Oesophageal spasm
- Peptic ulcer
Musculoskeletal causes like:
- Costochondritis
- Fracture rib
- Herpes zoster
Psychiatric condition:
- Anxiety
- Panic attack

Part I Chest pain


Is my patient haemodynamically stable?
Severe chest pain  morphine 2.5 mg IV (up to 15 mg); Maxolone 10 mg
I’d like to check the vitals, attached pulse oxymeter to check the oxygen saturation and give
him oxygen; hook up to ECG machine

Hx:
Where is your pain exactly? Central chest
When did it start? How long did it last? Did you take anything for it?
What were you doing when it happened?
What kind of pain?  Pressing type
Does it travel to your neck, arms and hands? to L.arm and hand
How bad? Anything make it better or worse?
Any similar episodes before?  on and off but subside by itself
Any pain in deep breathing (pericarditis)?
Any problem in digestion? Reflux, black bowel motion?
Is the chest sore to touch?
Pain in your calf while walking?

! 2!
CARDIOVASCULAR SYSTEM

Any PMHx like high blood pressure, DM, high cholesterol, heart problem?
Are you on any medication? No
Any Hx of stroke or bleeding problem?
Any FHx of heart problem? Father died suddenly at 60 years
SAD

O/E:
GA, BMI, VS, Heart exam (murmur, rhythm, signs of heart failure)

Ix:
ECG: Lead II, III, AVF (inferior); V1, V2  anterior; V3, V4  septum
Lead I, AVL, V5, V6  lateral
Blood tests: cardiac enzymes (troponin, CK), FBE, BSL, lipid profile, uric acid, U & E

Mx:
Oxygen high flow 8 liter
Aspirin stat if not given by ambulance  300 mg
GTN (glyceryl trinitrate) or Anginine 300 microgram sublingual (check systolic BP: if >100,
can repeat every 5 minutes, maximum 3 dose of half a tablet  1.5 tablet)
Call the senior medical officer

Part II – Cardiac Arrest


Basic life support protocol when the examiner tell about the arrest
While doing ECG, the patient collapsed.
- Danger
- Shout for help (Code blue)
- Shake the patient
- Airway  Head tilt or chin lift
- Breathing  chest movement
- Circulation  feel the carotid pulse
- CPR  30 compressions, 2 breaths (5 cycles over 2 minutes)
ECG show Ventricular Fibrillation
- Defibrillation  monophasic: 360 joule 3 shocks every 2 minutes (biphasic: 200-360-
360) (If defibrillator available  do Advanced Life Support  if not available, I’d like to
start the BASIC LIFE SUPPORT until the help arrive.)
- Give Adrenalin 1 mg IV every 3 minutes
- Check ECG

Part III - Patient gained consciousness


Check his BP --- low BP80/60
Patient will go for PCI (percutaneous coronary intervention: catheterization & angioplasty) or
fibrinolysis.

Indication for reperfusion therapy (PCI):


- Persistent STEMI of 1 mm in 2 contiguous limb leads or
- STEMI of 2 mm in 2 contiguous chest leads or
- New bundle branch block

! 3!
CARDIOVASCULAR SYSTEM

A patient came to you in ED with a chest pain, typical for acute coronary syndrome
- DR ABC secure IV line
- Give Oxygen 4-6 L
- Aspirin 300 mg (provided not hypersensitive)
- Morphine 2.5 – 5 mg IV (30% have nausea and vomiting)
- Maxolon (Metoclopramide) 10 mg IV
- BP >90 Glyceryl Trinitrate 400-600 microgram sublingual or spray
- ECG
- Pulse oxymetry

Patient collapsed
DR ABC
Call for help
Assist airway
No breathing  CPR 30:2 until defibrillator arrives
Assess the rhythm  Ventricular fibrillation  shock and continue CPR 2 minutes (5 cycles
= 200 compressions)
Assess the rhythm, if sinus rhythm  stop

Shock  respond can be VF or PVT


If Pulseless Ventricular Tachycardia (PVT)  shock again and continue CPR for 2 minutes
Assess rhythm again and shock  continue CPR for 2 minutes
If still not responding (after 4 minutes)  give Adrenalin 1 mg (after the second cycle),
continue CPR, shock again
If no sinus rhythm after the third cycle  give Amiodarone

Pulseless electrical activity


Adrenaline and continue CPR
Every 2 minutes assess the rhythm
If after 40 minutes no response  stop CPR

Assess the cause, look for 4 H and 4 T and correct the following (if found):
Hypoxia, Hypothermia, Hypokalemia and/or Hyponatremia, Hypovolemia and
Tension pneumothorax, Tamponade, Toxin, Thrombosis

Monophasic 360 Joule (start with the maximum)


Biphasic 200 Joule

www. resus.org.au

! 4!
6
CARDIOVASCULAR SYSTEM

AAA-counselling
GP setting. 67 yo male came in for check up. You discovered an abdominal mass and sent
him for U/S. It revealed AAA 7cm (infra-renal A.).

Task: counselling and management.

Explanation:
Mr X, I understand you are here for the U/S result. Here is the result and we found out you
have AAA. Do you know anything about it? (Draw a picture.)

It’s not an uncommon condition. We don’t know exactly what causing it. Some of the cases
run in the family. Do you have a family with the same diagnosis? Some may relate to HPT.
Do you have high BP? For how long? Are you on any medication for it? Which one are you
on? When was the last time you check your BP? Do you smoke? Have ever check your
cholesterol level?

Swelling/ dilation of the aorta is just below the renal arteries. The normal is up to 3 cm. 5 cm
is significant and 6 cm is dangerous because it can rupture even with coughing or straining.
In your case the swelling is 7cm. Lucky you we have picked it up now. We will fix it as soon
as possible. We have many patients like this. The surgeon is fantastic here.

My advice is to cancel your travel plan. If you go camping, and with a bad luck if the
aneurysm ruptures then you need an emergency surgery, the risk motality is high 50%. While
the risk of an elective surgery is 5%. I can explain to your wife if you don’t mind.

The surgeon will speak to you in details. Anaesthetist also will speak to you. They clamp the
vessel, stop the circulation and put the stent  Dacron graft on the wall of the aneurysm.
They stitch it together.

I will follow up you. We need to control the BP. Because it runs in the family, you can ask
them to see their GP, to do imaging (U/S to screen for relatives >50 years of age) if they have
the same condition (the first degree relative).

Now I’d like to (call the ambulance to) send you to the hospital.

! 5!
7 CARDIOVASCULAR SYSTEM

AAA- post operative d/w family


Hospital setting. You’re about to see the son of a man who had undergone a laparotomy for AAA.
He’s now in ICU and kept on assisted ventilation, he will be extubated tomorrow. He gave consent
before the operation to talk to his family about his condition.

Task: Talk to the son and deal with his concerns.

Qs:
What’s wrong with my father?
Why is he in ICU?
Why did he undergo operation when he had no illness? He was sitting on a ticking time bomb which
could have exploded anytime without any notice.
How long will he be in ICU?
Why are all these tubes hanging out from his body?
Is he going to die as he is ICU?
Am I going to have same problem? AAA is common in older males especially if there’s FHx of
AAA. As you know, controlling the risk factors (DM, hypertension, smoking, and
hypercholesterolemia) can reduces the chance of getting it.

Explanation:
Mr X, I know that you’re here to discuss the condition of your father. Before discussion, let me assure
you that your father is in a high care place which is the ICU and this is usually a routine place for
major operation patient to have 1 to 1 monitoring and to have the most advanced support, medical
observation needed for his follow-up.
Currently he’s under an artificial breathing by having a machine called the ventilator to control his
breathing and heart and lung function with accurate fluid management. As major abdominal operation
may affect the vital function which is the need for high care.
Your father had an abnormal dilatation in one of the biggest vessel of the body known as the aorta.
This condition is known as abdominal aortic aneurysm. To have the aneurysm, the wall of this vessel
is weakened by a degenerative process involving all layers of the aorta. The lining which is the
endothelium is damaged usually by smoking, HPT or precipitation of high lipids and triglycerides.
This causes the release of enzymes weakening the wall and ends up to the dilatation of the wall.
The size of the aneurysm can lead to more sequelae. Rupture of the aneurysm specifically if >5 cm
can lead to formation of clots known as thrombosis or the bigger the sac the pressure will be on the
surrounding structure.
Because an aneurysm may continue to increase in size, along with progressive weakening of the
artery wall, surgical intervention may be needed. Preventing rupture of an aneurysm is one of the
goals of therapy.
I believe the vascular surgeon made the decision after Ix like CT Abdomen. An elective surgery
which carries less risk than to wait for leakage or rupture of the aneurysm and to have an urgent
operation to be done. Death in elective surgery is 5% while in rupture 80-90%. Repair of this
operation by elective surgery has better outcome and this is why the surgeon went for this operation.
Do you understand this situation Mr X?
There are 2 approaches to AAA repair. The standard surgical procedure for AAA repair is called the
open repair. A newer procedure is the endovascular aneurysm repair (EVAR).
Most likely there is a plan which your father will be extubated or get off the assisted breathing
machine in 1-2 days. The specialist will decide the time to be kept in ICU and he will be discharged to
the ward. He will be in the care of the allied med. team, occupational therapist will put him back to
the normal tract of activity. He will be kept on blood thinning & strong painkiller like opiates to
prevent his blood from clotting. If you need to speak to the surgeon, I can arrange a meeting with him.

Rupturelow BP, shock

! 6!
8
CARDIOVASCULAR SYSTEM

Syncope (Aortic stenosis) (AMC 36)


GP setting. A 52 year old technician is consulting you b/o recent loss of consciousness.
Task: further history, PE, explain Dx and further assessment

DDx:
- Vasovagal
- Carotid sinus syncope
- Arrhythmias ---brady (heart blocks) or tachy arrhythmias
- Structural heart diseases ---valvular lesions, ischemic heart diseases, HOCM
- Epilepsy
- TIA & CVA (stroke)
- Orthostatic (volume depletion due to vomiting, diarrhoea or bleeding)
- Drug-induced  centrally acting hypertensive, ACE Inhibitors (Posture hypotention)
- DM (Hypoglycaemia)

Hx:
Before and during
- What happen exactly? ---playing tennis suddenly collapsed
- Any palpitation, chest pain, SOB, dizziness, vertigo, blurred vision, severe headache, speech
problem, N/V, sweating before collapsed?
- Did you lose your consciousness?
- Was it witnessed?
- How long did you lose your consciousness?
- Any change in the colour during the attack? (cyanosis)
- Did you hurt yourself when you fell? ---head injury, fracture, pain anywhere in arms, legs
- Any abnormal movements during the collapse?
- Did you wet yourself?
- Any previous episodes?
After
- When did you regain consciousness?
- Did you feel sleepy?
- Any headaches, visual changes, altered sensation, pins and needles, weakness in any part of your
body?
- What did you do after you regain consciousness, did you continue the game or went home?
Systemic review
- How’s your health in general?
- Do you usually get chest pain, SOB, palpitations, swelling in your legs, cough, breathing
problems, wheezing?
- Change in your appetite, vomiting, diarrhoea, urine
Risk assessment
- Hx of high BP, any heart attacks, diabetes, high cholesterol, stroke, cramps in the leg,
- FHx of cardiovascular problems or heart attack, epilepsy
- SADMA
- Stress level, Mood  conversion disorder

O/E:
GA, VS (posture hypotension), JVP, Carotid bruit
Heart exam  ejection systolic murmur (3/6), best heard over aortic area
Neurological examination Cranial nerves, motor, sensory

It’s most likely due to narrowing of one of your heart valves. Ix: ECG & echocardiogram.
(AS Syncope, chest pain and exertional dyspnoea)

! 7!
9
CARDIOVASCULAR SYSTEM

Syncope (Wolff-Parkinson-White Syndrome)


David, 24 yo presented to the ED complaining of loss of consciousness and fainting attack 1
hour ago while playing sports with his friends. His friend drove him to the ED and left.

Task: take history, PE, investigation, explain diagnosis & management plan.

Everything is normal, sorry I can’t find anything wrong with you. It could be anything.

Further Ix  FBE, BSL, ECG, U & E, carotid Doppler studies, Echo

You have a condition called WPW Syndrome. Do you know anything about it? It’s
something related to the conductivity of your heart which give you the fainting episode. It’s
not a common condition but sometimes it runs in the family. It can be treated with
medication. Our plan is to refer you to a cardiologist, maybe he will order some
investigations & start you on some medication.

Asymptomatic  observation
Medical treatment  amiodarone, disopyramide (Avoid digoxin, beta blocker, CCB) carotid
sinus massage if the patient is at high risk of AF
Tx  radio frequency ablation of the accessory pathway, implantable defibrillator
Surgical  surgical division of bundle of Kent

Ablation of the abnormal band causing pre-excitation syndrome


ECG-short PR interval, wide QRS complex. Delta wave or slurred wave

! 8!
10
CARDIOVASCULAR SYSTEM

Atrial Flutter (AMC 66)


ED setting. A 50 yo man c/o palpitation & dizziness over the past 3 days. He has not seen a
doctor for the past 10 years. At the last assessment, he was told his BP was elevated. His
current BP is 150/96, symptoms still presented. He’s now lying down in the trolley.

Task: relevant focus Hx. Present a summary to examiner then he will give you the exam
findings. Tell your DDx. Interpret the ECG.

DDx:
Atrial flutter
Atrial fibrillation
Supraventricular tachycardia
Ventricular tachycardia
Thyroid

Hx: Patient STABLE or not


Palpitation (LOTS RADIO)
- Fast beating of my heart going into my neck
- How fast? Around 150/minute
- Duration
- Sudden or not
- Offset ---stop suddenly
- How many times
- Were you doing anything particular when you get it
- Associated symptoms: SOB, loss of consciousness ---mild SOB, sweating, no nausea or
vomiting
- Anything makes it better or worse

Dizziness
- What do you mean by dizziness  near fainting experience like lightheadedness why
heart not contracting well
- Do you have it together with the palpitation

System review
- Thyroid  hot or cold—weather preference
- Weight changes

Past medical history


- Hypertension, DM, Ischaemic heart disease, thyroid disease (risk factors)

Social history
- SADMA  20 cigarettes/day, 5 glasses of wine/day, coffee 5 cups/day
- Job? Sedentary life style

Family history

Summary: (Case presentation AMC P131)


The pt is a 50 yo company secretary who presented with his 4th attack of palpitation &
dizziness over the past 3 days. He had 3 attacks after dinner & 1 whilst driving. Each attach

! 9!
CARDIOVASCULAR SYSTEM

lasts approximately 2 hours, they come on suddenly & stopped suddenly. The nature of the
palpitations is that they appear to be rapid, approximately 150/min and regular. The dizziness
always accompanies the palpitation. Associated with mild shortness of breath & sweating. No
chest pain, no N/V. He has a history of high blood pressure but no knows cardiac disease. He
has a high alcohol intake and has recently been under stress at work.
Conclusion: He is at risk of ischemic heart disease because of hypertension, smoking,
obesity, and sedentary life style. No evidence on Hx to suggest thyrotoxicosis.

O/E:
GA: Overweight, distressed
VS: P 150, regular
Heart exam: heart sounds show dual rhythm with no bruits, no signs of heart failure

Dx: Paroxysmal atrial arrhythmia, probably atrial flutter (sudden onset and offset, the rapid,
regular palpitation and the rate)

Causes: hypertensive heart disease, alcoholic cardiomyopathy, ischemic heart disease, occult
thyrotoxicosis

Treatment  Tx of arrhythmia + prophylaxis against thromboembolic complications


- A – rhythm control  Amiodarone
- B C D – rate  beta blocker, ca channel blocker, digoxin
- Complication  aspirin or warfarin
- Lifestyle modification

Atrial flutter is usually insensitive to antiarrhythmic drugs  cardioversion (direct current


shock) or pace cardioversion

! 10!
11
CARDIOVASCULAR SYSTEM

Infective Endocarditis
GP setting. A middle age lady c/o extreme tiredness and fatigue for the last 3 weeks.
(Fever and hand pain)

Task: take history, ask examiner for examination finding, diagnosis and management.

DDx:
Infection: HIV, ASK Q TO RULE OUT:
Haemochromatosis-addison - ANAEMIA
- THYROID
malignancy
- DM
anaemia - CANCER
DM - DEPRESSION
Hypothyroidism - STD
Depression - DENTAL PROCEDURE (I.E.),
Anxiety IF EVERYTHING IS -VE

Hx:
How long have you had the tiredness?
Any fever?
Any change in appetite, weight loss, fever?
How is your sleep? How is your mood?
Any liver disease? Any change in skin color?
How is your diet? Have you had any blood loss? Bleeding anywhere?
How is your period?
Do you have any weather preference?
Hoarseness of voice?
Any increase frequency of urine?
Do you feel thirsty?
Any FHx of DM, tumor?
Are you sexually active? Do you practice safe sex?
Any STD before? Stable partner?
Any recent surgical procedure?
Any dental procedure done? Any antibiotic taken prior to the procedure?
Did you have heart problem in the past?
Have you been diagnosed with any rheumatic fever?

Positive findings for this case:


- History of dental procedure
- Heart murmur
- Haematuria on urine dipstick
- Splenomegaly

O/E:
GA: pallor, jaundice, petechial haemorrhage
VS: Fever, BP, PR, RR
Signs of Infective Endocarditis:
- Conjunctiva pallor, petechial haemorrhage
- Red, painless skins spots on the palms and soles (Janeways lesion)
- Palmar erythema

! 11!
CARDIOVASCULAR SYSTEM

Hand Examination: no clubbing, Osler node


Lungs: bilateral crepitation
CVS: S1 S2, any murmur, pericardial rub, any signs of heart failure – JVP, leg oedema
Abdomen: Hepatosplenomegaly? Splenomegaly? LN
Urine dipstick: blood (+++)

Explanation:
From history and examination, Ms Smith, you have a condition called infective endocarditis
(fever + new murmur).
Because of your dental procedure, the bacteria mainly Streptococcal (Strep viridians) entered
the blood stream and into your heart. We need to do some more Ix to confirm the diagnosis.
- FBE, ESR, CRP
- 3 sets of blood culture taken 20 minutes apart from 3 different sites and time.
- While doing blood test I will also do the serology for complement level C3, C4, ASO.
- ECG and Echo

I will refer you to the hospital and a cardiologist will assess you
- They will admit you and start antibiotic (Do you have any medication allergy?)
- They will start with IV Benzyl Penicillin + Flucloxacillin + Gentamycin until the blood
culture is available
- You will need to stay in hospital until afebrile then discharge home, the nurse will come
and give the IV antibiotics (total course of IV 2 wks, oral till 6 wks)

Dukes criteria:
Major
- Blood culture positive on 2 separate sets
- Echo: any mobile vegetation or new valvular regurgitation
Minor
- Fever > 38
- Predisposing heart disease
- Positive serology test for C3, C4, ASO
- Echo not normal but not meeting the major criteria
- Vascular phenomenon due to vasculitis

To diagnose :
- 2 major
- Or 1 major + 3 minor
- Or all 5 minor

! 12!
12
CARDIOVASCULAR SYSTEM

Hyperlipidemia
GP setting. A male pt came back for blood test result: cholesterol 7.3 (N 5.5). HDL is normal,
LDL high, BSL normal. He is a manager in a supermarket. He does minimal exercise, eats
junk food, BMI 31, fat in abdomen. He hasn’t had any symptoms but he’s concerned b/o FHx
of AMI. His BP is 134/80.

Task: explain the result, management

Hx:
I’ve got your blood test result and it showed your cholesterol is higher and LDL is also a bit
high. It’s good that you don’t have the symptoms.
But I know both of your parents had heart attack & you’re a bit overweight which means
you’re at risk
I appreciate you’re concerned about your health. It’s the right time because with lifestyle
modification & with medication, if needed we can lower the risk
Before I talked about weight & cholesterol, I’d like to ask you some questions to know if
there are other risk factors
- I know you’re a manager & you don’t do a lot of exercise
- Is your job stressful?
- SAD
- Any family history of DM?
- The age of parents died with heart attack
Now I’ll talk about the risk of high cholesterol & overweight.
Risk of heart attack, stroke, DM, joint pain, back pain, gallbladder problem, kidney problem,
psychological problem
SNAP
I’ll give advice regarding healthy diet and increase fibre diet, more vegetable & fruit,
less fatty or take away and fried foods
Try to cook by yourself if you can
Do not take biscuits or chocolates or fried sacks in between meals.
Take wholemeal bread, instead of white bread
Take water instead of soft drink
Reduce chocolate & salt
Exercise of 15-30 minutes brisk walking bring change in your health
If you don’t have time, park you car away from your job & just walk to the office
You can use public transport and get off one station before
It’s good that you don’t smoke/drink
Try to reduce your stress at work by giving some responsibilities to others
Don’t try to reduce your weight suddenly, 5-10 kg/year is acceptable

Follow-up
In the first 2-3 weeks, you need follow-up to see whether you need support for diet advice –
refer to dietician
In 6-8 weeks, we’ll do blood tests again for checking cholesterol again and whether you need
medication or not
Give pamphlet

! 13!
13
CARDIOVASCULAR SYSTEM

Cardiovascular Risk Assessment


A 43 year-old female came for a routine check-up to your GP clinic. Her father died one year
ago of heart attack. Her result showed total cholesterol 7.5, HDL 1.5, LDL 1.7.

Task: cardiovascular risk assessment and basic management.

ASK FOR THE CV RISK CHART

Your cholesterol level is a bit high and you have positive FHx, we need to assess your
personal risk of heart disease and discuss management plan.

There’s a special chart to estimate the risk, the New Zealand guideline (ask examiner for the
chart). Before assessing the risk, I need to ask you a few questions
- Do you smoke?
- Do you have hypertension? We need to check your BP now (ask examiner for the BP)
- Are you diabetic?

Mary, here is the chart, there are some criteria:


- Gender  Different chart for male & female
- Diabetic or non-diabetic
- Smoker or non smoker
- Age  you’re 43 it means you’re here
- Your BP is 125/75
- Ratio of cholesterol  total cholesterol/HDL = 7.5/1.5 = 5

Mary, you’re in the blue quadrant, which means mild risk (<2.5%) to develop cardiovascular
disease in the next 5 years

Mx: Your personal risk is mild, but because you have a FHx & high cholesterol, we need to
talk about the management plan
- You need regular follow up for lipid level, BSL
- Diet is important to control you lipid  No junk food, no fish & chips, seafood, cheese
- Keep your body weight normal
- Regular exercise
- Quit smoking
- Control your alcohol intake
- Try to reduce your stress
- ?Start Lipitor (total cholesterol > 5.5)
- Written material
- Follow up

Before starting Lipitor, test:


- LFT
- CK (because of rhabdomyolysis)

! 14!
14
CARDIOVASCULAR SYSTEM

Hypertension with OCP


GP setting. A 25 yo female had BP reading of 150/108 on 3 occasions in the last 3 months. She’s on
OCP. She had no other symptom. PE normal. Her BP was normal 2 yrs ago.

Task: take relevant history, explain management, explain the effects of hypertension.

Systolic Diastolic
Grade I 140 - 159 90 - 99
Grade II 160 - 179 100 - 109
Grade III ≥ 180 ≥ 110
Check BP 3 times within 3 months.

Hx:
When I checked your BP, it was high in 3 readings in 3 separate occasions. So we need to find out the
possible cause
How are you feeling these days?
Have you noticed any chest pain, headache, SOB, heart racing?
Did you have any problem with your kidney or thyroid previously?
Have you noticed any recent weight changes?
How’s your waterworks/bowel habit?
Have you ever check your blood glucose/lipid level?
Do you feel thirsty or do you need to go to the toilet frequently?
Any family history of stroke, DM, hypertension?
Are you on any medication such as painkillers?
How long have you been using OCP? Any complications?
After using OCP, did you go for a follow-up?
When was your LMP?
What’s your occupation?
Any stress at work/home?
SADMA
Do you think you eat a healthy balance diet?
Do you do regular exercise?

Explanation:
I couldn’t find any abnormality except the OCP.
OCP can cause hypertension. So I’d like to advise you to stop it and use other kind of contraception
like condoms. I’ll check your BP after 2 weeks. If BP goes down, the cause is OCP. Meanwhile I’ll do
some investigations such as FBE, BSL, Lipid profile, U & E, LFT, TFT, Urine analysis.
If the BP doesn’t go down, the result shows abnormal, we need to start treatment.
At the moment, we don’t need any treatment.
First of all, you need to start a healthy diet.
Do exercise 30 minutes/day for 5 days.

Now, I’d like to explain about the effects of hypertension.


Hypertension can cause hardening of blood vessels called atherosclerosis which can lead to dilatation
or narrowing of blood vessels.
Heart  enlargement of heart, heart failure, heart attack
Brain  stroke
Eye  blurring of vision
Kidney  renal failure, damage to kidneys

To prevent these complications, you need to control your BP


I hope you’ll be fine if you stop the OCP

! 15!
15
CARDIOVASCULAR SYSTEM

Hypertension
GP setting. A 35 yo man was found to have a blood pressure of 165/95 on two occasions.

Task: take further history, ask for physical examination finding, and advise management.

Hx:
- Evidence of end-organ damage or complication of hypertension (heart, brain, kidney,
eyes)  indication to start medication from the start
- Primary or secondary
- Risk factors for other cardiovascular diseases

Have you experienced any chest pain, SOB, palpitations, leg oedema
Any pains or cramps in the legs
Headaches, weakness or numbness in the body
Any visual changes or impairments

Phaeochromocytoma
Any attacks of funny turns or pallor, chest tightness, throbbing headaches and palpitations
Cushing
Have you noticed any change in your weight
Any increase hair growth
Any skin changes like
Risk factor
Any PHx of renal problems, kidney diseases, heart attacks, DM
Abnormal cholesterol level
Any FHx of cardiac problems
SADMA

O/E: (To see evidence of organ damage and look for secondary cause)
GA: BMI
VS
Radiofemoral delays
Eyes : any abnormality, funduscopy
Face : plethoric (red face in Cushing) or moon face
Neck: any carotid bruit, JVP
Thyroid
Chest, heart, lungs
Signs of cardiac failure
Abdomen : auscultate for renal bruit, enlargement of kidney sizes (polycystic kidney)
Legs : Vascular examination (peripheral) – pulse, color
Urine dipstick

Explanation:
After I have examined you, I couldn’t find any evidence of secondary causes, your high BP is
most likely not secondary to any other problem. It’s a primary or essential hypertension
which accounts for up to 95% of hypertension cases. However, we have to do some blood
tests (to check for end-organ involvement and secondary causes).

! 16!
CARDIOVASCULAR SYSTEM

First line investigation


- FBE
- BSL
- LFT
- RFT
- TFT
- Cholesterol
- Uric acid level
- Urine analysis
- ECG with or without chest X-ray

Second line investigation


- Echocardiography
- Renal US
- Doppler US
- 24-hour urine VMA (for phaeochromocytoma)
- Serum renin-aldosterone ratio

Secondary causes (5-10%)


- Kidney 3-4% include glomerulonephritis, reflux nephropathy, renal artery stenosis,
polycystic disease of the kidneys
- Endocrine : DM, Conn, Cushing, Phaeochromocytoma,thyrotoxicosis
- Coarctation of the aorta
- Drugs
- Pregnancy

Mx:
Starting medication if there is:
1) Evidence of end-organ damage
- LVH detected by ECG, chest X-ray
- Hypertensive retinopathy
- Hypertensive nephropathy
2) If initial diastolic is >110; systolic is >180
3) Failure of non pharmacologic such as lifestyle, etc

Lifestyle modification for 3 months – weight reduction, reduce alcohol, reduces sodium,
increase exercise and stop smoking
Monitor BP on daily basis
Measure BP at the same time

If fails, start medication according to the age & other medical issue of patient  ACE
inhibitor (because can improve renal function too)

Aim of treatment: Monotherapy

ACE Inhibitor – Diuretic – Beta blocker – Calcium channel blocker


For asthmatic patient, start with Calcium channel blocker
Diabetic  ACE inhibitor

! 17!
16
CARDIOVASCULAR SYSTEM

Hypertension: Examination
GP setting. A 30 yo man had 3 high BP reading of 160/90 mmHg recently. He’s generally well and no
other medical problem. He’s asymptomatic at the moment. He smokes 20 cigarettes / d for years.
Task: do relevant PE while giving running commentary to the examiner and management.

Secondary causes of hypertension:TRACK PADS


T – Thyroid disease – signs of hyperthyroidism
R – Renovascular (renal artery stenosis) – bruit
A – Aorta (Coarctation of aorta)
C – Cushing syndrome – look for stigmata (moon face)
K – Chronic kidney disease (eg glomerulonephritis, nephropathy, polycystic kidney)
P – Phaeochromocytoma
A – Aldosteronism
D – Drugs (OCP, NSAID, Steroids)
S – Sleep apnoea

O/E:
- I’m looking for stigmata of Cushing (moon face, buffalo hump) / hyperthyroidism
- Feel the pulse for coarctation of aorta – rate, volume, radiofemoral delay
- Measure the blood pressure – sitting/lying and standing in both arms
- Exam of eye – look for any redness (polycythaemia)
- Funduscopy – any change in the retina, disc, papillary oedema
- Thyroid – look for signs of hyperthyroidism
- Cardiovascular system – locate apex beat, listen to the heart sound for any murmur
- Jugular Venous Pressure, carotid bruits
- Respiratory system examination – any added sound, bilateral basal crackles (congestive cardiac
failure)
- Abdomen – listen for the bruit, any distension, mass, ascites, organomegaly, dilated veins
- Palpate renal angle (for polycystic kidney) and auscultate for bruit (for renal artery stenosis)
- Pedal oedema

Ix:
- FBE
- U, C & E (for kidney disease)
- BSL
- Lipid profile
- Serum cortisol
- Renin & angiotensin ratio
- 24-hour urine catecholamine
- Plasma renin, plasma aldosterone
- LFT
- TFT
- Renal ultrasound
- Doppler ultrasound for renal artery
- Chest X-ray
- ECG
- ECHO
- Renal arteriography

Mx:
Further investigation, may refer
Follow-up
Lifestyle modification

! 18!
17
CARDIOVASCULAR SYSTEM

Obesity
Scenario 1: GP setting. A 55 yo female who was attended ED with BP 150/90 in 2 separate
occasions. Apart from pharmacological treatment, you advised her to lose some weight. She wants to
talk to you about weight management. Her BMI is 41.
Task: take history, explain health risk of being overweight, counsel regarding obesity.

Scenario 2: A 20 yo female university student, her BMI is 35. FHx of DM. All Ix normal.
Task: explore the risk of obesity and management.

Scenario 3: A middle-aged lady, BMI 45, saw you 1 year ago for joint problem. X-ray showed
degenerative changes. Advised her to lose weight by lifestyle & dietary changes but it didn’t work.
She’s here to find out if there are any other options for her.
Task: take relevant history & management.

Normal BMI 20-25; 26-30 Overweight; >30 Obesity

Critical errors:
1. Show no basic knowledge of pharmacological and/or surgical methods of Mx obesity
2. Tell the pt that she’s not a candidate for pharmacological or surgical Tx.
3. Talk only pharmacological or surgical methods without emphasising that lifestyle
intervention is very important.

Hx:
When pt gained wt
Reason for the gain
Duration of being overweight
Previous attempts of loss wt  Number of times, whether successful, what methods used, what was
helpful, reasons behind gaining wt again
FHx of obesity, related disease and risk factors
Dietary information, physical activity, social background, support and medication
MHx: complications of obesity, assess CVS risk factors, r/o endocrine reasons

What do you eat, please describe your one day diet


Do you prepare the food at home or buy fast food
Any snacks in between
Work, what do you do for a living
Are you actively exercise
Habits – SADMA
MHx: apart from hypertension, do you have diabetes, hypercholesterolemia
FHx: anyone in your family have any medical condition ---Mother has a stroke, Father also stroke &
in nursing home
Have you ever tried to lose weight?
What did you try?
Did you try exercise? How much?
Diet…what kind of diet
Have you check your sugar level or lipid level before

Health risks of obesity:


1. Cardiovascular system
a. Stroke
b. IHD, heart attach
c. Hypertension

! 19!
CARDIOVASCULAR SYSTEM

2. Metabolic
a. High cholesterol level
b. Type 2 DM
c. Infertility
3. Mechanical
a. Osteoarthritis
b. Obstructive Sleep Apnoea
c. Back aches
4. Others
a. Cancers (endometrial Ca, bowel Ca)
b. Gallbladder disease (stones)
c. Psychological problems

Mx:
It’s a very common condition. I’m very glad that you come back & decided to take action.

Lifestyle modification
You’re putting weight as a result of energy intake more than required. There is no simple effective
way to treat this condition. Our aim to decrease energy intake, increase physical activities
Energy in (food) = energy out (physical activity) + energy stored (weight)
This management should be done by multidisciplinary team, GP, dietician, physiotherapist.
You should set goals for yourself.
1) you should tell yourself no further weight gain.
2) lose wt slowly wt loss of 5-10% of body wt ↓health risk. Make a graph to monitor the progress.
3) increase the activity, sport, tennis, golf, cycling, swimming, walking.
You should limit your alcohol intake to 1-2 standard drink per day.
For diet, ↓fat food (the portion is important), ↑ fibre and vegetables. No sugar in tea or coffee.
No snack between meals.
Eat slowly, chew more (15 times) before you swallow.

Medication (BMI >30, or BMI>27 with complication, failure to lose wt on a program of diet,
exercise and behaviour therapy)
1. ↓fat absorption in the bowel
• Orlistat (Xenical) lipase inhibitor (SE: GIT disturbance)
• Mythyl cellulose (bulking agent)
2. ↓hunger (dopamine agonistsuppress appetite & ↑energy expenditure)
• Phentemine (caution: allergy, palpitation, glaucoma, HPT, pregnant and breastfeeding, high
cholesterol, IHD)
• Diethylpropion
3. Enhance satiety (Serotonin agonists)
• Sibutramine
• Fluoxetin and other SSRI

Surgery Laparoscopic adjustable gastric bands (LAGB), can be covered by medicare


- Last resort, BMI>35, Age 15-45, affect the quality of life, esp. with HPT and DM
- Pt should have commitment for diet after surgery: low fat+sugar, high fibre+protein, no snacks
- 2 weeks recovery period, first start with liquid diet, then soft diet, annual check-up
- >90% effective
- It’s adjustable and can be kept for many years
Risk
- General: infection, clots, organ or tissue damage
- Specific: reflux, vomiting, heartburn

! 20!
18
CARDIOVASCULAR SYSTEM

Congestive Cardiac Failure (AMC 43)


GP setting. A 53 yo clerical worker presented with bilateral swollen ankles. (Picture given)

Task: relevant Hx, provisional Dx, ask relevant PE findings to confirm the diagnosis.

Left ventricle failure:


- Blood is not coming out to the aorta
- Bilateral basal crackles because the blood flow goes back to left atrium and pulmonary
veins  pulmonary oedema in the alveoli
- More blood to the lung  dyspnea (shortness of breath) on exertion or at rest
- Finally will lead to orthopnea and nocturnal dyspnea
- Will develop to RVF

Right ventricle failure


- Any problem in lung – COPD – pulmonary artery pressure increase go to right ventricle –
right atrium - cause elevated JVP
- SVC – to the liver  hepatomegaly
- IVC drain from the abdomen  ascites
- Pedal oedema

Hx:
What brings you here today? (I’ve come to see you doctor because I have had swelling in my
ankles for about 2m and usually gets worse in the end of the day.)
Is it started gradually or suddenly? (gradually)
Both legs? (yes)
Any pain? Any skin colour changes? (no)
Swelling anywhere else apart from your legs ? (no)
Do you get SOB? (become breathless when walk up stairs/ walk fast and this pass when rest)
How much exercise can you do before you get SOB?
How many pillows do you use? (only one)
What about at night, do you get up from sleep and fell SOB? (no)
Do you fell your heart is racing than usual or you fell palpitation? (Yes, for the last few yrs)
Have you had any dizziness or black out ? (no)
Do you have any cough, sputum, blood in the sputum or fever? (no)
Have you had any chest pain, or chest pain on exertion? (not now)
Did you have any chest pain in the past? (yes, I had in the central of my chest but it was 4
years ago, lasted for 2 h, and I felt unwell for a few days afterwards.)
Any black bowel motion?
Weight changes? (no changes, normal weight and has healthy diet)
Skin and urine colour changes, tiredness, wt change, itchiness and bruising. (r/o liver disease)
PHx of hypertension (no)
PHx of rheumatic fever, IHD, DM, hypercholesterolemia, kidney or liver problems? (no)
Medication (no)
Do you smoke? (smoked for 20 years and stopped smoking last year)
Drinking (3 glass of wine/ day )
Job (has to stand for long hours)
FHx of IHD, MI, diabetes (mother died of stroke at age of 77, father died at the age of 90)

! 21!
CARDIOVASCULAR SYSTEM

Explanation:
From the Hx it seems that the swelling of your legs are due to heart condition which is called
cardiac failure. Congestive heart failure is present when the heart cannot pump enough
blood to satisfy the needs of the body. Weakened chambers allow blood to pool inside the
heart and nearby veins. This triggers fluid retention, particularly in the lungs, legs and
abdomen. In your case most likely due to the IHD, MI (the chest pain which you had 4 yrs
ago).
O/E: (You need to ask what are you looking for, but no result will be given)
I am looking for signs of cardiac failure and underlying causes, kidney and liver disease as
well as other causes of swelling like venous thrombosis , bilateral DVT,cellulitis.
(Bilateral basal crackles; Hepatomegaly; Ascites; Bilateral pitting pedal oedema)
GA: face swelling, dyspnoea (SOB), Mouth cyanosis
VS: BP, T, RR, Pulse rate and rhythm
JVP distension, pulse and pressure, carotid bruits
CVS:
- Inspection: apex beat displaced or not
- Palpation: apex beat, heave thrill
- Auscultation: cardiac sounds ,murmur, rub
RS: crackles at the base of the lungs, and look for signs o effusion.
Abd: hepatomegaly, hepatojugular reflex, splenomegaly and signs of ascites,
Check inguinal area for enlarged lymph nodes (enlarged LN cause pressure to the veins)
Palpate kidneys
Lower limbes exam: Oedema (pitting), symmetry, how match, Discoloration, Tenderness (r/o
DVT), Temperature, Varices
Urine dipstick

Ix:
ECG, echocardiography, CXR
Blood tests: cholesterol, FBE, BSL, U & E,BNP

Tx:
- Diuretics (frusemide)
- Spironolactone
- Beta blocker
- ACEI
- Digoxin

! 22!
19
CARDIOVASCULAR SYSTEM

Pericarditis
You’re a GP. A 50 year-old tram driver came with chest pain radiating to the back and
shoulder. He had flu few days ago.

Task: take history, examination, diagnosis, management.

Hx: Ask examiner if the patient is stable or not.


Are you in severe pain? Do you want a pain killer before we proceed?
Could you please tell me where exactly the pain is?
How severe is it?
Does it go anywhere else?
Is it related to any activity or effort?
Could you please describe the pain?
How does it feel like? Sharp? Pressing in nature? Dull?
Any relieving factor?
Anything which makes the pain worse?
Does it change with posture?
Any associated factors: fever, N/V, sweating, dizziness, heart racing, cough, SOB
Any previous chest infection, night sweat?
Have you ever been diagnosed with TB?
Any contact with people with TB?
Any trauma to the chest?
Any loss of weight and loss of appetite?
Any history of tumor or cancer, kidney problem? Clotting problem?
Any heart attack in the past?
Are you on any medication?
Any recent long trip?
Any recent operation?
DM, HTN, high cholesterol?

Pain started 2 days ago. Increase by deep breathing & cough, 2 days ago had flu-like
symptoms.

O/E:
GA
VS
Chest exam
Heart exam: the only finding is noise on auscultation of heart.
Ask examiner is that rub?

ECG - ST elevation with upwards concavity in all leads.

Explanation:
You are having a condition called pericarditis which is inflammation of the covering of the
heart probably due to viral infection as you had flu a few days back.

! 23!
CARDIOVASCULAR SYSTEM

I’d like to send you to the hospital and they will do further Ix:
- FBE, ESR, CRP
- U & E, creatinine
- TFT - hypothyroidism can cause pericarditis
- LFT
- Connective tissue disease screening like ANA, Ds DNA (double stranded DNA),
rheumatoid factors
- ECG, Cardiac enzymes, Echo (pericardio-centersis)
- Blood culture if fever

Mx:
Simple analgesic first
If severe, colchicine
Steroid and immunosupressant as last resort

I will refer you to the hospital for investigation and you will be assessed by cardiologist and
decide whether you need admission or not.

! 24!
20
ENDOCRINE
Hyperthyroidism
A 45 yo woman came to your GP clinic complaining of something wrong with her nerves.

Task: take history, PE, Investigation, Management

Hx:
What do you mean by something wrong with your nerves?
Since when?
Do you know any reason or precipitating factor?
Is it the first time?
Any associated symptoms such as tremor, headache, heart racing, weather preference?
How’s your appetite, any weight loss or weight gain?
Waterworks, bowel habit
Any lumps in your body?
Any eye problem, skin problem, hair problem?
Menstrual history, LMP, cycle, any change in pattern (maybe due to menopause, thyroid)
What’s your occupation?
Any stress at home or at work?
General health
Any family history of thyroid problem or cardiac disease
SADMA

O/E:
GA: BMI; Tremor, sweaty hands; Eye signs: lid lag, exophthalmos; Hair thinning
VS: P, rhythm, BP (sitting and standing), T
Lymph nodes
Thyroid – enlarge – smooth, nodular, thyroid bruit
CVS: widespread systolic murmur
Abdomen: mass, organomegaly
Oedema in lower limbs
Urine dipstick

Explanation:
I believe your condition is probably due to hyperthyroidism.
There is a major gland in your neck which produce the thyroid hormones.
Sometimes the gland becomes overactive and secretes more hormones which is causing your
symptoms

Ix:
FBE, U & E, LFT
TFT: T3, T4, TSH
US of thyroid
ECG (Cardiovascular monitoring is important)

Radio isotope scan if needed

I will refer you to an endocrinologist, who will discuss the management options with you:
- Medical : Thiouracil, Beta blocker, carbimazole (agranulocytosis)
- Surgery: Partial or Total Thyroidectomy

! 1!
21
ENDOCRINE
Hypothyroidism (AMC 35)
A 50 year old patient named Jenny, she did some blood test last week and came back to
discuss with you. Hb 110 (N 120-160), blood film showed macrocytosis, triglyceride 5
(Normal <1.7), cholesterol 8 (Normal 5.5), TSH 25 (0.5 – 5), slow heart rate. Constipation,
lethargic.
Anemia macrocytosis  deficiency of B12 & folic acid (MCH >100)

Task: explain result, diagnosis, outline management.

Critical Error: Failure to interpret the result.

Anemia macrocytosis  deficiency of B12 & folic acid (MCH >100)


Increase demand or not enough absorption.
Intrinsic factor attached to B12 & absorbed in the gut.
Anemia pernicious
There are parietal cells in the stomach. Intrinsic factor will attach to the B12 and absorbed.
For autoimmune disease, the intrinsic factor will be deficient.
Give folic acid and B12.
B12 is given with injection, with oral it won’t be absorbed.

Hypothyroidism
Most of the time is autoimmune disease, associated with each other. Most likely to have
another autoimmune disease.
It might be 2 things, might be B12 or Folic acid, and see which one is deficient.

Explanation:
Look Jenny, I’m sorry to tell you that I don’t have a good news for you. As I told you that we
were suspicious for hypothyroidism. The result showed unfortunately hypothyroidism.
Not an uncommon condition
Thyroid is the gland in front of your neck, butterfly like, very small.
This gland produces hormone called thyroxin.
This hormone is very important to our body, it affects almost each & every cell in our body.
It makes your heart slow down.
It slows down your gut too, so that you have constipation.
It affects your metabolism, it increases the fat in your blood, as you can see, high cholesterol
& triglycerides.

This condition is an autoimmune disease which means that our body secretes some antibody
that normally attack the bugs. But sometimes, it also attack our own cells and this is what
happen in your situation.

The good thing is that it can be treated easily.


Under production of this hormone, you don’t have sufficient of this hormone and have to be
replaced. I will refer you to the specialist. The specialist will give you this hormone from
outside. He will monitor the TSH level, they will monitor the ECG as well. As you don’t
have any heart disease and you’re still young, the specialist probably will start from low
dose. Thyroxine will be given 50 microgram increase until get normal TSH.
elderly and IHD- start fr 25 ug

! 2!
ENDOCRINE
What about my cholesterol?
Because you have under production of the hormone, you have increase fat in your blood.
However, with the treatment, the fat levels in your blood will return to normal. But it’s also
very important that you do regular exercise. Also I’d like to refer you to a dietician.

Do I need to take any medication for it?


Because you don’t have any other risk factors, do exercise and take healthy diet. If
afterwards, the results are still high then we’ll start with medication.

Jenny, other thing that I’d like to discuss with you is your blood result. It showed that you
have anaemia. I think the cause of your anaemia is B12 deficiency. I believe the condition is
called pernicious anaemia which is associated with hypothyroidism. It tends to come together
with hypothyroidism.
Are you a vegetarian? I have to check your B12 level.
Every 2 weeks, IM injection of vit B12. Normal level, increase the period.

What will happen if untreated?


Stroke, peripheral vascular disease, heart disease, slow heart rate, constipation….

How long do I need to be on medication? It’s a life-long treatment

the thyroid will function again? No

What are the SE of the hormone treatment?


We might give you more hormone than necessary, you can have diarrhoea, fast heart rate,
palpitation (atrial fibrillation/arrhythmia), sweating.

!!!!!!!!!!!!!!!!!!!!!!!!!!!!!!!!!!!!!!!!!!!!!!!!!!!!!!!!!!!!!!!!!!!!!!!!!!!!!!!!!!!!!!!!!

! 3!
23
ENDOCRINE
Non Toxic Multinodular Goiter

A 30 yo lady presented to your GP clinic b/o 2 yrs Hx of a lump in the neck


Task: take further history, take focus exam & advise investigation & management.

Hx:
Lump
- Where is the lump
- Onset when did you first notice the lump
- Progression Over this time, have you notice any change in the lump?
- Quick enlargement or gradually increase in size
- Is it painful or sore, any temperature (---thyroiditis)
- Have you noticed any other lumps in your body
Hypo or hyper functioning of the thyroid
- Appetite – (cancer)
- Any recent change in your weight
- Any change in bowel motions
- Are you becoming intolerant to heat or cold
- Any recent mood changes
- Any palpitations
Local compression symptoms
- Cough, any SOB, any hoarseness of voice, any noisy breathing (stridor)
- Any difficulty in swallowing
- Function of the thyroid – any shakes, tremor, any recent nervousness (thyroid toxicosis)
Possible metastasis Headaches; Bone pain; SOB; Chest pain
Female – menstrual history
How’s your health in general
Any past medical history
Any previous operation
Any family history especially thyroid diseases or cancers
Any radiation
SADMA lithium might cause thyroid problems; amiodarone
Diet: Goitrogenic diet ---cabbage family, turnips, broccoli, Brussels sprout, fish

O/E:
GA: BMI, hand tremor, skin (hypothyroid---dry), eye signs: exophthalmus, lid retraction, lid
lag, ophthalmoplegia (sign of Graves disease)
VS: P, rhythm, T
Examine the lump
- Inspect from the front
- Give a glass of water, 2 sips, to see if it’s moving with swallowing
- Is it one side, move or not
- Examine thyroid from the back
- Drink 2 sips of water if the gland move upwards with the swallow
- Palpate each lobe, push by hand and feel the lobe with the other hand
- Palpate the surface, consistency ---soft, firm, hard, any tenderness, any skin temperature
change, possible thrill
- Palpate cervical lymph node
- Percussion ---over the sternum for retrosternal extension
- Auscultation

! 4!
ENDOCRINE
- Pemberton sign, ask patient to extend the hands and see if there is any change in the face
CVS heart sound, murmur
Neurological exam: ankle jerk ---delayed relaxation in hypothyroid

Explanation:
After examination, it seems that it’s an enlargement of your thyroid gland, a gland situated on
the top of the voice box. An important gland, secretes the thyroid hormone that controls a lot
of functions in your body. There are a lot of reasons for the thyroid gland to be enlarged. To
know the cause I have to do some tests:
- TFT ---in this case it’s euthyroid
- Thyroid ultrasound ---multinodular or 1 nodule, solid or cystic
- FNAC from the biggest nodule or from the 2-3 biggest nodules
- (The radio iodine uptake is important in solitary thyroid nodule)
- For retrosternal extension check with chest CT scan

The result is back. I have good news that it’s non malignant. There is also investigation which
showed that there is no malignancy. It’s called Non Toxic Multinodular Goiter, it’s a very
common condition especially in ladies around this age. Possibly it’s related to a reduced
intake of iodine. Some food stuffs may cause it. Sometimes it runs in families, because of
abnormalities in TSH, called familial endemic goiter.

The treatment actually is just watching the gland, involving frequent visits. There’s a small
chance that it may grow to a toxic goiter.

If symptomatic, the treatment is by surgery:


- If developed any pressure symptoms, or
- Become thyrotoxicosis or
- If the FNAC showed a suspicious lesion, or
- If you want to move it for cosmetic reason

Advised also increase iodine in the diet.

Pemberton's sign is the development of facial flushing, distended neck and head superficial
veins, inspiratory stridor and elevation of the jugular venous pressure (JVP) upon raising of
the patient's both arms above his/her head simultaneously, as high as possible. A positive
Pemberton's sign is a sign of superior vena cava syndrome, possibly from a mass in the
mediastinum, such as a tumor or goiter (thoracic inlet obstruction due to retrosternal goitre or
mass).Apical lung cancers often cause a positive Pemberton's sign and a high index of
suspicion should be maintained in patients with symptoms of dyspnea and facial plethora
with an extensive smoking history.

! 5!
24
ENDOCRINE
Thyroid Nodule
A 50 year-old lady came to your GP clinic because she noticed a swelling on her left neck for
the last 3 weeks. This lump moves with swallowing.

Task: manage the case.

DDx:
Multi nodular goiter
Cyst
Adenoma
Carcinoma

Most likely you have a thyroid nodule.


Thyroid is a butterfly shaped gland in front of our neck.
It is a small gland with two lobes.
The gland is important for our body because it secrets some hormones which regulates some
functions of cells in the body.
It could be a single nodule, multiple nodule, cyst or in some case could be cancer.
Management depends on the type of nodule.

MNG- multi-nodular goiter


- Toxic  T3 and T4 increase, TSH decrease
- Non-toxic  T3, T4, TSH normal

For confirmation I’d like to do some tests.

1) TFT – hypo/hyperthyroid (T3, T4, TSH)


2) Ultrasound
- cystic, solid
- single, multiple
- any suspicious features presents or
- only lymph node
3) FNAC  cystic
FNAB  solid
- we will take tissue from the nodule for sample
- we will check this tissue under microscope to get an idea what kind of tissue it is
4) Isotope scan - function of thyroid nodule. Three types :
- cold means non functioning could be cancer
- hot means hyperfunctioning - could be adenoma and
- warm means over functioning, Graves disease
5) CT scan of head, neck and chest  to detect metastasis

MANAGEMENT
Management depends on the test results and the size of the nodule.

If cyst  aspiration can be done


If adenoma  total thyroidectomy/hemothyroidectomy

! 6!
ENDOCRINE
If overfunctioning  give medication such as:
- carbimazole
- propylthiouracil
- B blocker
- radioactive Iodine131 or
- total thyroidectomy + thyroxin replacement

For cancer  total thyroidectomy + thyroxin replacement

Complications of surgery
General  bleeding, infection, anesthetic risks
Specific
1) Nerve damage (recurrent laryneal nerve) - hoarseness of voice
 To prevent this, use laryngoscope before and after surgery.
2) Tension hematoma - as thyroid gland is highly vascular. Haematoma is common.
When bleeding accumulate it compress the wind pipe and patient can die because of this.
Take out the suture at bedside, call for help, move to the theatre and open all the layers.
3) Hypoparathyroidism  Hypocalcemia  need calcium replacement
4) Hypothyroidism (thyroxin replacement)
5) Thyroid storm

I will see you when your test results come back.


Offer reading material

! 7!
25
ENDOCRINE
Papillary Thyroid Carcinoma
A 20 yo female came to your clinic c/o neck lump in the thyroid gland. You examined her
and did all Ix. FNAB came back confirming the diagnosis of papillary thyroid carcinoma.

Task: Talk to the patient about her Dx, give appropriate reassurance, Mx

Explanation:
I’m sorry to tell you that you have a nasty growth in your thyroid gland and the result came in
telling that there is a papillary thyroid carcinoma
This is the most common type of cancer in the thyroid
Unfortunately it can affect all ages especially the young people
It’s a slow growing malignancy but it can spread to LN.
The good news is in your case, there’s no LN involvement. You came early and the prognosis
is good

Would you like me to continue or shall we make another appointment?


Mrs Smith, I’d like to ask you some questions
Did you experience any difficulty in breathing or swallowing?
Have you noticed any change in your voice?
Have you noticed any lumps in your body other than this?
Does anyone in the family have the same problem?
Did you have a previous exposure to radiation

I will refer you to the surgeon


There are 2 types of operation
1. Conservative surgery – take off the lobe related to the lump, isthmus and leave 40% of
the other lobe
2. Radical surgery – take all the whole gland and lymph nodes

Complication of the operation


- Anaesthesia
- Bleeding
- Infection
- Injury of recurrent laryngeal nerve and will affect your voice
- Hypocalcemia (from the parathyroid gland)

Radiotherapy: Iodine 131 if the tumour invade the underlying structure (laryngs, trachea).

Maintenance treatment:
Thyroxin tablet to compensate
This tumor is TSH dependent, when the thyroxin level is normal, TSH will be kept low (if
TSH is high, there is a chance of recurrence of this papillary ca because this is a hormonal
dependent cancer).
Prognosis- better in young female
Follow-up
- CT scan on the neck, chest and bone (bone scan) for metastasis
- Blood test (serum thyroglobulin)
- Yearly check-up for 2 years

! 8!
26
ENDOCRINE
Type 2 DM Counseling (AMC 73)
A 35 year-old lady with BMI 35 came to your GP clinic for the result of her random blood
sugar which is 15 mmol/l. She is on ACE inhibitor and thiazide diuretic. She had not had any
signs and symptoms of DM.

Task: explain diagnosis and management.

Critical errors:
Not stopping medication
No telling to monitor BSL for 3 times a day

Your blood test result showed that your blood sugar level is higher than normal.
I suspect you may have DM.

I will explain to you what is DM, what is the cause, risk factors, how to manage it, how to
control it, how to prevent the complication.
During my explanation, if you have any questions, please feel free to stop and ask me.
At the end of the consultation I will give you the reading materials.

There are 2 types of DM, type I and type II


Type I usually happens in childhood/early age.
Type II is due to insensitivity of body cells to insulin hormone, to vegetate blood sugar in our
body which means insulin resistance, we called this “insulin resistant DM”.

Why do I have diabetes?


Causes could be familial, diet, lack of exercise. In your case, medication could be the cause.

It is not a curable disease, but controllable.


Don't worry we can control diabetes very effectively and prevent complication with your
cooperation and specialist help.
Many people are living their normal life with DM.
People might have some symptoms like polyuria which means frequent waterworks and
thirsty (polydipsia).
It could affect many parts of our body or could be asymptomatic that means no sign or
symptoms.
I need to arrange some tests to confirm the diagnosis
- OGTT  Fasting blood sugar + 2 hour after meal
- Fasting Blood Suger
Baseline test:
- FBE
- RFT
- Lipid profile
- HbA1C
- Urine microscopy and culture

Mx:
1. Stop diuretic
2. Control blood sugar
3. Regular monitoring of blood sugar 3-4 times a day by glucometer and HbA1C every 3
months by doctor. RFT, urine dipstick

! 9!
ENDOCRINE
4. Lifestyle modification
- smoking, nutrition, alcohol, physical activity (BMI)
- BP control
- cholesterol control
- reduce stress
- small cut

First, we will control your blood sugar, next we will aim to prevent complication by
multidisciplinary team.

You need to take healthy diet, avoid junk food.


I can see your weight is above normal. You need to reduce your weight by exercise and diet.
If you want, I can refer you to a dietitian for healthy diet advice.

If you have a dog, take the dog for a walk

If you have a car, park the car in the station and use public transport.

I’m going to refer you to a specialist to assess your condition. You need to stop diuretic and
switch to another medication.

You need to visit your GP regularly


You need to check your BP, lipid level regularly.

If you want I will refer you to a support group.

To prevent complication, you need to check regularly:


- Eye  refer to ophthalmologist
- Foot care  podiatrist
- Kidney  nephrologist
- Skin care  skin specialist

Regular follow up
Reading material
Red flags  hypoglycemia

If you are not able to control your blood sugar with lifestyle modification in 2-3 months, you
might need to start on medication.
After that, you might need to take medication to control your blood sugar level.

My friend is using insulin; do I need to use insulin?


Very unlikely because your diabetes is type II. But sometimes to prevent complications you
might need to take insulin. For you, lifestyle modification and medication are first.

Diabetes is diagnosed as follows


1. If symptomatic (at least two of polydipsia, polyuria, frequent skin infections or
frequent genital thrush).
fasting venous plasma glucose (VPG) ³ 7.0 mmol/L
random VPG (at least 2 hours after last eating) ³ 11.1 mmol/L
2. If asymptomatic:
At least 2 separate elevated values, either fasting, 2 or more hours postprandial, or the 2
values from an oral glucose tolerance test (OGTT).

! 10!
27
ENDOCRINE
Hypoglycaemia
GP setting. A 57 yo man with type 2 DM recently commenced on insulin and you have booked an
appointment for him to a diabetic educator next week. He’s on protaphane (long acting insulin) 20
unit bid. This morning he had a pre episode of being dizziness with nausea, sweating and light
headedness. His friend brought him to your clinic urgently.

Task: relevant Hx, what tests you’d do in your clinic and how would you manage your pt.

Diabetic patient has to check 3 things:


- Check Blood Sugar
- Check insulin dose
- Meals

Hx:
Mr Smith, could you tell me this morning what’s your symptoms and signs exactly?
(compared between the symptoms previously and now)
Have you check your blood sugar regularly like 3-4 times a day?
Did you check your blood sugar this morning?---No
Did you take your insulin today?---Yes
Have you had your breakfast today?----No, I wasn’t hungry
Have you had your dinner last night?
Do you do exercise?---Yes, at least 2 days and yesterday I had a good walking for 1 hour
Any headache, vomiting?
Did you lose your consciousness?
Any weakness any part of your body?
Any speech problem, visual disturbance, difficulty swallowing?
Any recent trauma to the head or falls?
SADMA

Ix: Random Blood Sugar (<3mmol/L is hypoglycaemia)

Explanation:
You have hypoglycaemia, that means your blood sugar level is lower than the normal level. The cause
of your condition is you haven’t checked your blood sugar & you took insulin and you didn’t have a
good & proper breakfast.

I’ll give you glucose drink now and recheck your BSL in 10 mintues.

You need to check your BSL 3-4 times/d, if you find the level is too low or too high most of the time,
let me know and we may need to adjust your insulin dose.
Eating good food regularly, don’t skip meals especially when you are on insulin Refer to dietician
to give an idea of the suitable food. How he can avoid some kinds of food.
Do not smoke. Cutting out alcohol or drinking only a little.
Exercise is important, but do not over stress yourself.

Hypoglycaemia action plan


Keep some Jellybeans in your pocket, if you have any symptoms like dizziness, sweating, tremor,
palpitations, take 6. Don’t repeat unless still unwell after 10 mintues. Follow either a complex
carbohydrate meal.

Glucagon 1ml IM
10-25 ml 50% dextrose

! 11!
28
ENDOCRINE
Diabetic Ketoacidosis
In a countryside hospital. A male 20 year-old came to see you complaining of tiredness.

Task: history, physical examination investigation & management.

Hx:
When did it start
How did it start
Is there anything else you feel at the same time?
Any fever, pain, lump
Any skin changes  color, skin rash
Any weather preference
Any visual problem
Any problem with bowel
Any problem with urination – increase frequency, color – light
Any thirsty feeling
Is your mouth dry
Are you generally healthy
Any chronic condition, asthma, diabetes, any renal problem, liver problem
Any history of STD
How’s your mood, appetite, sleep, sexual problem
Social Hx: Occupation; SAD; Family support
Any family history of diabetes, thyroid problem

O/E:
GA: acidotic breathing, Ketone smell, signs of dehydration
VS: RR increase hyperventilation, BP postural drop, PR, T
Systemic review – normal
Urine dipstick : sugar +++, ketones +++
BSL 16.4

Ix:
ABG to check pH, bicarbonate, potassium
Metabolic acidosis with compensatory respiratory alkalosis
PH 7.2 (normal 7.35-7.45)
pO2 - 96
pCO2 - 28
HCO3 – 14
K+ - increases

Explanation:
I highly suspect you have diabetes based on history, physical examination
First episode with DKA, medical emergency admit to ICU
Start IV line 15-30 ml/kg/hour, first 2 hours
7.5 ml/kg/hour after that
Short acting insulin IV 0.1 unit/kg/hour infusion
Monitor urine output
NaHCO3, sodium bicarbonate 8.4% 70-100 ml only if pH<7 (risk of arrhythmia)

If K>4  no need
If K 3-4  30 mmol/L
If K <3  40 mmol/L
When glucose level <12, we can add 5% dextrose 100 ml/hour

! 12!
29
ENDOCRINE
Addison’s Disease
A 30 yo woman is complaining that her skin has become darkening over her body. She recently loss
weight and feels lethargic.

Task: take history, PE, Investigation and Management.

Causes of Addison’s disease


- Autoimmune (80% in UK)
- TB (most common worldwide)
- Infection
- Cancer (primary or secondary)
- HIV
- Lymphoma
- Histoplasmosis
- Drug – heparin

DDx:
- Addison’s disease
- Hemochromatosis
- Phaeochromocytoma
- Cushing
- Hyperthyroidism
- DM
- Anemia
- Infection
- Cancer

Main Symptoms
- Skin darkening
- Vomiting
- Abdominal pain
- Fatigue
- Postural drop of blood pressure – dizziness
- Weight loss, nausea, vomiting
- Myalgia, arthralgia
- Later – depression

Hx:
When did you notice the skin pigmentation
Could you tell me the distribution of the pigmentation
Is the pigmentation getting darker
When did you start to feel lethargic
Do you have any headache, dizziness
When did you notice weight loss, how much
Any change of your appetite, any nausea, vomiting, diarrhoea or constipation
Do you feel anxious
Sweating, palpitation, tremor, weather preference (to rule out hyperthyroid, phaeochromocytoma)
Do you feel thirsty, do you need to go to toilet ferquently (DM)
Do you have any cough, chest pain, night sweating (Chest infection)
Any contact with TB patient
Any abdominal pain
Period – how’s your period
Are you sexually active, stable partner (HIV)
How’s your general health, any autoimmune disease

! 13!
ENDOCRINE
Are you taking any medication (Drug induced)
SAD

O/E:
GA: pigmentation all over the body, darker on palm and mucous membrane of mouth, BMI
VS: BP always fluctuate, sometimes postural drop
Thyroid, lymph node
Systemic
Abdomen – organomegaly
Urine dipstick

Ix:
FBE, HB low
BSL low
Urea & Electrolytes – Na low, K high, Mg high, Ca high
LFT high
ACTH stimulation test (not for pregnancy, OCP)
Serum cortisol level low
CT scan abdomen
Chest X-ray (to exclude TB, Pneumonia) – no need in this case
Abdominal X-ray for calcification

Explanation:
From the history & examination, you’ve got a condition called Addison’s disease, which is not
uncommon.
There is a problem in one gland of our body called adrenal gland which is responsible for secretion of
some hormones (cortisol, aldosterone)
(Destruction of adrenal cortex leads to cortisol, aldosterone deficiency)
It has effect on the cells of our body
I need to admit you to the hospital to have a replacement of the hydrocortisone by the endocrinologist,
20 mg IV in the morning, 10 mg at midnight
If postural hypotension is not controlled, they will give you mineralocorticoid (fluorocortisone)
Hydrocortisone bid lifelong (not given late in the day)

Advice
You need to wear bracelet in your hand
You should carry a cortisol card in your bag
You need to keep a cortisol injection at home for emergency
Do not stop medication by yourself
You need to be followed-up by your GP regularly
You need to visit your GP for any kind of condition (infection, stress)
If you need to do a surgery, you need to consult with your endocrinologist

RED FLAG (Addison crisis)


- Severe abdominal pain
- Dizziness, sweating, palpitation
 you need to go to ED directly

Phaeochromocytoma
- No skin pigmentation
- BP very high, PR high
- Headache, sweating
- Investigation  Adrenalin high

! 14!
30
ENDOCRINE
Phaeochromocytoma (Incidetalomas) (AMC p 303)
A 66 year-old male presented to you with right upper quadrant pain for possible gall stones
and USG was ordered which showed a focal round mass of 5 cm at the position of right
adrenal gland.

Task: take history, Examination, Investigation, Management.

DD:-Pheochromocytoma
-Cushing

10% rule
10% malignant
10% extra adrenal
10% bilateral
10% familial

Symptoms: sweating, headache, pallor, flushing, pyrexia, seizure, visual disturbance,


palpitation, chest tightness, pulmonary oedema, abdominal pain, constipation, high BP

How do you feel right now, any pain


How is your tummy pain
In addition to gall stones, the US result showed a mass on the top of your kidney at the area
of adrenal gland
For this reason, I’d like to ask you a few more questions and I’d like to do a few more Ix

Is it cancer?
Well, it’s not clear yet, it just showed a mass
Have you ever felt dizzy, nauseated, heart racing, any funny turns (seizure), any kind of
headache, sweating, weather preference
Have you ever checked your BP, normal or high
Do you have easy bruising
Have you noticed any change in your body weight, appetite
Abdominal pain
Do you feel weak or tired
Have you noticed any increase of abnormal hair growth (Cushing)
Any sexual problem, cramping sensation in your legs
Polyuria, polydypsia (DM) --- more thirsty than before, passing water more frequently
Any family history of thyroid problem, hypertension
MEN syndrome  thyroid, parathyroid, pituitary, adrenal
Past medical history
SAD

O/E:
GA: skin pigmentation
Symptoms of Cushing: Cushinoid face, truncal obesity, abdominal striae, hirsutism, easy
bruising
BMI (body fat distribution)
VS: BP (episodic HPT), PR, Temp
CVS, Chest, Abdomen

! 15!
ENDOCRINE

Explanation:
After history & examination, I couldn’t find any abnormalities, but you have a history of
funny turn. For that reason, I’d like to do some tests:
- FBE
- U & E: normal (Sodium high, K low when has episodes)
- Aldosterone level high
- BSL high
- Serum calcium high
- RFT – urea, creatinine
- ECG, echo
- 24 hour VMA level
- Urine catecholamine level
- Serum cortisol level
- MIBG scan (nuclear scan radioiodine labelled agent)
- CT/MRI of adrenal gland

From the blood test & US report, I suspect you have phaeochromocytoma (a catecholamine
producing tumor).
Normally a part of the gland secretes a hormone called adrenalin but in your case, a large
amount of hormone has been secreted which is responsible for your symptoms
I’d like to refer you to a surgeon who will arrange an operation for you
Before the operation, you need to be prepared with some medications for 1-2 weeks : first
you will be given alpha blocker then beta blocker (to avoid crisis from unopposed alpha
adrenergic stimulation)
Laparoscopic key hole operation
After the operation you need to stay in the hospital for a few more days to monitor your blood
pressure
INCIDENTALLOMA < 2 CM:-> MONITOR ACTIVITY AND SIZE.
IF >5 CM:-> SURGICAL INTERVENTION EVEN IF NOT SYMPTOMATIC.
REFER TO SURGEON, ENDOCRINOLOGIST.

INVESTIGATIONS:
FBC, UE, BSL, TFT, CALCIUM, PTH
24 URINARY CATECHOLAMINES, 24 HR URINARY VMA
S. CORTISOL, S. ALDOSTERONE, RENIN. RENIN/ALDOSTERONE RATIO.

ECG, ECHO

MIBG LOOK FOR ACTIVITY IN OPPOSITE ADRENAL GLAND.

PREPARE FOR SX:


- 1-2 WEEK CATHECHOLAMINE SUPPRTION TO PREVENT LIFE THREATENING INTRA OP ADRENAL CRISIS.
- GIVE ALPHA BLOCKER FIRST: PHENOXYBENZYLAMINE
- THEN BETA BLOCKER: PROPRANOLOL

INTRA OP CONTROL OF FLUCTUATING BP WITH NITROPRUSIDE, PHENTOLAMINE.

USUALLY LAPAROSCOPIC SX.

CONTRAINDICATIONS TO A LAPAROSCOPIC APPROACH:


VERY LARGE TUMOURS, MALIGNANT TUMOURS, OR EVIDENCE OF TUMOURS AT MULTIPLE SITES

! 16!
31
ENDOCRINE
Massive Weight Loss
A 45 year-old female came to GP clinic complained that she lost 7 kg of weight over the last
6 months. Appetite is good, no other abnormalities.

Task: take history, examination, DDx, Investigation.

DDx:
- Malignancy (lung, GIT, lymphoma, leukaemia, melanoma)
- Autoimmune
- Endocrine/metabolic (DM, thyroid)
- Infection (TB, HIV, atypical pneumonia, infective endocarditis)
- Psychological (Anorexia, eating disorder)
- Decrease intake
- Malabsorption (Coeliac, IBD)
- Menopause

Hx: (head to toe)


When exactly did you notice that you’re losing weight
Did you notice any recent change in appetite since then
Any change in your mood
Any change in your bowel habit
Any abdominal pain
Any blood in stool
Any lump in the body
Any fever, night sweat
Any vomiting
Any recent travel
Have you noticed any excessive drinking of water or passing more water
Did you eat more than you usual
Any irritability or tremors in the hands (thyroid)
Any hair loss (thyroid)
Did you find it difficult to flush your stool (Coeliac)
Any change in your period
Any chronic medical illness
How’s your waterworks
Any skin rash
Are you sexually active
Are you in a stable relationship
Any recent diagnosis with STD
Any family history of malignancy
Any other GIT problems like ulcerative colitis, Crohn’s disease
Do you drink alcohol, how much, how long
Smoking, how much, how long (bronchial carcinoma)
Are you on any medication
Any longstanding use of any medication
Pap smear and mammogoaphy

! 17!
ENDOCRINE
O/E:
GA: ask everything 
- Alert, anemia, jaundice, any rash, any lymph nodes, any clubbing, BMI
- I’d like to check for any skin mole or any new mole in the body and scalp
- Examination of the throat – any lump or swelling under the tongue, any malignancy
Thyroid
Breast
Full respiratory system: equal breath sounds, any wheezing
CVS
Abdomen
PV
PR

Any office test available – random BSL

Ix:
- FBE
- U&E
- CRP
- TFT, LFT, RFT
- Chest X-ray
- ECG
- US abdomen
- Gastroscopy
- Colonoscopy
- Fecal Occult Blood
- Tumor markers
- Calcium (osteoporosis)
- Magnesium
- CT abd,pelvis

! 18!
32
ENDOCRINE
Pleomorphic Parotid Adenoma
A picture of swelling over the left cheek, the patient said that the swelling is already there for 5 years.
Not painful. Firm on palpation, not lobulated.

Task: examine the patient and tell DD, manage the case.

Causes: Alcohol intake (parotid enlargement); Nutritional

Examination:
GA: looks healthy, active, not drowsy
VS
Clinched teeth  masseter muscle will contract  localization of the swelling (behind the masseter
muscle, in front of the ear)
- Size: 5x5 cm
- Shape: rounded
- Consistency: firm, not lobulated
- Margin: well defined
- Mobility: mobile, not attached with underlying structures
- Temperature
- Bimanual exam of the parotid gland  check the opening  next to the second upper molar tooth
or middle third of the line from trigus to the middle of the upper lip
Lymph node enlargement
Check facial nerve (sit in front of the patient) No sign of facial nerve involvement
- Rise eyebrow to check forehead, push it down
- Close eyes and try to open
- Show teeth  nasolabial fold compared for 2 sides
- Blow and whisper

DDx:
Unilateral:
- Mixed parotid tumour
- Tumour infiltration
- Duct blocking by stone

Bilateral:
- Mumps
- Sarcoidosis
- Alcohol intake
- Malnutrition
- Severe dehydration
- Mikulicz syndrome (all 3 salivary glands enlarged together)

Lump or enlargement of the whole gland


Parotid swelling can be lump (malignant or benign)  to differentiate  look for facial nerve
involvement, consistency, wt loss, rapid growth, LN involvement, painful or painless
 Course, pain, lobulation, involvement of facial nerve
Diffuse swelling
- viral (parotitis)
- blockage (stone)  suppurative or not

Ix:
US (?no fine needle  spreading)
Surgeon  parotidectomy

! 19!
33
ETHICS & LEGAL

Consent For Below Knee Amputation (AMC 120)


A 30 yo man has open crush injury to his L leg in a motor vehicle accident. He’s conscious, no injury
in other area. The orthopaedic registrar has assessed him and immediate below knee amputation has
been recommended. He’s now stable and has been given painkillers.

Task: no need for Hx and exam. Obtain consent from pt and answer examiner’s questions.

How do you feel at the moment?


Before we discuss what should be done, I’d like to ask you a few questions
Do you want me to call your friend or family?
Assess patient’s competency
Do you know what’s today date, month, and year? (time, place, person)
Do you think you can concentrate well, any drowsiness, dizziness?
Are you using any drugs apart from painkillers?
How much damage do you think your leg has?

I’m sorry to tell you that you have a massive wound on your left leg, we call it a crushing type where
some of the vessels, nerves and tissues are completely crushed and not viable.
After discussing with the surgical team & my registrar, it seems that you need an urgent operation
You will be managed by a multidisciplinary effort with a team of specialists
The aim will be to save your leg by all possible means. Broken bones, tendons, bleeding vessels,
nerves can be repaired, even a completely amputated leg can be replaced. But I’m sorry in your case,
the blood vessels, nerves and tissues were crushed, nearly dead. Some are irreparable and may be too
graved to be salvaged. In such a situation, the team may need to take a decision in the operation
theatre to amputate your leg which may be necessary to save your life.
Before going to the theatre, we need your agreement and sign the consent to do so
Do you want me to explain again or is it clear?
The operation if needed is below knee amputation
With recent advances, this operation is combined with fitting of a prosthesis which is very effective
Most patients with them is fully functioning
You can still follow your compassion with sports

Brian, there are some hazards of not doing the amputation. The dead tissue will release some toxins
and some other products which can have a serious effect on your body like sudden cardiac death, gas
gangrene which is a major cause of death. The infection spread so quickly that is it difficult to control
with antibiotics or medicines.

I’m asking you to give consent as a last resort, that is if everything else fail to save your leg, only then
it will be done. If you still insist to refuse, then the surgeon will not do the amputation, but I want to
confirm again that my explanation is very clear and you are aware how serious it is.
If you need further discussion with the surgeon, it can be arranged.
I’ll tell them about your final decision.

I have some concern about the validity of patient’s consent.


I would like to discuss with my supervisor.
VALID CONSENT
1. Competency of patient – Patient must be mentally competent
- Patient’s ability to comprehend and retain relevant information
- Patient needs to weigh risk and benefit before making a final decision
2. Patient’s understanding
- Consequences of having and not having the operation
- Different treatment options
3. Making decision free of coercion

! 1!
34
ETHICS & LEGAL

Brain Death (AMC 59)


A 38 year-old school principal came to your GP clinic requesting information of brain death because
he needs to make a discussion on it with someone at a seminar. He has already printed out information
about it from the internet.

Brain death
Brain death occurs when a critically ill patient die after being put to a life support. It may be due to an
accident, stroke, or MI. The heart continues to beat but the pt cannot breath anymore. The brain is no
longer functioning. The heart doesn’t need the brain to beat because it has its own system to trigger or
initiates impulse.
In Brain death the person is not alive, not going to recover.

Coma
Coma is similar to a deep sleep with exception external stimuli can trigger the brain to react, either
less or no. No external stimuli can trigger the patient to wake. The patient in coma is still alive. The
recovery is possible.

Vegetative state
vegetative state in which the person has lost higher brain function or cortical function. But the lower
brain function is not damaged. The patient may not be able to swallow by themselves but can still
breath spontaneously. Therefore the heart can beat, respiratory function is still good. Eyes can open
on stimulation but the patient can’t wake up or talk. The limbs can move. HR, RR, BP maintained.
They can cry, get upset, randomly laugh or pull faces. Motor reflexes are present.

Tests: CONDITION 059. TABLE 1. Brain death protocol.

Predetermined criteria before test (make sure to check)


• Body temperature more than 35 °C after resolving the temperature with blankets for 20 minutes
• No drugs which depress the central nervous system (CNS) given for the last 48 hours (longer if
CNS depressants given in large amount or for a long time)
• No neuromuscular relaxants given for the last 12 hours
• No endocrine problems eg hypothyroidism, hypopituitarism
• PaC02 > 50 mmHg  check ventilator
• No hypoglycaemia

TESTS to confirm :
• 1. Pupils fixed and unresponsive to light - CN- 2, 3
• 2. Absent corneal reflexes CN - 5, 7
• 3. Absent pain response in cranial nerve distribution CN- 5
• 4. Absent gag reflex on endotracheal tube movement CN- 9, 10
• 5. Oculocephalic reflexes absent (no 'dolls' eyes' response)
• 6. Vestibulo-ocular reflexes absent (no nystagmus) -CN- 8 COWS
• 7. No spontaneous respiratory response after 10 minutes (patient ventilated on 100% oxygen at a
rate of 4 breaths/min with a tidal volume of 7 ml/kg). Arterial blood gases taken at 5 and 10
minutes.

Diagnosis to be made by 2 doctors independently including the intensive care consultant. Neither will
be a member of the transplant team where organ donation is considered. 2 groups of tests, preferably
separated by 24 hours. The results of examination must be recorded in the case notes or a suitable
devised form.

Organ donation: it should be mentioned in driving license.

! 2!
35
ETHICS & LEGAL

End-of-Life Request From a Terminally Ill Patient (AMC 124)


Guidelines:
1. Refuse to do saying by law in Australia we are not allowed to do it.
2. Empathy
3. Solve the problem- eg pain, palliative care team, make the pt comfortable
4. Palliative support group
5. Tell them the truth

Classification
1. Suicide Self-killing by means such as hanging, drug overdose or carbon monoxide
poisoning. No involvement of others.

2. Physician-Assisted Suicide Provision of means for patient to kill themselves, such as a


prescription for self-poisoning, or insertion of an intravenous line for a patient to inject
lethal drugs. Requires involvement of doctor.

3. Passive Euthanasia I Refusal of treatment by competent person Refusal of antibiotics in


advanced malignant disease, or advance directive refusing resuscitation. No direct
involvement of others.

4. Passive Euthanasia II Withdrawing or withholding life-sustaining treatment from


incompetent patient . May require involvement of others.

5. Active voluntary euthanasia  doctor involves voluntarily, active participation, assisting


patient to die

6. Active non-voluntary euthanasia

7. Doctrine of double effect give lethal dose of one drug

PASSIVE Euthanasia
- Refuse to take medication
- DNR  do not resuscitate
- Stopping life support

The doctor has to respect the wish of the competent patient. The best interest of the patient 
dying with dignity.
Stop resuscitation if there is no improvement.
Take consent from family.

! 3!
36
ETHICS & LEGAL

Domestic Violence
Your next patient in general practice is Fiona Cresp, a 25 year old mother of 2, well known to
you. She has visited you 4 times in the last 6 months. First time she came with 5% burn on
the left hand, second time she came with some injury on the right hand. The other two times
she came with a complaint of tiredness for which you investigated her thoroughly and
everything was normal. At the time you counselled her accordingly and advised about life
style changes and stress management.

This time she has come with a complaint of an injury to her head.
Task: focused Hx, examine the pt, discuss Dx and Mx the pt.

HOPC:
“Two days ago Fiona accidentally hit her head on a door frame when she was rushing around
the house chasing her son. She did not think much about it and thought it was not bad,
however, today the area appeared quite swollen and painful. She thought it might be better to
have it checked out and to have some antibiotics”.
If the candidate asks more detailed questions Fiona will admit that she actually was hit by a
kitchen plate which her husband had thrown in an anger tantrum and she was in the wrong
spot and got hit. She assures you that her husband did not mean to hit her with it but that he
just became angry because he was told on that day that he lost his job as a personal care
assistant. He got drunk when he came home, became very angry and started to throw things
in the kitchen, when Fiona got hit by a plate he was throwing. Fiona believes he did not want
to hurt her, but he has had a history of being short tempered and often has arguments with her
and also with people at work and in other situations.
If the candidate shows empathy and asks further questions you can tell her/him that he
actually often becomes aggressive towards you, especially when he drinks alcohol and over
the last 6 months he gets regularly drunk at least twice a week and on a number of occasions
he has hit you, causing bruising at several sites of the body. The beating started shortly after
the birth of the last child when the family faced financial problems because you couldn’t go
back to work but had to care for your child. Your husband also lost several jobs because of
his aggressive behaviour and most of your friends have withdrawn from your family and you
are very lonely. Even his and your parents have become disenchanted because of his
argumentative and aggressive behaviour.
You haven’t spoken to anybody about the situation because you had hoped that it would
rectify itself and be only short lived, because when your husband is sober he promises
regularly that he will change and improve.
PHx. + FHx.: unremarkable
SHx.: You live with your husband and 2 children, 3years and 8 months respectively. Your
younger child has got cerebral palsy following premature birth and some hypoxic event
during delivery. You are aware that the prognosis is not very good and hence stopped taking
him to the hospital. You take care of him at home though you are very busy with his care but
you can save some money. He has not developed any complication yet, but his limbs are very
stiff. You feed him by spoon and he takes that and he has settled in at home quite well. You
used to work as a PCA but stopped working since your delivery. Your husband does house
keeping job but has lost that too 2 days back (the same day you got injury). He is short
tempered and had argument with his supervisor and hence was sacked. He drinks alcohol and
sometimes excessively.

! 4!
ETHICS & LEGAL

You don’t drink alcohol, don’t smoke and you are not on any medication. You don’t have any
suicidal thoughts though you feel depressed. You have lost interest in sex as your husband is
very aggressive in that too. No loss of appetite, weight or sleep. No loss of energy.

Examination : distressed looking lady, vitals stable. There is a 2 x 2 cms laceration present
on the head, looks 2 days old, red, tender and swollen. Multiple unexplained scars on the
head and multiple bruised in different areas of her body are present.
Take photographs of the injuries!

Management :
1. Cleaning and dressing of the wound, pain relief if necessary.
2. Explain to the patient that it looks like domestic violence and that she needs some help.
If patient refuses tell her that she is unsafe in her situation now and there are various
resources available with which this can be stopped and a crisis management plan can be
instituted:

 Offer to organise admission to a refuge


 ensure informed consent for all actions
 consider notifying police (if she agrees)
 show empathy
 build the victim’s coping skills and self esteem
 mention about community sevices
 support services
 women’s support group
 domestic violence resource centre.
 social services/ police/ social workers.

Don’t forget to see her child with child protection services (CPS) and fix an appointment
once this issue get solved.

Possible questions to be asked if suspect domestic violence


 has your patner ever physically threatened or hurt you?
 is there a lot of tension in your relationship?
 sometimes partners react strongly in arguments and use physical force. Is this happening
to you?
 Have you ever been afraid of any partner?

other approach – HELP


H – hear what the woman has to say about her
History – what effect has the violence and abuse had
onset and pattern of abuse
worse case of abuse and greatest fear
E – assess women’s self ESTEEM
L – assess her life situation
does she have regular partner
any supportive people
what is the financial situation
P – PRAISE her efforts so far for whatever she has done

! 5!
37
ETHICS & LEGAL

Domestic Violence
A 13 year-old girl, Sarah, with her Mother came to your GP clinic for certificate because she
has missed school for a couple of days. You noticed the girl has poor eye contact. Physical
examination you found bruise on arms & legs.

Task: take further relevant history, manage the case.

Hx: (confidentiality)
I understand Julia, Sarah does not want to talk to me.
I’d like to talk with Sarah individually---No, I’d like to stay
(If yes  I’ll ask my nurse to be the chaperone while you wait outside and I talk to Sarah)
I’d like to ask why you want a medical certificate for Sarah---she missed her school
Why did she miss her school?---she had flu
Any other reason?---no
During the PE I noticed some bruises on Sarah’s arms & legs. Is this the first time---no
How did she get the bruises?---maybe she hit somewhere
Does she have any blood clotting problem?---no
Any family history of clotting problem?---no
Is she on any medication?---no
How about her general health?---everything is all right

OK Julia, everything we talk here is confidential, except if it cause harm to you or others

Are you living with Sarah’s Father?---no with my new partner


Do you have other kids---Yes, I have a 9-month old boy
Any problem at home, how is your relation with your partner and Sarah?
How does your partner cope with Sarah?---very badly
How is your relation with your partner?---he hit me and Sarah
Does he hit your other kid?---no

Does anyone at home smoke?---yes


Does anyone drink alcohol at home?---yes, my partner
Anyone taking illicit drugs?---yes, my partner, he’s taking marijuana
How about Sarah?---no, she does not smoke, drink alcohol nor take any illicit drugs
How is Sarah doing at school, how is her progress?---not good recently, since the new
partner came
Anything happen recently?---yes, he hit me and Sarah wanted to help me

Explanation:
You’re not the only one who’s suffering from domestic violence. Do you want me to inform
police?
It’s OK Doctor, my partner will change
OK, we’ll see what will happen. If you change your mind, I’m here to help.
But for Sarah, she is still minor, it’s my medical obligation to inform the DHS.

They will organise some tests and they will do some service what is good for Sarah
We need to work in a multidisciplinary team, GP, DHS, and Police at a later stage
I don’t want to inform anyone Doctor

! 6!
ETHICS & LEGAL

DHS will provide safety for Sarah and a counsellor for you & Sarah

Julia, I understand you do not agree to inform the Police


If you change your mind, when you agree to inform the Police, I will help you
If baby  admit the baby to the hospital

DOMESTIC VIOLENCE:
Arguments  Build up  Violence  Repentance  Honeymoon  Arguments

If Violence happen, what you need to look:


- Safety
- Offer  take picture as documents (DHS will take pictures)
- Sometimes not physical, but mental abuse

! 7!
38
ETHICS & LEGAL

Confidentiality (Ethical dilemma)


A 70 year-old man with bowel cancer in sigmoid colon, presented with complete obstruction.
He had an emergency operation. CT abdomen showed metastasis in the liver. Now the
daughter has come to you and you’re the HMO in the surgery ward. She wants to tell the
diagnosis & prognosis to her father by herself.

Task: Manage the lady with ethical dilemma, tell her what to do and what will be the
prognosis to her father and what is the management for the father.

Do you have the authority from your father, the next of kin…
Do you have the consent….
Legal obligation of the doctor to tell the patient
Talk to my consultant…
This is my legal obligation to tell your father
Palliative care – not survive for a long time, as long as he lives he need the palliative care, try
to keep him comfortable life

EXAMINER, does she have the consent from the father…


Do we have the consent from the patient to talk to his daughter….
Good morning, how can I address you, my name is….
I see from the note that you want to talk to me about your father
What is your main concern….
SORRY we can’t allow you to do that because it’s our legal obligation to tell the patient what
happened and what’s going to happen….
In my knowledge in Australia the doctor is legally obliged to tell the patient…..
If you need more information…I will ask my consultant to talk to you…

Look your father had this problem….


From the scan it showed that it has gone to the liver….
Do you know what does it mean….
This is beyond our control, it’s not treatable anymore, he will not live for a long time but we
don’t know until when, we will make him comfortable
The oncology/multidisciplinary team will refer him to the palliative team

Can he go home?
He can go home if it’s manageable, depends on his condition and progress, the palliative
team will see him and decide
He will go back to the aged care facility (low care and high care)
A nurse will visit him at home….

! 8!
39
ETHICS & LEGAL

Confidentiality (AMC)
You’re a GP. Your next patient is Bill, 67 years old, after you finish his annual check-up,
he’s asking you how his wife was. His wife, Ann, is 65 years old, who’s also your regular
patient. Three days ago, she told you that her husband Bill keeps telling her that she was
forgetful and vague. You assessed her and found nothing wrong. Bill also told his son &
daughter that they need to put Ann in the nursing home.

Task: respond to Bill’s question and request about her condition.

I respect my patient’s privacy


I can arrange family meeting to discuss together

How long have you been married


How’s the relationship between you & Ann
Between parents & children
Any financial stress
Any stress in your life
What exactly is your concern
Has there been any accident because of her forgetfulness
Has she been lost
Any incidence with the grandchildren
Any history of fall

The role player will keep on insisting and at the end became abusive

I’m sorry I cannot talk about your wife’s condition because of confidentiality
I understand that you’re very worried about her but I cannot expose her records to you
without her consent. I have some legal obligations towards my patient and I have to respect
the patient’s right. Your worry for your wife is very natural but there are ethical issues
involve here. I’m happy to arrange a joint consultation with both of you where we can discuss
this in further details. I’m sorry I’m not able to help you in this matter. Do you have any other
questions?

! 9!
40
ETHICS & LEGAL

Jehovah’s Witness with Heavy Vaginal Bleeding (AMC)


!
A 31 weeks pregnant lady with heavy vaginal bleeding. She’s conscious, BP 80/50 mmHg. She’s
Jehovah’s witness and refuses blood transfusion. Her husband is overseas.

Task: counsel the patient.

You have a condition called antepartum haemorrhage, where there is heavy vaginal bleeding
The reason for that is because the placenta is in an abnormal position
It can be placenta abruption
It’s a risky situation for you & your baby
Your BP is quite low and you need blood transfusion
The blood transfusion is necessary in order to prevent the high likelihood of death either you or your
baby due to blood loss.

I understand your religious opinions and do respect them but you need to understand that this is a
dangerous situation

We can give you blood products such as:


- the major blood products like :
1. RBC
2. Platelets
3. fresh frozen plasma
- minor blood products like :
1. fibrinogen
2. anti thrombin 3
3. factor VIII
4. factor X

Urgent delivery by C section is the best measure to save mother’s and baby’s life. The fetus may
survive if operation is done immediately but if blood transfusion is refused, the chance of maternal
death is high. The baby does not die of blood loss but of hypoxia.

Synthetic blood substitutes like Haemaccel or Macrodex (volume or plasma expander) are not very
useful because they do not carry oxygen. They can improve the blood volume but cannot improve the
hypoxia. This is the limiting factor.

We may have to do a C section or remove your womb to save your life. We need consent from you.
I’ll try to contact your husband and discuss this matter with him.

I can call the Jehovah’s witness group to be with you.


You’ll have to sign a form for consent for C section and refusal of blood transfusion.

What is the legal issue?


The patient has right to autonomy and self determined her treatment.
In Australia, the fetus has no rights justifying treatment being forced on mother against her wishes.
Once the baby’s born, the scenario changes.

Management of patient:
- Continue with the volume expander
- Urgent C section
- If required hysterectomy

! 10!
41
ETHICS & LEGAL

Valid Consent For Cholecystectomy


You are the surgical resident in surgical outpatient. A patient came for an elective
cholecystectomy who has a recent episode of cholecystitis.

Task: obtain a valid consent for the operation, answer examiner’s questions.

What, why, how


Explain in broad term about the surgery
Why the patient is having the surgery
Draw a picture and talk about the risk and complications
Complete with patient signing the consent form (ask pt to sign)

Causes
4F – fatty, fertile, female, forty
- Hereditary
- High cholesterol
- Heavy drinking
- Obesity

Procedure
Laparoscopic cholecystectomy + intraoperative cholangiogram  keyhole surgery (4 holes)

Risk
Bleeding, infection,anesthesia
Bile duct injury 1 in 300, leaking of bile may convert to open surgery  subcostal incision

Duration of stay is overnight if keyhole surgery depending upon patient’s recovery


Send gallbladder to pathology and result back in 2 weeks
Dressing remove in 2 days
Back to normal activity in 2-3 days
No heavy lifting for 4 weeks
No driving for 2 weeks
Everything will be fine in 2 weeks

(If patient is 85 years old, dementia, came from nursing home and has fracture of neck of
femur, need surgery  ask any power of attorney or consent from next of kin)

! 11!
42
ETHICS & LEGAL

Palliative Care – Leukemia Relapse


A 55 yo lady has been diagnosed with leukemia 2 years ago. She was under remission for the last 2
years and she’s doing regular follow-up. She’s here in your GP clinic because of her blood test result
which she has done in her last visit. Blood picture showed 10% blast cells. You spoke with the
haematologist who said that she had recurrence and it will not respond to treatment. She has no other
medical treatment, her husband was in the waiting room.

Task: tell patient about the result, counsel and answer patient’s questions.

Hx:
How are you? I understand you’re here for the test result. Are you all right to hear your test result or
do you want me to call your husband.
I don’t have good news for you. Your test result showed that your leukemia has come back.

Can I have chemotherapy or radiotherapy?


I have discussed with your haematologist, unfortunately there is no treatment option

How do you feel now? I know that it would be very hard for you, it’s upsetting, but Salina we have a
lot to do. You’re not alone. You have my support all the way, also support from the palliative care
team

What is the palliative care team?


Let me explain to you the aim and what they will administer to you
Aim:
- Provide you with the best quality of life during your illness
- The team anticipates the complications of your illness and prevent them or treat them
- The team is consisted of me as your GP, nurse, specialist, oncologist, religious leader,
physiotherapist, occupational therapist, dietician, psychologist, counsellor, social worker, pain
specialist, volunteer worker, committed support group
What you need to do:
- Need to lead a healthy lifestyle and diet
- Do regular exercise
- Always think positive

What Palliative Team administer:


- Patient can stay at home
o Patient can feel free, happy
o Family member present
- Patient can stay at an aged care facility
o Supervised by GP and palliative care nurse
- Tertiary hospital or palliative care unit
- Hospice
o Environment like at home
o Supervised by a special nurse

You have all the specialists support


What you need, we’re ready to help you
We can arrange family meeting for mental support, if children overseas they can come

Complication of leukemia
- Bleeding
- Infection
- Hyperviscosity

! 12!
43
EXAMINATION CASES

Chronic Liver Disease With Haematemesis (AMC 70, p 552)


You’re an intern in the ED, a pt presented with haematemesis 500 ml. Previous endoscopy 6
months ago showed dilated vein in the lower oesophagus (oesophageal varices, portal
hypertension & chronic liver disease). The pt has Hx of alcohol abused.

Task: Relevant focus PE. Give commentary as you process.

Is my patient haemodynamically stable or not: SaO2, BP lying & standing, pulse, capillary
refill < 2 seconds
Start with DR ABC

Bleeding – put 2 large IV cannula (14 or 16 gauge)


Take blood for blood type, cross and match

PHYSICAL EXAMINATION
GA: chronic liver disease, liver failure, and signs of portal hypertension
- Chronic liver disease
o Leuconychia, clubbing
o Palmar erythema
o Hepatic flap
o Jaundice
o Enlargement of parotid gland
o Gynaecomastia
o Spider naevi
o Ascites
o Splenomegaly
o Prominent veins
o Bruising
o Testis atrophy
- Liver failure
o Hepatic flap
o Encephalopathy – confused, coma
- Signs of portal hypertension
o Splenomegaly
o Hypersplenism (increase function of the spleen)
o Prominent veins
o Haemorrhoids
- PR : malena

! 1!
44
EXAMINATION CASES

Stroke
A 60 year-old male came to ED with weakness of left arm & leg.

Task: take history, PE, Investigation, DD and Management.

Hx:
When ---LOTS RADIO (<24 hrs  TIA; if >24 hrs  stroke)
How did it start, suddenly or gradually?
Did you lose your consciousness?
Is it the first time?! Last week vision blurring, headache, blackout & weakness which
resolved by itself.
Is it getting worse or improving  improving ---TIA
Any weakness?
Were you alone or is there anybody with you?
Headaches, dizziness, visual disturbances
Speech difficulty swallowing, tongue bite and neck stiffness
Upper and lower limb – tingling, numbness, pains & needles
Chest – any palpitation, chest pain, SOB
GIT – nausea, vomiting, did you wet yourself or lose control of your bowels?
PHx (risk factors of TIA & Stroke) DM, HPT, arrhythmia, valve problems, heart attack,
high cholesterol
Family history
SADMA
Exercise
Any stress
Where do you live, with whom do you live (patient needs support – social situation)

O/E:
GA: facial asymmetry, special posture, BMI
VS: BP, HR (regular or irregular), T, RR
Assess gait and speech  receptive (name objects) & expressive (repeat a sentence)
Assess GCS
- Eye opening---spontaneous
- Verbal ---normal
- Motor
- Orientation ---Do you know where you are, what is the date (time, place, person)
Neurological exam
- Cranial nerves
- Upper & Lower limbs
CVS examination  carotid bruit, fundoscope

DDx:
- Stroke – Ischaemic / Haemorrhagic
- TIA
- Migraine (atypical migraine)
- Epilepsy

! 2!
EXAMINATION CASES

Ix:
FBE
BSL
Lipids
LFT (for baseline)
Coagulation profile
U&E
ECG & Echo
Doppler for the neck
CT Brain

Explanation:
From history & examination, you may have a stroke, we need to do some tests to make sure
the diagnosis or to find out the cause.
We look at the heart, liver and check the vessels in your neck.
(Involve the patient with each step of the management)
GP  refer the patient
Hospital  admit the patient, call the neurology registrar

Mx:
If ischaemic, we need to start you on aspirin to thinning your blood.
Might consider refer to physio, dietician
If problem with speech, refer to speech pathologist
Occupational therapist

We will work together as a team to help you return to your normal life.
No driving for 6 months.
Refer to social worker if living alone.
Lifestyle modification
Occupation therapist will assess after 6 months whether the patient can get back the license or
not. Will make the necessary changes at home and will try to train the patient to return to
their normal life eg use the other hand for cooking, driving, etc.

! 3!
45
EXAMINATION CASES

Stroke
An old man has acute onset of weakness, numbness, dysphasia for 15 minutes. His wife
brought him to the hospital, symptoms still present with him. BP 200/100, P 80. You’re an
intern. The wife asked you about the condition. Pt has pacemaker for previous heart block.

Task: Explain patient’s condition to the wife and explain management plan.

Ask for patient’s consent to reveal information.

Your husband has a condition called stroke or brain attack. When one part of the brain
stopped working properly, because of limited or decreased blood supply. It may be clots
blocking the artery or haemorrhage/bleeding

It’s common after 65 years of age


Symptoms are confusion, difficulty walking and raising arms, numbness, weakness, speech
problem, visual problem

There are some risk factors:


- Increase blood pressure
- Hypercholesterolemia
- Smoking
- DM
- Heart disease

Ix:
- CT scan, to differentiate what kind of stroke
- Blood test
- ECG, echo
- Carotid US

As soon as CT scan is back, we start Aspirin


Most common is ischaemic (85%), haemorrhagic (15%)

We’ll start by lowering his BP only if BP ≥ 220/120 mmHg, SBP ≥ 220 mmHg, or DBP ≥
120 mmHg because lowering the BP will deteriorate his neurological condition

Long-term Management
- Diet: healthy diet, no junk food
- Keep weight within normal range
- Stop smoking, reduce alcohol intake

Medication:
- To keep his BP in normal range
- Needs to review his medication to prevent clot

After previous attack, if any of the following occurs, react FAST


RED FLAG: FAST (Action plan)
F – Face assymetry ask him to smile, any changes?
A – Arm movement  difficult to raise arm, any weakness

! 4!
EXAMINATION CASES

S – Speech  speech problem


T – Time ASAP call 000
“My husband is having a stroke”

After discharge  rehabilitation program

You’re not alone.


We’ll work as a multidisciplinary team to help you:
1. Physiotherapist
2. Occupational therapist
3. Social worker
4. GP follow-ups

Reading materials

! 5!
46
EXAMINATION CASES

TIA
A 40 year-old female presented to your GP clinic with right upper arm weakness & difficulty
in speaking which resolved in a few hours.

Task: do physical examination and explain your diagnosis and management to the patient.

Equipment required:
Pocket torch
Funduscopy
Red top pin
Tuning fork for hearing 256 Hz (for vibration 128 Hz)
Snellen chart
GCS and Mini Mental charts

Today I’d like to focus my examination into 2 systems which are:


- Neurological exam including cranial nerve exam to find the extend of the injury
- CVS including carotid bruit to find the cause (85% stroke is ischemic, that means there is
a clot from somewhere)

CVS exam

Upper limbs
Look - Bulk of muscle, fasciculation
Neck stiffness
Power
Tone: wrist, elbow
Reflexes
- Biceps
- Triceps
- Brachioradialis
- Digital
Coordination for cerebral nerve – toe to heel test, Romberg test (patient standing close eyes
hand forward  hold the patient in case fall)
- Finger nose
- Alternating hand tests
- Tapping (ask patient to tap your hand)
Sensation: (spino thalamic pathway – posterior column)
- light touch with cotton wool,
- neurological pain with pin,
- vibration with 128 tuning fork,
- position

Tone: hands & wrists


Power (chicken wings)
Reflex
Vibration 128 Hz (chest & thumb)
Sensation : light touch with cotton wool, pin sensation (pin prick)
Proprioception (fingers up & down)
Coordination (nose finger and dysdiadochokinesis, alternating hand test)

! 6!
EXAMINATION CASES

Lower limbs
Power
Tone: Knee, Ankle
Reflexes
- Knee
- Ankle
- Toes
- Babinski/plantar (outside to inside – normal flexion, if toes extend  upper motor injury)
Vibration
Coordination for cerebral nerve – toe to heel test, Romberg test (patient standing close eyes
hand forward  hold the patient in case fall)
- Toe finger
- Heel shin
- Tapping (ask patient to tap your hand)
Coordination: heel shin, toe finger test (while patient lying down)
Cerebellum
- Romberg test
- Tandem (heel-toes gait)
Sensation: (spino thalamic pathway – posterior column)
- light touch with cotton,
- neurological pain with pin,
- vibration with 128 tuning fork,
- position

Lower limbs
CVS to find the cause

Explanation:
It’s a Transient Ischaemic Attack, transient means for a short time, ischaemic  the blood
supply is being cut going to your brain or Mini Stroke.
Have you ever heard about it?
By itself it’s not a problem, but it’s a warning sign. If it’s not treated it can cause stroke.
I’d suggest you to go to ED, the neurology will assess you & they will do all of the
investigations  Blood investigations & CT Brain.
If there is a bruit  Doppler DO ECG -> AF
Management will depend on the investigation result.
The specialist will tell you in more detail
If results normal  ischaemic stroke  neurologist will give aspirin

Risk Assessment - ABCD


- Age
- BP
- Clinical presentation
- Doppler
If <4 discharge patient and follow-up

! 7!
47
EXAMINATION CASES

Primary survey + GCS


A 25 year-old man was injured in a motor vehicle accident presented to ED. Pupil react
sluggish, eyes response to painful stimuli, doesn’t follow command but he moans
periodically. His right arm is deformed and does not respond to painful stimulus. His left
hand moves purposely towards painful stimulus. Both legs are stiffly extended. Task perform
focus examination. Tell examiner what you are doing and why after each component. Explain
the findings at the end of the exam. Investigation you would do.

Go in, personal protective equipment


Gloves, goggles, gown

D – Danger for yourself, patient


R – Response  check patient’s response and call for help
A – Airway + cervical spine
B – Breathing  ventilation
C – Circulation  haemorrhage
D – Disability  GCS
E – Exposure

Acute bleeding

EYE
4 – open spontaneously
3 – open on verbal command
2 – open to pain
1 – none

VERBAL
5 – orientated
4 – confused
3 – inappropriate words
2 – incomprehensible sounds
1 – none

MOTOR
6 – obey command
5 – localizes pain
4 – withdrawal
3 – flexion (decorticate posture)
2 – extension (decerebrate posture)
1 – none

Explain GCS finding to examiner

INVESTIGATION
- FBE, U & E, BSL
- Blood grouping & cross match and hold for 2-4 unit
- Trauma series
- C-spine X-ray or preferable CT, MRI

! 8!
EXAMINATION CASES

- Chest X-ray, Abdominal X-ray, Pelvis X-ray

GCS 8 or less than 8  intubation

Not to order too much investigations in primary survey


You can do all others that you want, after you stabilize him in secondary survey

GCS: INTUBATE IF GCS < 8

I WOULD LIKE TO ASSESS THE PATIENT WITH ABCDE APPROACH


PUT THE PT IN CERVICAL COLLAR FIRST & PELVIC BELT
A PT MOANING -> PATENT AIRWAY
B LOOK FOR CHEST MOV, SATS, AUSCULTATE (RULE OUT TENSION PNEUMO THX)
C P, BP (ANYTHING SUGGESTIVE OF ONGOING/ ACUTE HAEMORRHAGE)
- PUT TWO LARGE BORE CANNULAS AND TAKE BLOODS FOR FBC, XMATCH 4 UNITS, COAGS, UE
D LOOK FOR NEURO DEFICITES
E* EXAMINE FOR INJURIES, NEUROVASCULAR COMPROMISE FROM ANY #/ D
**NOT IN PRIMARY SURVEY…

I WILL GET HELP FROM A SENIOR REG OR CONSULTANT


NEED RADIOLOGICAL IX: CT/ MRI FOR C SPINE
XRAY - CXR, PELVIC XRAY, XRAYS OF LIMB ACC TO INJURY
DO FAST SCAN TO LOOK FOR ANY INTRAABDOMINAL INJURIES

! 9!
48
EXAMINATION CASES

Coma +GCS (AMC 44)


A young patient was found unconscious by a flat mate, he doesn’t know anything about his
friend. In ED airway patent, patient breathing without difficulties, circulation is stable. Temp
37.5, BP 140/70.

Tasks: are to perform examination to determine the level of unconsciousness and try to
identify the cause, tell examiner what you’re doing and why. Towards the end of
examination, the examiner will ask you a few questions.
(Secondary survey)

GCS
Find out the cause

Critical error:
GCS
Neck stiffness

DR ABC
GCS
Response to pain – press in the nailbed, or press on the glabella between the eyes, the patient
will bring his hand on the glabella (localized pain), or rub the chest

Inappropriate words

Examine to find the cause from head to toes


Head  Any sign of injury, trauma, bleeding, bruise
Examine the eyes
- The movement of the globe of the eyes
- Shine torch to assess the reactivity of pupil (miosis or mydriasis) – look for pin point
pupil (very small) due to opiate
- PERLA
- Corneal reflex
- Fundal examination
Neck  neck stiffness
Breathing – movement of the chest
- Type of breathing – hypoventilation (depressant drugs), hyperventilation
- Pulse rate, rhythm, character
Assess the cubicle fossa for IV drugs marks
Abdomen
Thigh – for insulin injection

GCS
- Eye 4
- Verbal 5
- Motoric 6

Lowest 3, Highest 15

! 10!
EXAMINATION CASES

EYE OPENING
Press the glabella, if open eyes  2
If patient draw his hands to the glabella  5
Patient says any bad words  3

VERBAL
Orientation – ask patient where are you? – if he reply “in the hospital” – 5
Confused 4
Swearing 3
Incomprehensive voice 2
No response 1

MOTOR
Move hands 6
Move on the pain 5
Apply pain and patient withdraw his hands 4
Flexion 2 3
Extension 2

GCS for this patient 8 – 12


If fall from horse <8  intubation

Tell examiner  I’m not sure of the nature of the injury, I’d like to put a cervical collar

DDx: after rule out neck injury


- Head injury
- Drugs
- Infection meningitis
- CVA like stroke
- Subarachnoid haemorrhage
- Metabolic disorder (hypo or hyperglycaemia)
- Psychiatric problems

Investigation
Head injury & SAH – CT or MRI (if unavailable or negative)  Lumbal puncture
Blood
Urine analysis
Check for alcohol level
BSL
ABG

! 11!
49
EXAMINATION CASES

Unconscious pt: Fall From a Horse


A 10-year-old girl brought in by ambulance in a country hospital after falling from a horse 30
minutes ago, the nearest city hospital (have all the facilities) is 200 km away. You’re an
HMO but you’re at home.

Task: Talk to the nurse about what to do by phone, how to manage the pt.

I’m Dr…., who’s calling please, can I have your name?


DR ABCDE

D  Danger – patient and doctor;


Patient: where is the patient? In hospital bed, in the road, near the water, near a fire…
Doctor  doctor security

R  Response – ask the patient….hello Mr/Mrs…, how are you?


If no response, call for help. Tell nurse…please call for help (Code Blue).

CERVICAL COLLAR for cervical injury


Has the patient been put a cervical collar on the neck? If not, please do it.

A  Airway – check for oral cavity, ill-fitting denture or bones  take it out; any vomitus,
fluid  suction. (Guedel)

B  Breathing – look for a chest movement (equal or only on 1 side), listen to the breathing
sounds and feel the breathing of the patient against your cheek. (tenderness if pt is conscious)
Auscultate lungs for any fluids. CXR
Oxygen saturation for unconscious, if < 92 give 4-6 liters of O2  INTUBATION.

C  Circulation – pulse, BP; put IV line  take some blood for test. Give fluids if BP is low
and there is no fluid in the chest. Check arterial blood gas, FBE, blood grouping (maybe
patient will need blood). ECG

D (Deformity/defect)  GCS & PEARL

GCS if 8 or less  Intubation. Tell nurse to call anaesthetic surgeon to do the intubation.
PEARL  Pupil is equal and reactive to light  good sign – if not, bad sign
CT

E  Take off clothes and explore from head to toe for any sign of injury  head injury,
fracture, bleeding,
Airway tube – 3 sizes: small, medium, large  measure the size from the patient’s ear lobe to
the canine tooth  adjust according to the 3 sizes.

I will call the AIR AMBULANCE to transfer the patient because the patient will need a CT
Scan. Please do a close monitoring on the patient’s vital sign, put catheter, and send blood for
investigation.

Ix for primary survey of MVA


X-ray  CXR, Spinal X-ray, Pelvic X-ray. ECG, CT

! 12!
50
EXAMINATION CASES

Thyroid Examination
A 23 year-old Jenny presenting to you at your GP with a lump in her neck. Please examine
the patient, tell the examiner the examination finding and manage the patient.

AF  check pulse

Jenny, I’m Dr…..


I’ve been instructed to examine your neck today, is that OK with you
I’ll wash my hands & make it as warm as possible
During the exam, if you feel any pain, just let me know so that I can stop

Jenny, today I’m going to examine your thyroid gland because thyroid gland has implication
on different parts of our body like the eye, heart, lower & upper limb

Start with GA
Patient is appropriately dressed for the temperature of today
Well-built or not
No features of…
There’s a lump in her neck about 5-7 cm

Can I examine your hands….there’s no sweat, no clubbing (thyroid archiopechy)


Stretch your hand & fanning the fingers and see tremor
If in doubt, put a piece of paper to see if shaking

Can I check your pulse ---80 reg synchron


BP is normal

Looking at the neck, there’s a swelling of 5 by 7 at the….


No skin changes, rash, discharge, feasible palpation, redness
Warmth, mobility, pulsation, thrill

I’d like to give some water to my patient, please take a sip and hold it…please swallow
The swelling moves with the swallow
Can you please show your tongue?
No protrusion of the tongue ---thyroglossal cyst?

I’d like to touch the swelling, is it OK with you


From the back, touch the normal side then the site of the lump  consistency, isthmus

Left side clearly swollen, smooth


Could you drink again and swallow
Move the swelling up and put the finger if I can go underneath on the lower border
If the lower border can be felt, no hand raising test (Pemberton test)
If the border cannot be felt  do pemberton test

Auscultate right & left lobe looking for bruit

Check submental, submandibular, preauricular, posterior auricular, occipital  no lymph


node enlargement

! 13!
EXAMINATION CASES

Look at patient in an oblique way from the back, tilt the patient’s head  to see for
exophthalmos

Look from the front of the patient  look at the eye for lid lag (slow movement), lid
retraction, ophthalmoplegia
Don’t move your head, please follow my finger from up to down ---no lid retraction
Make an H sign ---no ophthalmoplegia

Do chicken wings, and press on the biceps  no proximal myopathy


Check peritibial myxedema
Ankle jerk

Fold hands & stand up check for proximal myopathy in the hip

EXPLANATION
Jenny, there is a lump in your neck.
I’m suspecting a simple goiter, I’m not sure yet so I’ll refer you to an endocrinologist who
will do some tests such as:
- FBE
- TFT
- Ultrasound of the thyroid gland for consistency
According to the ultrasound, the endocrinologist will take more steps from there.

This is caused by a deficiency of some elements mainly iodine


- Live in areas away from the sea
- Take salt not iodised
- Rarely eat seafood

! 14!
51
EXAMINATION CASES

Snake Bite (AMC)


At a country side hospital, a lady presented with snake bite bellow the right ankle.

Task: take history, first aid, investigation and management.

FIRST AID
I know you had a snake bite
Please lie down and try to avoid movement
I will take a swab
I will put a bandage until the inguinal area
Wooden splint may be used to avoid unnecessary movements
Here is a jug of water, please drink it.

HISTORY
Did you see the snake
What were you doing when it happened
Any nausea, vomiting, blurred vision, headache, dizziness, sleepy eyes
Difficulty in breathing and swallowing
Any blood in urine
Do you have any kidney problems or any bleeding disorder or blood disease
Are you on any medication like aspirin, warfarin, corticosteroid
Any allergy
When was your last tetanus vaccine

At the site of the bite: any swelling, enlarged lymph nodes or bruises
Neurological: ptosis, fixed dilated pupil, drooling, dysphasia, reflexes?

INVESTIGATION
1. Swab from wound site
2. FBE
3. Urine test
4. CK

Snake venom detection kit (If no bite mark, we may use this to test urine).

If urine and swab test negative - repeat in 3 hrs. If positive, start anti venom.

THERAPY
1. Subcutaneous adrenalin 0.25 mg before giving anti venom.
2. IV anti venom is diluted in Hartmann solution (1:10) - 30 minutes infusion
3. Keep adrenalin (to prevent allergic reaction), antihistamine, corticosteroid and O2 ready at
all time
4. Oral prednisolone for 5 days - to prevent serum sickness

Give anti venom:


If still (+) in urine  repeat anti venom infusion

If bleeding or bruises or blood in urine  give FFP

! 15!
EXAMINATION CASES

If there is no anti venom, we may give Neostigmin to prevent paralysis

If no symptoms - no anti venom


Any neurological symptoms or bleeding symptoms - give anti venom

Don’t forget to contact tertiary hospital if any symptoms - contact for advice
Add tetanus vaccine if tetanus is taken > 5 years
No antibiotic is needed.

! 16!
52
EXAMINATION CASES

Snake Bite
Country hospital ED. 22 yo Mr Young who has been brought in by a friend by car. They were
bush walking when Mr Young was suddenly bitten by a snake. It struck him on R lower leg
and quickly disappeared into the bush. He thought it looked brown and was about 2 meters
long. Mr Young is complaining about pain in his lower leg, he looks sweaty and quite
frightened. VS: BP 125/75, P88/min + reg., RR 22, T 37, SaO2 99% on RA).

Task:
1. administer first aid using the provided items
2. explain to the patient why and what you are doing
3. take a brief, further history and perform a focused examination of the patient
4. answer the examiner’s questions

HOPC: as above, no previous exposure to snake bites. No headache, N or V, no bruises, no


muscle weakness

FIRST AID:
There are a variety of bandages, tourniquet, cotton wool balls, scalpel blades, a venom
detection kit, wound dressing and a walking stick (resembling a back slab splint) you have to
chose what you need for the first aid measure:
REMEMBER to take a swab from the bite site first!!!

PRESSURE IMMOBILISATION TECHNIQUE: Snake venom spreads via the lymphatics


which can be prevented by applying a firm, broad crepe bandage. This is applied to the bite
site first, but then has to extend over the entire limb, distally and proximally!
Absolute rest is most important and the limb should be immobilized with a splint
The first aid measure can be removed when resuscitation equipment is in place and
antivenom is given (if necessary).

EXAMINATION:
GA: The patient looks a bit sweaty and anxious.
VS: BP 125/75, P88/min + reg., RR 22, T 37, SaO2 99% on RA
There are clear fangmarks (bite) just above the right ankle with small drops of fluid (venom)
on the skin.
• Ptosis, diplopia and blurred vision
• Dysarthria
• Peripheral muscle weakness
• Bruising / haematoma
• Haemoglobin- or myoglobin-uria
• Respiratory failure
• Circulatory failure

Venemous Australian / world snakes:


1. small skaled inland taipan
2. brown (VIC)
3. Taipan
4. Tiger (VIC)
11. Copperhead (VIC)

! 17!
EXAMINATION CASES

12. Indian Cobra


21. Red bellied black snake (VIC)

Venom effects:
Neurotoxins (tiger, death adder, taipan)
Anti / Pro –coagulants, defibrination (brown, tiger, taipan)
• Blood clotting, fibrin degradation products
• PT, APTT
• D-dimer
• thrombocytopenia
Myotoxins (tiger, Mulga)
• CK
• myoglobinuria
Haemolytics
• hamoglobinuria
Nephtrotoxins (brown)
Cardiotoxins (brown)
• ECG
Local pain – oedema – lymphadenopathy

Mx:
1. D R A B C
2. First aid: pressure immobilization & Splinting
3. Ix: VDK, urine-analysis for myoglobin / haemoglobin, FBE, group and hold, clotting,
UEC, CK, ABG, ECG
4. Venom detection kit (VDK) + Antivenom (if symptomatic)
Antivenom is only indicated in real or suspected snake bite with clinical or laboratory
evidence of envenemation!
• subcutaneous adrenaline with 0.25 mg in adults and 0.01/kg in children 5 min before
antivenom.
• Oxygen, antihistamine, corticosteroids and resuscitation equipment should be available!
• Prednisolone 50 mg orally for 5 days to prevent serum sickness

! 18!
53
EXAMINATION CASES

Tremor
Intentional tremor  cerebellar injury  ask patient to do something (finger-nose)
Postural tremor  Parkinson disease
Resting tremor  Parkinson disease
Flapping tremor  hypoxia (COPD), hypercarbia, or liver disease
Benign essential tremor  tremor on one of the arm

To see tremor  ask patient to walk

Hx:
How long ago did you start having it?
Any other body parts trembling? --- sometimes the head
Is the tremor affecting your writing ability or handling cups & objects? (interfering with
activities)
Is it worse when in emotional stress, nervous?
Is it embarrassing?
What makes it better or worse?
Is the speech affected?
Drink alcohol  liver problem
Chronic lung condition – COPD
Any medication? (especially the ones toxic to the liver)
Any thyroid problem?
Problem with high blood pressure?
Anyone in the family with tremor?

O/E:
GA: jaundice, cyanosis, any discoloration of the skin
VS
Respiratory: signs of COPD
CVS: signs of heart failure
Abdominal: liver, portal hypertension

Posture
Facial expression  lack in Parkinson
Drooling saliva
Reduced blinking
Gait  difficulty in starting walking, shuffling gait, no arm swing, difficulty to stop
Hands is examined with pillow underneath  see the pill rolling tremor (rest tremor)
Finger tapping (bradykinesia – can’t coordinate the movements properly)
Tell patient to stretch the hands  flapping tremor

Test the tone


- Cog-wheel rigidity  move the patient’s hands
- Lead pipe rigidity  bending the elbow
- Face
o titubation (tremor of the head)
o lack of facial expression
o reduced blinking
o drooling saliva

! 19!
EXAMINATION CASES

o Glabellar tap – patient will blink

Ask patient to talk (name, address, phone number)


- monotonous tone (without tone)
- repeat words or ends of the words (palilalia)

Eye movement
- look up, down and side (down looking is loss first, then up, then horizontally)
- write name, address, phone number  micrographia (very small & unintelligible) (can be
affected in benign essential tremor, but writing can be understood)

Cerebellar function
- finger-nose test  intentional tremor  tremor will get better but tremor at rest (+)
- Dysdiadochokinesia  coordination of the movement  will not be able to do it
- Toe to heel walking

Romberg test
Stand behind the patient when patient close their eyes

Therapy:
- Benzodiazepine
- Propranolol (preventer)
!

! 20!
54
EXAMINATION CASES

Hernia Examination (AMC 56)


A 40 yo man presented with groin lump.

Task: perform inguinal scrotal assessment.

Is there a hernia or not


Is it reducible
Is it femoral or inguinal
If inguinal, direct or indirect

Intestine come from internal ring and bulging, follows spermatic cord and goes to scrotum 
Indirect inguinal hernia
If coming from superficial external ring and bulging out directly, not through a canal 
Direct inguinal hernia.

If above the inguinal ligament: inguinal hernia


If below: femoral hernia
In obese patient or if the bulging is very big, it’s difficult to differentiate

Any lump in scrotum: Hydrocele, Varicocele or Indirect hernia

To examine if there is a hernia or not


Stand up to give pressure  see if there is hernia
Patient lying down, tell patient to cough and put your hand in the inguinal area to see if there
is hernia
If in the scrotum  indirect
Put the finger through the scrotum, follow the spermatic cough and ask patient to cough, if
the mass hit the tip of your finger  indirect

Tell patient, is it reducible? If reducible, tell patient to reduce it first then put your hand to see
if it’s reducible

LOOK
Look at the site, above or below the inguinal
Scrotum swelling or not
Size big or not
Assess the change of color – incarcerated (red) or strangulated (bluish)
- Reducible : hernia is coming and going
- Irreducible or Incarcerated: hernia is coming down but not reducible, blood supply not
affected
- Strangulated: the part of hernia get necrotic, blood supply is cut, operation as soon as
possible, underlying structure is blue
Ask patient to cough, if it’s bulging

FEEL
The surface of the structure
Intestine or not
Consistency
Temperature

! 21!
EXAMINATION CASES

Tenderness
Assess cough impulse
Movement to reduce the lump (if patient can do it first then you do it)

SPECIAL TEST
Fluctuation
Transillumination
Compressibility (might be a lymph node, lymphoma or sebaceous cyst) – soft or hard, lymph
node cannot be compressed
Move to see if it’s fix or not, does it move with the skin
- Sebaceous cyst  moves with the skin
- Lypoma  moves freely below the skin because it derives from subcutaneous tissue
- Lymph node  if fixed
- Mass in the muscle  moves with muscle
- Osteophytic changes of the bone  fixed, might be from the bone

Intermittent pain, nausea, vomiting, cough


- Meningitis
- Ear infection
- UTI
- Intususception
- Obstruction (pyloric – green vomit), malrotation of the bowels
- Fever, any discharge from ear, crying while passing urine, color of vomit, is it the first
time, the stool color change or not
- Tummy any signs of pyloric stenosis, assess the orifices, hernia orifices in the umbilicus
and inguinal
- Genitalia exam: a part of abdominal examination – lump in the scrotum – indirect hernia

! 22!
55
EXAMINATION CASES

Diabetic Foot Examination


Health care plan for diabetics

Plan to see diabetic educator – should check how they use the glucometer
HbA1C every 3-6months – measure of glycaemic control
Type II diabetes – early introduction of insulin, is beneficial
Contain the diabetes before there are changes in the eyes, kidneys and heart –
They might have depression with their diabetes
Eye check should be once or twice a year with an ophthalmologist

45 year old woman wearing a t shirt and a hospital gown, sitting comfortably at 45 degree
angle on the couch
Examine her lower limbs

Inspection:
Hairless atrophied skin
Ulcers – (check for pressure sores at the bunion area and lateral side)
Feet Clawing – contraction of the toes due to muscle atrophy
Superficial skin infection – boils
Pigmented scars (from previous ulcers that heal)
Nails: fungal infection
Thighs Wasting of quadriceps
Injection sites
Charcot Joint – a very swollen knee which lost propioception – painless, not red, very
irregular, non tender

Feel: temperature and pulses

Move: power and tone

Neurologic Examination (Test sensation at levels not dermatome)


Microfilament (plastic needle)  continuously touching from the base of the toes on the
dorsal side up to the leg (gloves & stocking sensation loss) “please let me know when you
feel the sharp object”
Propioception – I’m going to move your great toe (hold from the sides of the great toe, this is
up, this is down. Now, close your eyes and tell me where I am moving the toe. Is it up or
down?
Vibration – tuning fork (128 hertz or anything above 50 hertz)
Put on chest – could you feel the vibration?
Close your eyes
Put the tuning fork on any bony prominence (ascending order: Toes, medial and lateral
malleolus, knee, ASIS)

Reflexes – hyporeflexia – ankle jerk(S1-S2) and knee jerk (L3-L4)


Plantar response – no need – extrapyramidal response

! 23!
56
ENT & OPHTHALMOLOGY

Trachoma (AMC 57)


You’re a doctor working in a GP clinic in a remote setting in the Northern Territory. A nurse
made a time to see you to discuss eye problems she has noticed in the local Aboriginal
community. The nurse has taken digital photographs of eye problems that were noticed in a
number of affected individuals (photos taken of four separate individuals).

Task: explain what disease is illustrated & its epidemiology, Mx, answer the nurse’s Qs.

Follicles (multiple stacks)  scarring  distort the eyelid  entropion  trichiasis 


rubbing on cornea  ulcer, infection of cornea  scar of cornea  blindness

Follicle  hand contacnt  flies  follicle (sign of active disease)

If >5 Follicles – a sign of active disease caused by Chlamydia trachomatis– high risk of
transmission

F – Follicles (intracellular bacteria, multiplied inside the cell)  active proliferation


I – Inflammation
S – Scar – contraction, eyelid turned inside (intropion)
T– Trichiasis – eye lashes turned inside – rub on the cornea – cause irritation, ulcer &
infection
O – Opacity – cornea will not be transparent and the patient will have a terminal blindness

Mode of transmission:
- Flies
- Hand contact
- Fomites (towels, cloth to clean the face)

Epidemiology:
Common in the hot climate, dusty and overcrowded areas. Caused by Chlamydia trachomatis.
It’s endemic in the tropical areas especially aboriginal communities in rural parts of Australia

Treatment:
S- Correctional eyelid surgery for entropion and trichiasis
A- Antibiotics Azithromycin 1 dose + Tetracyclin 1-3% eye ointment tds for 6 weeks, treat
the whole family
F- Face hygiene, hand washing
E- Environment: good ventilation of the houses, cover the garbage, try to kill the flies,
improvement of water supplies and sleeping areas

The condition is trachoma which is a Chlamydial conjunctivitis, very common in the


Aborigenese. It’s transmitted by human contact and flies, so whenever hygiene is not good,
it’s more common. It spreads very quickly and is very contagious not only in children but
also in adults. It’s the most common infectious causes of blindness in the world.

Repeated attacks can cause scarring of the eyelids which turned the eyelashes inwards. These
lashes rub on the cornea causing corneal ulceration and opacification and finally, blindness.
Good treatment is available and is SAFE.
Main issue the whole community should be treated. No sharing of towels or clothes.

! 1!
57
ENT & OPHTHALMOLOGY

Myopia (AMC 45)


A 17 yo boy c/o worsening of vision, difficulty of driving, cannot see road clearly. Both parents wear
glasses.

Task: Examination, explain condition, manage the case.

INSPECTION
Ptosis
Corneal abnormalities
Ulceration

Color of sclera
- Jaundice
- Pallor
- Red or blue

Exophthalmus
Examine 2nd nerve
- Visual acuity
- Visual field
- Funduscopy

How to check visual acuity


- Snellen chart
- If not able, finger counting
- Hand movement
- Light perception

Fundoscopy
- Cornea
- Lens
- Retina

I’m looking for any optic atrophy, papillory oedema, retinal detachment, vein or artery thrombosis,
cotton wool or flame haemorrhage
Check pupil reaction, light and accommodation
Patient has problem with acuity but no problem with visual fields and no changes on fundoscopy.
Only short sighted eyes. Common condition. Onset in the teens age. Positive family history.

DrawImage is focused before the retina (back layer) that’s why you can’t see clearly.

Retina becomes thin and stretched more, becomes fragile, holes can develop and detachment.
I’ll refer you to an optometrist, you need to wear glasses or you can use lens.
You need to wear glasses. The glasses will focus the image on your retina.
Eye check every year.

4 type of refractive errors


Myopia – short sightedness
Hypermetropia – long sightedness
Presbyopia – decrease of muscle power after 40 years of age
Astigmatism

! 2!
58
ENT & OPHTHALMOLOGY

Glaucoma
A 45 yo female c/o difficulty in vision for the last couple of months. A visual acuity exam
showed her vision of 6/18 in both eyes and did not improve with pinhole test.

Task: take history, examination, management.

DDx:
Cataract
ARMD
DM
Temporal arteritis
Glaucoma

Differentiate with pituitary tumour  usually for tumour there’s a defect in the temporal field
(in 1 eye) of the affected site. Visual acuity will not be normal.
Glaucoma  peripheral visual field defect.
Macular degeneration cannot see the central vision (opposite with glaucoma)

Schlemm canal draining the fluid  ischaemia & necrosis of the fibers  loss of vision

Pathology blocking the angle in acute glaucoma  the angle decrease to 40 degrees (normal
60 degrees)  prevent draining of fluid  causing very severe eye pain  blindness,
redness, nausea, vomiting  pressure has to be decreased if not will cause permanent
blindness.

Building of the fluid inside the eyes causing pressure to the nerve & fibers

Hx:
Onset  affecting one or both eyes
How did you notice it?
Do you bump into people’s shoulders while you’re walking?
Sudden or gradual?
Progression getting worse or better
Type of visual loss  central or peripheral
Problem with sunlight? (to exclude cataract)
Did you see the road signs when you’re driving?
Do you wear any glasses?
Have you noticed that you have to change your glasses more frequently?
Associated symptoms  redness, pain, nausea, vomiting, headache
eye infection, trauma
General health  DM, hypertension
Medications
Family history
SAD

! 3!
ENT & OPHTHALMOLOGY

O/E:
GA
VS
Inspection
- Redness
- Discharge
- Ptosis
- Pupil size
Acuity (with glasses)
- Snellen chart  handhold chart which the patient hold
Visual field (without glasses)
Eyes movement
- H movements (10 directions)
Pupil – PEARL (pupil size light accommodation) equal reactive to light & accommodation
 normal but in acute galucoma fixed mid dilated pupil (no direct or indirection reaction
to light)
Ophthalmoscope  DO NOT dilate the pupil (I’m suspecting the patient has glaucoma and I
don’t want to aggravate the eyes)
- Assess cornea  ulceration for viral infection, scratches for foreign body, red reflexes
(no red reflex in tumour and cataract)
- Anterior chamber  blood (hyphaema), pus
- Assess retina  hypertension or diabetic changes
- Optic disc  cupping (+  increase), margin (not clear in papillary oedema)
- Tonometry  pressure increase 30 (normal 10-20)

Explanation:
Most likely you have a condition called glaucoma, which is an increase of the fluid
production, or maybe a decrease of the fluid drainage causing increased pressure in the eye.
Because the eye is a close organ, the fluid cannot go anywhere and can cause damage to the
nerves.
It’s a common condition, controllable if detected early. It develops slowly. Cause loss of
outer vision of both eyes. If untreated, it could lead to blindness.

Mx:
I’ll refer you to an ophthalmologist, he probably will start you on medications such as:
Timolol  to increase the drainage
Acetazolamide  to reduce the fluid production
Pilocarpin  to open the angle (stretch the iris and cause meiosis)
------close the TAP

If not controlled, a laser surgery can be done  open holes in the iris (iridotomy).

Improved with pinhole  it means refractory error  if not improved it means other causes.
Open angle  chronic glaucoma; close angle  acute glaucoma.

! 4!
59
ENT & OPHTHALMOLOGY

Age Related Macular Degeneration


A 70 yo c/o increase difficulty reading newspaper over the recent months.

Task: take further history, ask for physical exam findings and advise on the management.

Important points in visual disturbances: Age; Sudden or gradual; Red and painful eye

DDx:
ARMD
Cataract
Glaucoma
DM
Presbyopia

Gradually worse over 5-6 months


Blurring of vision
No pain, no flashing or floating
Difficult to recognize faces and read road signs
Central part getting worse  blurring
Mother blind at age of 80
Smoke for a long time
Visual acuity – Slightly reduced on the right side
Visual field – central scotoma on the right eye
Funduscopy – normal
Eye movements normal
See wavy lines

Hx:
Onset
Is it in one or both eyes? (usually start in 1 eye then involves both)
Can you tell me what’s the disturbance in the vision like?
I’ve seen in the notes that you’ve had difficulties reading newspapers
What about in recognizing faces (visual acuity)
Watching TV, read street signs, telephone directories
Does light improve your vision?
Any halos around objects?  Cataract
Distorted images  lines look wavy
Any floaters or flashing light in your vision
Any pain in your eyes, any headache
Noticed any recent increase of watering of the eyes
Redness or discharge
Any previous eye problems, surgeries, trauma
Do you use glasses or not, what for and since how long
General health DM, HPT
FHx of DM, glaucoma, ARMD
SADMA

! 5!
ENT & OPHTHALMOLOGY

O/E:
GA
VS
Eye examination
- Inspection: look at the eye in general
o Eyelid dropping
o Conjunctival injection or chemosis (congestion of the bloods  thick sclera)
o Cornea – visible ulceration, scar
o Visual acuity
o Visual fields
o Pupil reflex
o Fundoscopy (red reflex to rule out cataract)  normal result does not exclude macular
degeneration, sometimes only raised macular area
 Retina
 Macular area
 Optic disc
- Palpation
o Feel for orbital tenderness
- Amsler-Grid test: special chart with lines, on the central part of the vision the lines look
wavy

Explanation:
Most likely your visual changes is a condition called macular degeneration which is a
common condition in people with increasing age. It’s due to some degenerative changes in
part of the eye called macula which is an area that is responsible for our central vision.
The cause is unknown but there are some risk factors such as:
- Increasing age
- Family history
- Smoking
- Poor diet (not full of nutrients)

I’d like to refer you to a specialist because he may need to do further test which is called
fluorescein angiogram, a test to know which type of macular degeneration.
There are 2 types:
- Dry type 90%
- Wet type 10%  rapid and sudden deterioration of vision  needs laser therapy

Unfortunately this is a progressive disease, there is no definitive treatment.


But there are some measures to slow down the progression which are:
- Lifestyle changes - eating healthy diet
- Stop smoking
- Vitamin A, E and Zinc might be helpful
- Protect the eye with sunglasses

! 6!
60
ENT & OPHTHALMOLOGY

Vestibular Neuritis
A 45 yo female presented with sudden onset of N/V, and vertigo for several hours.

Task: take history, examination, investigation & management.

DDx:
Peripheral cause
- Labyrinthitis
- Simple vertigo
- Meniere’s disease
- Drugs
- Trauma
- Chronic otitis media
- Acoustic neuroma
- Cervical spondylosis
Central cause
- Brain stem disease
- Vestibulo basilar insufficiency
- Infarction in the brain stem

Characteristic of Meniere’s disease:


- Recurrent vertigo
- Progressive hearing loss
- Tinnitus
- Vomiting

Acoustic neuroma
- Vertigo
- Hearing loss
- Ataxia
- Tinnitus in the lateral
- Cranial nerve involvement

Vestibular neuritis
- Nausea, vomiting, vertigo
- No hearing loss

Cerebellar cause
- Tumor
- Infection
- Infarction
- Migraine
- Multiple sclerosis

Hx:
What do you mean by vertigo---everything is spinning around me
When did it start---a couple of days ago
Is it the first time---yes

! 7!
ENT & OPHTHALMOLOGY

What were you doing when it started? Suddenly or gradually? ---suddenly (If benign
positional vertigo  it starts for a few second after moving head)
Any posture make it better or worse like walking or running?
Have you heard any ringing sound?
Any problem with hearing?
Any problem with vision, any flashing of light, any halos around light?
Any numbness or weakness in any part of your body (multiple sclerosis – one side paralysis)
Any headache
Any problem with your walking? (gait)
Any problem to maintain balance
Any recent head injury or falls?
Any recent infection of ear, nose and throat---I had flu recently
Any problem in your neck pain or stiffness (cervical spondylosis)
Any significant past medical and surgical condition DM, HPT, heart disease or stroke
Are you on any medication
Did you take any medication for your condition?
SADMA
System review
Appetite and weight loss (to exclude malignancy)
FHx

O/E:
GA
VS: pulse, BP, temp, RR
Eye: nystagmus
Ear: otoscope normal, no discharge
Neck: tenderness to rule out cervical spondylosis
CVS
Cranial nerve
Neurological examination

Special test:
Tests for vertigo
- Can you look right (if patient has vertigo while looking to the right side – it means the
lesion is on the left side  opposite side)
- Can you look left

Hallpike maneuver
This test is to determine if a patient’s dizziness is caused by an inner ear disorder.
Patient sits on a table, a doctor lays the patient down with the patient situated so that his head
hangs over the table’s edge. While the patient lies down, the doctor simultaneously turns the
patient’s head to the left and to the right. Patients often develop dizziness and nystagmus very
quickly from this maneuver if they have an inner ear disorder.

Nystagmus is an involuntary eye movement that generally causes fast movement of the eyes
in one direction alternating with a smoother eye movement in the other direction.

Caloric test
Cold or warm water is irrigated into the external auditory canal using a syringe. The
temperature difference between the body and the injected water creates a convective current

! 8!
ENT & OPHTHALMOLOGY

in the endolymph, produce currents in opposite directions and therefore a horizontal


nystagmus in opposite directions.
Hot water – fast phase of nystagmus to the same side as the hot water filled ear
Cold water – fast phase of nystagmus to the opposite side from the cold water filled ear
COWS  Cold Opposite, Warm Same

 HOT WATER  COLD WATER


 Nystagmus Nystagmus 

Ix:
- FBE
- Chest X-ray
- Cervical X-ray
- CT scan to exclude tumor
- Audiometry

Explanation:
You’re having a condition called vestibular neuritis, it’s a common condition in this age
group. Female are most commonly affected. But the good thing is that it’s a transient
condition. It will disappear by itself. Sometimes it takes a few days, sometimes a few weeks.
What cause it?  in your case, virus

Mx:
- Bed rest
- If you have problem on the affected side, try to look to the opposite side where you don’t
have the problem
- Medication: Prochlorperazine (Stemetil)
- Dimenhydrinate (antihistamine)
- Oral prednisolone
- Diazepam
- If patient is concerned or if there is any problem with hearing or cranial nerve, refer
patient
- I will do regular follow-up
- We’ll check again and review your investigation result, if not improving I’ll refer you to
the specialist.

! 9!
61
ENT & OPHTHALMOLOGY

Cholesteatoma (Conductive Deafness)


A 26 year-old John came to your GP clinic complained of whitish ear discharge. Picture
showed pearly white growth on tympanic membrane.

Task: take history, examination, management.

Hx:
How long have you been having this ear discharge?
Which ear? Right, left or both
What is the color? Whitish? Purulent yellowish? Any bloody discharge?
Is it smelly? yes
Is it affecting your hearing?
Any headache? Any dizziness? Fever? Any weakness any where in the body?
Any similar condition before?
Any recurrent ear infection? If yes, what was the treatment?
Any family history of hearing problem? Or similar problem?
What is your occupation?
Have you been exposed to very loud noise on a daily basis?
Do you use medication for a long time?
Any head trauma or ear injury?

Explanation:
According to history and examination, I found that you have a right sided conductive hearing
impairment, which is most likely due to cholesteatoma. Have you heard of this conduction
before? It is a condition in which squamous cell epithelium grow in the middle ear cavity
which is similar to your skin tissue grow. It is locally destructive and that causes your hearing
loss. So it is a serious condition and also increase your risk of intracranial infection by
erosion of the bone. CT-need to access extent of disease.

So I will refer you urgently to the ENT specialist and you will need surgical intervention.
They will also do hearing assessment with audiogram.

! 10!
62
ENT & OPHTHALMOLOGY

Otosclerosis (AMC 54)


GP setting. A young woman who gave birth to her 1st child 1 month ago c/o of hearing loss, which
she first noted about midway through her pregnancy. It has become progressively worse since and
affects both ears. She is otherwise well and her infant (breastfed) is thriving.

Task: further focused Hx concerning her hearing loss. Perform PE and hearing test. Tell the examiner
the type of hearing loss present. Inform the pt of the most likely cause of her hearing loss. Suggest to
the pt what further action is indicated for her hearing loss, including a prognosis.

Hx:
Is it one ear or both ears affected
How severe it is?
Any pain, discharge
Did you hear better with noisy background?
Any past history of infection like otitis media, meningitis
Any history of trauma
Exposure to loud noise
History of medication such as gentamicin
FHx of similar condition mum

O/E:
Expose both ears. Always start with the normal ear.
1. Inspection: any discharge, smelly or not,
Otoscopic examination: Any wax, discharge, eardrum - red, bulging, any mass?
2. Distraction hearing test
Whisper a words to one ear while distracting the other ear. Ask to repeat the word you say.
3. Rinne test (if AC>BC, normal or sensorineural loss)
Tuning fork test: with 256 Hz or 512 Hz
First let the patient feel the vibration on the sternum.
Then ask if he/she can feel the vibration when it is placed on the back of the ear on the mastoid
bone. Instruct him to tell you when he stop hearing. Once he says he cannot hear. Place the tuning
fork in front of the ear and ask him if he can still hear it.
4. Weber test
This time, I will put the tuning fork on your forehead and please tell me which side you hear it louder.
Weber without Weber lateralisation to left Weber lateralisation to
lateralisation right
Rinnie Normal Sensorineural loss in right Sensorineural loss left
AC>BC

Rinnie (Left) Conductive loss in left Conductive +


BC>AC sensorineural loss left

Rinnie (Right) Conductive + Conductive right


BC>AC sensorineural right

Mx:
Otosclerosis is familial (autosomal dominant)
Refer for audiometry test to confirm conductive disorder then refer to specialist for surgery 
stepedectomy and vein grafting

! 11!
Fracture History GP$or$ED$
How$did$it$happen?$ Classify$the$fracture:$close$or$open$
What$is$the$mechanism$of$the$injury?$ If$it$is$closed:$displaced$or$nod$displaced$
$ Treatment:$
I$fell$down$–$ask$Why$did$the$patient$fall?$ 1.$reduce$fracture$if$displaced$
LOC?$Trip?$Car$accident?$Alcohol?$ 2.$immobilize$
Especially$elderly$patients$ 3.$exercise$
$ $
+pain$in$the$hand?$–$trauma$anywhere$else$in$the$ Refer$–$but$put$a$back$slab$–$to$stabilize$the$
body?$ fracture$site,$give$painkiller$$
Did$you$hit$your$head?$ I$cannot$manage$it$here$–$$
$ Undisplaced$–$immobilize$$
Any$pain?$$ Colle’s$–$even$if$undisplaced$–$refer$to$ED$
Swelling?$ Reduction:$under$anaesthesia$
Any$wound?$On$top$of$the$injured$site?$ Local$
Any$deformity?$ Regional$
Bruising?$ General$
Function$of$the$area:$can$he$move$his$arm?$ Aim$of$the$reduction:$make$normal$bone$
Any$numbness$in$the$hand?$ alignments$$
Any$color$changes$in$the$fingers$or$in$the$area$ the$movement$of$the$reduction$is$opposite$the$
Any$pain$in$the$hand$(ischaemic$signs)$ deformity$
$ $
Any$past$history$of$any$condition?$ Open$reduction$–$surgery$indications$
Look$for$pathologic$fractures$ Close$reduction$fails$or$impossible$
Any$$past$surgery?$Anaesthesia$allergy$ $
Any$medications?$Allergies$ Analgesia$important$
$ If$you$were$successful$to$reduce$it$
PE:$ Lost$reduction$after$a$successful$closed$reduction$$
Examine$the$fracture$site:$Look$feel$and$move$ +$multiple$bone$or$soft$tissue$injury$
Look$at$the$site$of$fracture$for$swelling$bruising,$ $
deformity,$color$change$or$any$wound$ After$reducing,$immobilize$the$fracture$
Feel$again$for$swelling,$deformity$or$any$ Splint,$POP$–$back$slab$or$complete$
tenderness,$temperature$–$and$distal$areas$ Prefer$complete$$
Move:$can$you$move$actively$the$hand$ Supracondylar$–$back$slab$for$2$days$and$then$
If$there$is$a$fracture,$displacement,$vascular$ replace$with$complete$
compromise$ Internal$fixation$or$external$fixation$
Examine$the$joints$above$and$below$ $
Fractures$of$the$upper$limb$commonly$associated$ Open$fracture$–$fracture$penetrates$the$skin$
with$dislocations$ If$there$is$a$wound$over$the$fracture$site$
Quick$examination$of$other$parts$of$the$body$ 1. Cleaning$the$wound$with$Normal$saline$
Any$other$sites$injured$ 2. Wound$debridement$(remove$the$necrotic$
Investigation:$Xray$(A/P$and$lateral$+/S$special$ tissue)$
view)$ 3. stabilize$the$fracture$size$through$external$
Scaphoid$–x$ray$$ fixation$
$ 4. close$the$site$of$fracture$–$stitching$or$skin$
If$the$x$–ray$is$normal,$but$there$are$symptoms$of$ graft$
fracture$ 5. Give$antibiotic$
Do$bone$scan$or$MRI$ $
Management:$ POP:$keep$the$hospital$for$2S3$hours$if$nothing$
happens$send$home$
Check$after$24$hours$to$check$for$signs$of$
ischaemia,$if$nothing$wrong,$return$in$2$weeks$
time$–$Fracture$clinic$and$usually$they$follow$it$
up$by$xSray:$Any$signs$of$union$
If$everything$is$alright$–$continue$keeping$the$pop$
In$upper$limbs$–$6$weeks$
Scaphoid$–$8$weeks$
Lower$limbs$–$12S18weeks$
Include$joint$above$and$joint$below$the$fracture$
site.$
Exercise:$
Exercise$fingers$and$shoulders$to$avoid$stiffness$
Complications:$
1.$$joint$stiffness$
2.$$malunion,$non$union,$
3.$$chronic$pain$
Specific$complications$depending$$
Mid$shaft$–radial$nerve$–$drop$hand,$sensory$loss$
at$the$back$of$the$forearm$
Elbow$joint$–$neurovascular$involve$
Neck$of$fibula$–$common$peroneal$nerve$(foot$
drop,$lost$of$sensation$in$the$outer$aspect$of$the$
leg$and$dorsum$of$the$foot)$
Shoulder$–$axillary$nerve$(deltoid$patch$and$
abduction)$
Poster$dislocation$of$hip$–sciatic$nerve$
Forearm:$SUlnar,$median$and$Vascular$–$
ischaemia$and$contracture$–$volkmans$
Colles$–$median$and$avulsion$of$the$extensor$
pollicis$tendon$
$
Fibula$–$above$the$kneeS$cast$
If$above$syndesmosis$–$internal$fixation$
If$below$–$stable$$
$
62
GASTROINTESTINAL TRACT

GORD (AMC142)
Your endoscopy result has come back.
Do you still have the heartburn?
(Draw picture) You have a condition called Gastro Oesophageal Reflux Disease (GORD), it’s
a reflux oesophagitis. There is an inflammation in the lower end of the food pipe by gastric
acid refluxing there from stomach.
Causes:
Acid
Smoking 15-20 cigarettes/day
Alcohol
Overweight (BMI increase)

If we don’t treat, the food pipe will have:


- Multiple ulcers cause bleeding
- Inflammation causes muscular spasm (longstanding causes stricture)
- The cell will change to Barrett oesophagus and lead to malignancy (oesophagus has
squamous epithelium but 3 cm of the lower end is columnar same as the stomach)

Mx:
Reassure patient that it can be treated and reversible
At your stage, it’s reversible and we need to work together. I will start you on medication
(Omeprazole). But the most important thing is lifestyle modification

Lifestyle modification:
- Reduce alcohol, quit smoking, reduce BMI: Physical exercise, 30 minutes walk
- Avoid NSAIDs; Aspirin for migraine has to be changed with an alternative medication
- Avoid spicy food
- Refer to dietician
- Posture while sleeping, elevate the head with 2 pillows

Medical treatment:
- Proton pump inhibitor: Omeprazole 20 mg once daily
- Follow-up to see if the medical therapy is working or not, if not repeat endoscopy
- May need surgery (last option)  fundoplication (keyhole surgery)
- Hiatus hernia is common(80%), if para oesophageal hernia has to go for surgery

Indication for Surgery


- Resistant to treatment  patient with persistent symptoms plus dysphagia, haematemesis,
weight loss  surgery
- Large paraoesophageal hernia
- If the medicine has side effect like persistent diarrhoea

Side effect of surgery:


- Bloating
- Decrease ability to belching or burp
- Early satiety

If endoscopy normal, need to go for pH profile, introduce device & monitor (manometry)
!

! 1!
63
GASTROINTESTINAL TRACT

Haemorrhoids
A 25 year-old man complaining of rectal bleeding in the last 3 weeks.

Task: take history, physical examination, diagnosis

Hx:
How long have you been suffer from this condition? ---3 week, I have chronic constipation
Does it happen when you open your bowel or continually?
How much is the bleeding?
What colour is it, fresh red or dark red?
Anything else in the stool, like mucus?
Itchiness? Pain?
Do you have any sense of incomplete emptying of your bowel?
Have you noticed any change of your bowel habit?
Any abdominal pain, weight loss, any unexplained tiredness?
Any alteration of bowel habits?---constipation followed by diarrhoea
Sexual orientation
Any chronic cough?
Are you on any medication---NSAID, Panadeine forte (codeine causes constipation)
If female, ask for any difficult labour
What’s your job (nature of the job)?
Tell me about your diet, any vegetable or fruit?
How much water do you drink a day?
FHx
SAD + coffee intake

O/E:
GA; VS; Chest, heart, abdomen, LN
Rectal exam:
- Inspection: any protruding mass, bleeding, fissure
- Palpation: comment on anal sphincter tone, any palpable mass, tenderness, blood on
examining finger, prostate,

Ix: First proctoscopy, sigmoidoscopy if any suspicious finding


Indication of Colonoscopy:
- Family history of colon cancer
- Abdominal pain and other symptoms suggesting cancer
- Over 50 years old
- Sigmoidoscopy  found blood and mucus

Explanation:
From the history & examination, you’ve what we called haemorrhoids (piles). It’s varicose
veins of rectal and anal area which can prolapse outside the anus like your condition.. It looks
like grape-like lumps.
There are 3 different types:
1. Internal (within the anus)  not painful , only notice when they bleed
2. Prolapsed (protrude through the anus when pass stool)  painful
3. External (perianal haematoma) painful haemorrhages under the skin around the anus)

! 2!
GASTROINTESTINAL TRACT

Causes:
Constipation (excessive straining)
Familial
Job  heavy manual work, sitting for long periods
Pregnancy

Symptoms: bleeding, pain, mucus discharge, itching, incomplete bowel evacuation and pain
Risk – repeated bleeding  anemia

Mx:
Prevention is the best Tx  lifestyle modification
- High fibre diet with plenty of fresh fruit, veggies and wholegrain cereals or bran.
- Try to complete your bowel action within a few minutes and avoid using laxatives.
- Clean your anal area with soft tissue
- Try to reduce the usage of NSAID, Panadeine forte as they cause constipation.

I’ll prescribe painkillers and some cream Astringent cream (absorb excessive fluid and
relieve congestion) helps to shrink haemorrhoids

I’ll refer you to a surgeon


1. Sclerotherapy – we’ll inject substance in your bulge mass to shrink
2. Rubber band ligation
3. Haemorrhoidectomy – surgical operation
!

! 3!
64
GASTROINTESTINAL TRACT

Palliative Care (Pancreatic Cancer) (AMC 124)


A 75 year-old man who has inoperable pancreatic cancer. His daughter came from interstate
asking for palliative care for her Father’s condition.

Task: counsel the patient’s daughter and answer daughter’s questions.

Patient’s Questions:
- How long can he live
- What is palliative care
- What will you do for his depression
- What about pain management, how will you manage my Father’s pain
- What are the side effects of morphine

Ask examiner if the daughter has authority to talk about her Father’s condition

Your Father has a condition we call end-stage pancreatic cancer which cannot be cured.
I’m very sorry to say that. Do you know about that?
I know but he didn’t realize that he’s dying, please don’t tell my Father
As a patient, he has a right to know about his condition, we have to tell the patient about his
condition. If he wants, he can arrange his will before the time of his death. Although we can’t
add the days to his life but we can add better life to his days. We can improve the quality of
life. He will be managed by a palliative care team with multidisciplinary approach to
minimize his pain. Step by step.
First simple analgesic
Mild opioid
Strong opioid
We’ll give the right drugs with the right dose at the right time that will reduce pain 80-90%.

Apart from pain management, we’ll give him psychological support.


I need to know if your Father is depressed or not---yes, he’s depressed and wants to die, he
has suicidal idea (examiner will tell this).

Palliative Care Team


He’ll also be managed by psychiatry team, usually they will give antidepressant.
They also have spiritual worker, social worker who will look after your Father and involve
oncologist team.
Nurse
If need financial support, they will arrange
If you want you can join Australian Cancer Council Society

How long has my Father to live?


Very hard to answer because it varies from person to person.

! 4!
GASTROINTESTINAL TRACT

If you really want to know about it, the oncologist may give some idea, but no one can
predict it with certainty.

How will the pain be managed?


With painkiller step by step according to patient’s condition (done by pain specialist)
PAIN MANAGEMENT
- Tricyclic antidepressant
- Paracetamol, NSAID
- Codeine, Tramal, Naproxen
- Morphine, Fentanyl, Hydromorphin, Doxycodeine, Methadone

Morphine not >100 mg per dose  will cause respiratory depression

Side Effects of Morphine?


- Addiction
- Constipation
- Nausea
- Vomiting
- Urinary retention

Will give laxatives and antiemetics to minimize the side effects.

Aim of palliative care:


- To give optimal quality of life
- Control symptoms
- Give patient support
- Give carers support
Give time/idea to patient for advance planning

! 5!
65
GASTROINTESTINAL TRACT

Acute Appendicitis (AMC 140)


A 24 year-old female, a medical student, comes to ED due to right abdominal pain.

Task: Hx, ask physical examination findings, arrange Ix, diagnosis & management.

DDx:
Ectopic pregnancy
Torsion / rupture of ovarian cyst
PID
Renal stone
UTI
Appendicitis
Intestinal obstruction

Hx:
Pain Q – LOTS RADIO
Any bowel changes recently?
Menstrual history  any missed pills
Any urine problem – burning sensation
Sexual history
STD or PID history
Any vaginal discharge/bleeding currently
Any similar episode in the past
General health
SADMA
Social history

O/E:
GA: pale, dehydrated, distressed
VS: BP, PR, RR, T
Abdominal examination
Inspection
- Scar; Bruises; Pigmentation; Distension; Visible peristalsis
Palpation
- Mass; Organomegaly; Guarding; Rigidity
- Mc Burney sign (RIF tenderness)
- Rovsing sign (Rebound tenderness)
- Psoas sign  hip Flexion give pain in RIF
- Obturator sign  Flexion in hip and knee, and internal rotation increase pain
PR Exam
PV Exam

Ix:
Urine dipstick
Urine BHCG  Serum BHCG
FBE, CRP, ESR
U&E
US

! 6!
GASTROINTESTINAL TRACT

Explanation:
I suspected you have appendicitis.
I’ll call the surgical registrar to assess your condition.
We’ll put IV cannula and fluid, nil by mouth as you might need an operation.
Surgical registrar might do an US and decide to do an operation.

Appendicitis is a common condition.


Appendix is a small worm shaped pouch, has no particular use but can cause problem.
In Australia 1 in 7 can have appendicitis.
If untreated  30% rupture, can cause all abdomen infected.

Complication of operation:
Very rare
But maybe infection, bleeding, clotting problem, injury to other organ
But we can prevent it.
We can give antibiotic to prevent infection, compression stocking and early mobilization can
prevent clotting problem.
You can be discharged from the hospital when your bowel gain normal function.
You can go to gym after 2-3 weeks operation.
!

! 7!
66
GASTROINTESTINAL TRACT

Globus Hystericus (AMC 7)


GP setting. A 26 year-old male presented with difficulty of swallowing.

Task: take history, physical examination, diagnosis, investigation, and management.

Causes:
- Functional – muscle tension
- Neurological – stroke, myasthenia gravis
- Mechanical – stricture, tumor, goiter

Hx:
When and how did it start?
Is it with liquid or fluid?
Could you please tell me exactly what do you feel? (I feel a lump)
At what level?
What makes it better/worse?
Any heartburn, any thickness of skin? (scleroderma)
Does food come back again from your mouth or nose? (regurgitation)
Have you noticed any weight loss, abdominal pain or cramps?
Any nausea, vomiting, any blood in the vomitus?
Any changes in bowel and waterworks?
Any headache, weakness in your arms and legs?
Any abnormal feeling – numbness, tingling in your face, limbs?
PMHx: Chronic condition; Thyroid problem; Abdominal surgery
SADMA
FHx of similar condition or serious problem like stroke, cancer, mental illness
Any stressful situation - Financial problem, job, family
What about your mood, sleep, appetite

O/E:
GA: pale; BMI; Any skin changes  scleroderma
VS
Systemic
- Cranial nerve exam
- Oesophageal obstruction test: give a glass of water, put stethoscope on left upper
quadrant, ask to swallow, you will hear noise in 10 seconds  no obstruction, test (-)
- Throat inspection and laryngoscopy

Explanation:
All examination is normal which is good news
Most likely stress if causing these problems but we need to rule out organic cause
I’d like to do some blood tests, chest X-ray
I’ll refer you to specialist, you may need endoscopy
Laryngoscopy and pharyngoscopy will be done
Most likely all investigations will come back normal
This condition is not uncommon
Let me assure you, it will settle down with time

! 8!
67
GASTROINTESTINAL TRACT

Oesophageal Carcinoma (AMC 133)


GP setting. A 50 yo male presented with a 15-yr Hx of reflux oesophagitis and heartburn.
Recently he’s having increasing difficulty to swallow food, loss of appetite, lost 3 kg. Given
omperazole for the last 12 months.

Task: relevant Hx, ask for exam finding, arrange further Ix if needed, explain diagnosis.

DDx:
Stroke
Parkinson disease
Medication induced like neuroleptic
Radiation exposure to the head & neck
Pharyngeal pouches
Mechanical obstruction in oropharyngeal in general (malignancy)
Oesophageal causes
- solids & liquids
o progressive: achalasia (yellowish skin, masked face, fish mouth), scleroderma
o intermittent: diffuse oesophageal spasm
- solids only
o progressive: oesophageal carcinoma
o intermittent: lower oesophageal: web or ring, peptic stricture

Hx:
How long have you noticed that dysphagia or inability to swallow? I feel like something is
stucked in the middle of the chest. This happened 3 times in 3 different occasions.
Is it worsening since noticed or the same?
Do you have any difficulties in swallow solid, liquids or both?
Does swallowing associated with pain or painless?
Can you eat a full meal? (can’t finish a regular meal)
Do you think you can’t finish your meal b/o your appetite?
Any change in the appetite?
Any associated cough?
Any recent chest infection or sore throat?
Any hoarseness of voice?
Any lumps in the body?
Any weight loss? How many kg, when started, over how many months?
From your notes you were Dx with reflux oesophagitis, any change in your reflux symptoms?
Any associated abdominal pain or severe regurgitation, any vomiting?
Did you notice any aggravating or relieving symptoms to your dysphagia?
Any bad odour from mouth? (pouch ---halitosis)
Any radiation exposure?
PMHx of stroke or TIA, cancer?
Are you on any medications?
Any stresses at home or at work?
Any abnormal feeling of lump in the throat? (globus hystericus)
Any family history of malignancies?
SADMA

! 9!
GASTROINTESTINAL TRACT

O/E:
GA: any cachexia, any pallor, BMI
VS
Cranial nerve examination
Hand for any clubbing, tremors
Mouth: perform oesophageal test for obstructiongive the pt a glass of water & ask him to
drink, put stethoscope over the left upper quadrant of abdomen & measure the time ---should
hear the splash or murmur sound when water is passing within 7-10 seconds. If delayed, there
is partial obstruction
Neck: LN, Thyroid enlargement
Lung
Any organomegaly, tenderness, ascites, any mass

Explanation:
From my history & exam finding with a history of aggressive dysphagia & weight loss, the
most likely diagnosis is an oesophageal lesion which is probably a neoplasm of malignant
origin or nature. As the physical finding are none contributory, I think the patient needs
further investigation. I will go for a barium swallow first to help identify the site of the
stricture if it’s intra or extraluminal & I will refer my patient for a specialist
gastroenterologist to have the most definitive investigations which is the endoscopy. The
specialist may arrange also CT scan while I will run the basic blood test.
Other DD which is unlikely:
- Achalasia
- Scleroderma
- Oesophageal spasm
- Ring or web of oesophagus
- Oesophageal stricture
!

! 10!
68
GASTROINTESTINAL TRACT

Intestinal Perforation
A 50 year-old male came to ED because of sudden onset of severe abdominal pain.

Task: Take further history, ask examination, investigation and management.

Acute pain + left side radiation + fever = Diverticulitis

Hx: (stable or not, offer painkiller)


PAIN Q
Somatic pain  sharp or severe, usually in the right or left side
Visceral pain  vague, midline, epigastric, periumbilical, suprapubic
Pain is continuous and progressively increasing
GIT – how’s your appetite
Any nausea, vomiting, heartburn, jaundice
Abdominal distension
Bowel motionfresh blood in the stool
Constipation  for 5 yrs
Weight change
Urinary symptoms – loin pain, haematuria, dysuria
Respiratory – cough, shortness of breath, chest pain
Temperature
DM complication - DKA
Any abdominal operation before
Past history of peptic ulcer or hernia
SADMA

O/E:
GA: pale, posture
VS, Chest,
Abdomen – rebound tenderness, bowel sound sluggish
Per rectal normal (no blood)

Ix:
- FBE
- ESR, CRP
- Stool (pus and blood), blood culture
- Abdominal US / CT (detect fistula or perforation)
- Erect CXR air under diaphragm
- Erect and supine abdominal X-ray

Explanation:
After having X-ray, we found one of your part of your gut perforated or ruptured for some reason
I will call surgical registrar or surgeon
Nil by mouth
Nasogastric Tube for decompression
IV line – IV fluid
Antibiotics  Ampicillin/Amoxycillin + Gentamicin + Metronidazole
Urgent laparotomy  resection of affected segment of bowel – if needed they will do a temporary
colostomy in proximal colon
!

! 11!
69
GASTROINTESTINAL TRACT

Lower GI Bleeding (Diverticulosis)


A 63 yo man presented to your GP clinic because of rectal bleeding that started yesterday.

Task: take further history, PE, investigations, manage.

Hx: (Is my patient hemodynamically stable?)


Bleeding Qs Onset; Frequency; Colour – fresh or coloured
Mixed with the stool or a gush of blood after defecation?
Mucus? Painful? Itchness?
First time or happened before?
Have you noticed any masses or piles around the anus area? (haemorrhoids)
GIT
- Any mouth changes, ulceration
- Any hardness of epigastric or abdominal pain
- Change in appetite and wt
- Tummy distension
- How is your bowel habits -- constipation?
- Have you noticed any recent change in bowel habits?
- What’s your normal diet like?
Any fever
System review quickly  chest pain, shortness of breath, palpitation
Past medical history
FHx of serious condition like bowel cancers
SADMA

O/E:
GA: pale, BMI
VS, Chest, Heart, Abdomen
Rectal exam
- Haemorrhoids
- Any rectal pathology
- Tip of the finger when you removed the gloves if there is any blood
Proctoscopy

Explanation:
I need to investigate because you’re suffering from low GI bleeding which can be due to a
variety reasons. I need to do some more tests:
- FBE: iron def anemia
- Barium enema & colonoscopy if the patient is not having diverticulitis  Barium enema
showed colonic spasm & diverticula; Colonoscopy to exclude cancer. Cancer can cause
diverticular disease with the obstruction & increase of pressure
Segment of diverticula  narrow & thick (increase intracolonic pressure)

Diverticular disease or diverticulosis, finger-like outpouching from the wall of the bowel,
usually multiple. Reason not known, usually happen in patient chronically constipated and
less fibre diet. Increase of pressure in the lumen.

Can be asymptomatic discovered by chance


Symptomatic with left lower abdominal pain, constipation, sometimes bleeding

! 12!
GASTROINTESTINAL TRACT

Can present with infection  Diverticulitis with fever & systemic symptoms
Can stay uncomplicated
Can cause intra abdominal abscess
Can perforate and secondary infection
Fistula formation

Does it cause cancer? NO --- non pre-malignant

Uncomplicated usually symptomatic like take high fibres in the diet.


Pain  analgesics (no morphine)
Mebeverine  anti spasmodic for colic also for irritable bowel
Stool softener  lactulose (non stimulating)

Chronic symptoms not responding to treatment  indication for surgery


1. Diverticulitis not responding to treatment
2. Recurrent diverticulitis
3. Suspect cancer

Surgery  resect the segment of the bowel which has the diverticulum (diverticular).
!

! 13!
70
GASTROINTESTINAL TRACT

Diverticulosis
A 57 yo lady presented with constipation. Her father Dx with colon cancer at the age of 65,
he’s alive and well, he’s now 90 yo. You sent her for colonoscopy and the diagnosis is in the
picture. You have called her to discuss the diagnosis with her.

Task: Explain the diagnosis, manage, answer her questions.

The result showed that you have a diverticulosis (symptomless)


It’s common, 1 in 3 person >60 years old have diverticulosis (30% of population)

In your condition, you have chronic constipation.


This constipation, how many years do you have constipation----many years

Hard stool creates some pressure in the colon in the weak points when you pass stool.
Because of these pressures, the lining start to blow up into a diverticulum (balloon-like).
There is no symptom with this.
But some stool can block the diverticulum and become a good media for the bacteria and
become diverticulitis, inflammation.
Bleeding will result from this and may cause infection.

Risk factors
• The main risk factors are age over 50 years and low dietary fibre.
• Obesity is an important risk factor in young people.
• Complicated diverticular disease has an increased frequency in patients who smoke, use
non-steroidal anti-inflammatory drugs (NSAIDs) and paracetamol, and those who are
obese and have low-fibre diets

Try to adjust your food, more fibres in your meals and drink plenty of water.
Even when you eat cereal, try to find one kind with plenty of fibres
Try to use wholemeal bread or multigrain.
Try to have enough amount of fresh fruits and vegetables everyday.
Fibres at the beginning make you feel uncomfortable (bloating) for a couple of weeks but
later your bowel will adapt and settle down.

! 14!
GASTROINTESTINAL TRACT

I’d like to inform you that you’re at risk of bleeding.


If you any change in your bowel habit, abdominal pain + fever, please come back and
see me, you may have an infection because of the blockage in the opening of the
diverticulum.
I’ll admit you & give antibiotics.

Ix: Barium enema, X-ray, Colonoscopy, CT if perforation

X-ray of diverticulosis.

http://www.patient.co.uk/doctor/Diverticular-Disease.htm

! 15!
71
GASTROINTESTINAL TRACT

Colonic Villous Adenoma


A 45 year-old male presented with rectal bleeding. Colonoscopy showed 2 polyps, excised
and the morphology showed villous adenoma.

Task: answer the patient’s questions, discuss the risk factors, talk about prevention.

Villous is finger-like shaped polyp


Has a high risk of developing into cancer

I know that you’re here to discuss about the result of the colonoscopy. The morphology of the
polyps which we took out showed a condition called villous adenoma. Adenoma by itself is a
benign condition but carries some risks of bowel cancer, which depends on the size and the
type of adenoma. Villous adenoma carries a higher risk of cancer. It’s a common condition.
It’s most likely silent with periodic rectal bleeding.

We have some screening tests for early detection of this condition called FOBT which test for
the presence of blood in the stool. We suggest this test to be done every 2 years after 50 years
of age.

Villous adenoma carries also other risks:


- You can have mucus diarrhoea
- Loss of potassium and electrolytes
- Weight loss
- Risk of recurrence

If it’s only 1 polyp, risk of recurrence is 25%


If it’s 2 polyps, it has 50% of recurrence
Because you’re in a risk group, we need to repeat colonoscopy every year

! 16!
GASTROINTESTINAL TRACT

Do I need surgery in the future?


If you have the polyps again, the surgeon will do again colonoscopy and polypectomy, send
to the pathology to be examined. If it shows abnormal changes, you may need a surgery. Now
your colon is free of polyps but keep an eye on bleeding or diarrhoea. If there are any
symptom or if you have any concerns, please come back to see me. If not, the next
colonoscopy will be in 1 year time.

If there is villous adenoma or a family history of familial adenomatous polyposis,


colonoscopy should be done for family members starting 10 years earlier. Normal population
start at 50 years, for this population start at 40 years.

Arrange for other members of the family screening test.


!

! 17!
72
GASTROINTESTINAL TRACT

Colon Cancer
A 48 year-old female has come for a biopsy result which shows adenocarcinoma of colorectal
area. Her father had colon cancer at the age of 58 years old.

Task: explain the result for biopsy, management and answer patient’s questions.

The biopsy result showed there is a nasty growth in your large bowel

The good news is that early detection and treatment has the good prognosis

Stage 1: Confined to mucosa


Stage 2: Mucosa + muscular layer
Stage 3: Mucosa + muscular layer + lymph node involvement
Stage 4: Metastasis

5-year survival rate


Stage 1: 90-95%
Stage 2: 75-80%
Stage 3: 50%
Stage 4: 25-30%

I’ll refer you to a bowel cancer specialist.


They will do an elective surgery for you with adjuvant chemotherapy ± radiotherapy.
They will decide according to your condition.
Adjuvant chemo +/- radiotherapy improve the survival rate.

Follow-up after therapy


Investigations needed to be done up to the age of 75 years
FBE
U&E
LFT every 3 months
CEA every 6 months
Chest X-ray or CT scan of abdomen every year
Fecal occult blood every year
Colonoscopy 1-2 year

What about my kids?


How old are they?
Apart from your father, anyone else has bowel cancer in the family?
Any genetic test done to your family?
Because you and your father has bowel cancer, they have moderate risk of having cancer
They need to do regular screening test, 10 years earlier than your age
Dietary advice, lifestyle modification, exercise
If there is any concern need to see a doctor

What about my siblings?


How old are they?
They have high risk. Refer to clinical genetic services and cancer genetic services, the
specialist will do some test and make up a management plan.

! 18!
GASTROINTESTINAL TRACT

Screening Test:
Normal population: FOBT every 2 year from the age of 50-80 years

Category I – Mild risk


First degree relative diagnosed with colon cancer after 55 years
FOBT every 2 years + colonoscopy every 5 years from 50 years old
Go to doctor if any symptoms

Category II – Moderate risk


First-degree relative diagnosed with colon cancer less than 55 years old or
One or two first degree or second degree on the same side diagnosed with colon cancer 
colonoscopy every 5 years, start 10 years earlier than the relative with cancer
If colonoscopy is not available, double contrast barium enema

Category III – High risk


If 3 or more persons in the family have cancer, genetic testing positive
Other types of cancer – breast, ovarian
Familial adenopolyposis coli
Refer for genetic counselling
Colonoscopy every 1-2 years at the age of 25 years or even younger
!

! 19!
73
GASTROINTESTINAL TRACT

Duke’s C Colon Carcinoma


A 60 yo man who presented with an acute large bowel obstruction 3 days ago, requiring
laparotomy. The cause was a stenotic lesion in the sigmoid colon. There was no evidence of
metastases during the operation. Histology report:
There is a 15 cm long specimen of sigmoid colon with a 3x3 cm large tumour, having spread
through the bowel wall, invading through the serosa and the muscularis propria. The
histopathology reveals an adenocarcinoma and seems to have been removed with sufficient
safety margin. There are 6 adjacent lymph nodes of which two show some infiltration with
tumour cells.

Task: Explain the pathology report (provided) to the son, outline further Mx and Tx

Mx:
1. Pt will have to stay in hospital for about 1 week unless any complications arise.
2. The total recovery time to return to her pre-op status will take up to 1 month.
3. The oncologist / specialist will organize for chemotherapy (standard management of
Duke’s C, with good prognosis). This would most likely happen as an outpatient
treatment, although it might depend on side effect of the treatment like nausea, vomiting,
general malaise, hair loss etc.
4. Follow-up will be in the outpatients department every 3 months for about 2 years, then
yearly and she will have regular tests during that time to monitor the situation, especially
stool for faecal occult blood, the carcinoembryonic antigen (CEA), colonoscopies and
scanning for metastases.
5. The five year survival is about 30 to 50%

! 20!
74
GASTROINTESTINAL TRACT

Haemochromatosis
GP setting. A 62 yo retired accountant, John, comes for a general check-up in his first year of
retirement. He has never been sick, no operations. Always has been fit and well although in the last 10
years he has gained about 10 kg b/o his workload and reduced physical activities. However, since
retirement 12 mths ago he has taken up physical exercise and lost 6 kg. But he feels that his energy
levels are low and he finds it more and more difficult to do the things he would like to do.

Task: further Hx, PE and Ix, answer patient’s and examiner’s Qs

HOPC: John noticed increasingly a general lack of energy, if not even lethargy for the last 3 to 5
mths. He is chronically tired, although he seems to sleep well. He has to pass urine more often,
especially at night time & he is often thirsty. His libido has been reduced but he thinks all these things
are probably normal for his age. His wife and friends don’t understand. They actually feel he looks
well, quite tanned as if he was on chronic holidays.

O/E:
Patient looks well and quite tanned, no anaemia, jaundice or cyanosis.
BP 140/80, P 72 reg, RR 18, SaO2 98% on RA, afebrile.
Except for hepatomegaly with a firm non tender liver edge there are no other pathological findings on
physical examination.
Ix:
Urine office test shows high glucose.
Other tests can be sent off but results will not be available.
Most likely Dx: Haemochromatosis
DDx: diabetes mellitus, hypothyroidism, Addison-, Cushing disease

Diagnostic tests for haemochromatosis?


• Iron studies: serum iron elevated
Serum transferrin saturation (>45%) , transferrin and ferritin (>250 micro
gram/L) increased
• LFTs: elevated
• Liver biopsy: demonstrates hepatic siderosis and cirrhosis
• Genetic studies: HFE gene can show mutations
• Screen family members

Hereditary haemochromatosis is a common autosomal recessive disorder with an incidence of 1:400.


The total body iron concentration is increased and excess iron is deposited in organs.
• Liver - siderosis and cirrhosis, later hepatoma!  signs of chronic liver disease
• Skin - pigmentation / bronze  tanned skin
• Pancreas - “bronze” diabetes  lethargy , polyuria, polydipsia
• Heart - cardiomyopathy
• Joints - arthropathy  arthralgia
• Pituitary - hypogonadism  impotence

Tx:
• PHLEBOTOMY: weekly venesection; 500 mls of blood removes 250 mg of iron.
Until iron levels are normal, then every 3 to 4 months to
maintain iron level below 150 microgram/L.
• Desferrioxamine (chelating agent) removes iron via kidneys
• Diabetes mellitus, cardiomyopathy etc. to be treated when indicated

! 21!
75
GASTROINTESTINAL TRACT

Elevated Liver Enzymes


Mr Smith, 55 years old who presented to you for the first time with a letter from the previous
GP saying that Mr Smith recently moved to your area and the only medical issue he has is
elevated liver enzymes for more than 4 years with no special reason.

Task: Hx, PE findings, order Ix, explain the DDx and your Dx, Management.

Hepatocellular damage can be caused by:


1. Alcohol
2. Drug
3. Hepatitis
4. Fatty liver
5. Hemochromatosis
6. Autoimmune Hepatitis
7. Alpha 1 anti trypsin deficiency
8. Wilson’s disease

Obstructive
1. Drug induced
2. Choledocolithiasis
3. Neoplastic liver
4. Pancreatic cancer
5. Primary sclerosing cholangitis
6. Primary biliary cirrhosis
7. Sarcoidosis

Hx:
Alcohol  how much
Drug  recreational, IV drug usage
Hepatitis  PHx of surgery, blood transfusion, IV drug, tattoo, body piercing, travelling
overseas
Sexual Hx  partner, STD
Any other c/o  tiredness, change colour of the skin, any problem with fertility, arthritis
joint paint, heart disease, cardiomyopathy, gallbladder stone, any SLE
Any Hx of inflammatory bowel disease (ulcerative colitis or Crohn’s disease)
Any abdominal pain, tiredness, change of bowel habits
Loss of appetite, wt loss, lumps in the body
FHx of  hepatitis, liver disease,
Life style  weight gain, diet, fatty food, cholesterol, fat, lipids triglycerides
Stress, mood
Have you seen any specialist before?

Positive finding: No symptoms, just tiredness and elevated liver enzymes, BMI 32

Explanation:
Mr Smith, I don’t know what’s causing you, I need to order some other tests.

I want all hepatitis markers, cytomegalo, Epstein Barr virus serology

! 22!
GASTROINTESTINAL TRACT

Haemochromatosis  ask for iron study (serum ferintin and transferring saturation)  VERY
HIGH, transferrin saturation very high
Autoimmune hepatitis  ask for autoimmune antibodies
Alpha 1 antitrypsin deficiency
Serum copper and serum ceruloplasmin (protein which carry the copper)
U/S
You have a condition named haemochromatosis. It’s a hereditary/genetic disease, runs in the
family. Autosomal recessive disorder.
It’s a common disease.
Let me reassure you that we will try to control.
We can prevent the complication but it’s not curable.
Excessive absorption of iron from your bowel go to different parts of your body which is the
joints (arthritis), heart (myocarditis), liver (hepatoma), testis (testicular atrophy, infertility),
skin (bronze skin), pancreas (bronze DM)
Follow up of liver enzymes
Give donation -- venesection --- blood bank, take some blood every now and then depending
on iron study, continue on and off forever (iron study high  venesection; iron normal  no)

Diagnosed for the first time should be refered to gastroenterologist

Follow up is important

Ultrasound every few years for liver changes (cancer)


Screen family (first degree relative  only adults) for iron study

High GGT
o alcohol
o pregnancy
o phenytoin
o pancreatitis

Crestor, Lipitor  mau cause elevated liver enzyme secondary to anti cholesterol medication

! 23!
76
GASTROINTESTINAL TRACT

Biliary Colic
A 30 yo female c/o colicky epigastric pain. Last night she ate fried chicken, and US showed
gallstones.

Task: take history, explain the US result and management.

Risk factors: Fat Fertile Female Forty Flatulence (bloating)

Colic biliary colic


Colic + Fever cholecytitis
Colic + Fever + jaundice  choledocholithiasis
Colic + Fever + jaundice + chills and rigors cholangitis

Hx:
Pain Q - LOTS RADIO
Any aggravating and relieving factors
Pain radiates to back or shoulder
First time or not
Any nausea, fever, jaundice – yellow skin, urine color changes – dark urine
Fever, chills and rigor
Any bloating, pale stool
General health
Any family history of similar problem
SADMA
How many children do you have?

Explanation:
USG showed some gallstones.
They are small, hard stones developed in your gall bladder.
Gall bladder normally collect bile which is secreted from the liver.
Bile is important for digestion.
Sometimes bile precipitated in the gall bladder and stones are formed.
It is a very common condition.
It can cause pain in your tummy and we call it biliary colic.

Complications:
It can cause inflammation of the gall bladder, obstruction, infection and pancreatitis.
You can have fever, yellow skin, urine colour change to dark colour and pale stool.

To avoid further problem, your gall bladder with stones need to be removed.
The best time for the surgery is before other complications start.
We’ll do a laparoscopic (keyhole) surgery.
You will need to stay in hospital for 1-2 days and you will get a full recovery within a week.

For diet: please avoid fatty food, and over eating.


If you have pain now, I will give you pain killer and refer you to the surgeon and give reading
material.
Antispasmotic-mebeverine for bloating

Red flags:
Signs of infection: fever, severe vomiting
Signs of dehydration
Need admission – IV fluid, antibiotics

! 24!
77
GASTROINTESTINAL TRACT

Cholangitis
A 50 year-old male came to your ED with upper abdominal pain and fever for the last 2 days.
You’re an HMO in ED. Manage the case.

DDx:
- Right lower lobe pneumonia
- Cholecystitis
- Hepatitis B
- Pancreatitis
- Peptic ulcer
- Intestinal obstruction
- Ischaemic colitis
- Mesenteric ischaemia
- DKA

Charcot’s TRIAD: Fever + Jaundice + Abdominal pain

Hx: (stable or not, offer painkiller)


Pain Q: onset, type, severity, radiation, association, duration, offset
Fever, Chills and rigors
Jaundice: eyes, skin, dark urine and pale stool
Itchiness
Hepatitis
Travel history
Unprotected sex, IV drug abuse, Blood transfusion, Tattoo, piercing
Alcohol
SADMA
Past medical history of gallstones
Gall bladder removal (Common bile duct can form stones)

O/E:
GA: 5F – female, fat, forty, fertile, fair skin
VS: yellow skin
BMI
Signs of chronic liver disease
Abdominal exam: distension, prominent veins, feasible mass
Any tenderness, rebound, hernial orifices, Murphy’s sign (only if there is gall bladder)

Explanation:
The most likely diagnosis is cholangitis – draw a picture.
Most common cause is the obstruction which is caused by stones, precipitated by bile stasis.
Bile flow is not good  stones, obstruction and stasis of the bile.
But not 100% sure, so I want to do some Ix:
FBE
LFT
U&E
CRP
Amylase and lipase

! 25!
GASTROINTESTINAL TRACT

Coagulation profile
Blood culture (because of chills and rigors)
Urine microscopy and culture and sensitivity
Urine dipstick (bilirubin, nitrate. WBC, RBC)
X-ray to exclude pneumonia
US

Mx:
- I’m going to call the surgical registrar who will come & assess you
- If cholangitis, most likely you need Admission
- Nil by mouth
- IV Fluid
- Analgesic
- Antibiotics  Ceftriaxon + Metronidazole or Amoxycillin + Gentamicin +/-
Metronidazole
- ERCP (diagnostic and therapeutic)  it’s a flexible tube with camera, pass through
your mouth to your stomach and assess the opening (sphincter) between your
common bile duct & intestine. If necessary, if there’s obstruction, they may make a
cut in the sphincter or this opening to remove the stone in the basket
- MRCP (very sensitive – diagnostic)

Am I going to be all right?


We have very good experienced surgeons, you will be taken care of.

! 26!
78
GASTROINTESTINAL TRACT

Jaundice After Cholecystectomy


A middle aged man came to your GP practice with abdominal pain, fever and jaundice.

Task: take history, examination findings and discuss the management with the examiner.

DDx:
Cholangitis
Choledocholithiasis
Pancreatitis
Hepatitis
Liver metastasis
Pancreatic cancer

Ix: FBE, ESR, CRP, Blood culture, LFT, Urine dipstick, US, ERCP, Gold standard MRCP

Mx:
Admission
Nasogastric tube
IV fluid
Cannula
Antibiotic: Metronidazole and Ceftriaxone

! 27!
79
GASTROINTESTINAL TRACT

Acute Cholecystitis Or Cholangitis


ED setting. 49 yo Mrs. Jones presented with severe abdominal pain which started suddenly about one hour after her
dinner (6 hours ago) and is still there. She tried some panadeine forte at home but it hardly helped. She could not get
comfortable at all although found a little relief by curling up in bed. She felt nauseated, took some Maxolon but later
vomited a couple of times. When she started to feel quite hot her husband decided to bring her to hospital because he
had never seen her that sick before.
Task: Hx, perform PE, discuss most likely Dx with pt, and explain Tx options

HOPC:As above. The pain is constant in the R upper abdomen, radiating to the R side of the back, worse on
inspiration. Threw up undigested dinner which was quite a fatty meal of lamb shanks. Later she brought up yellowish
fluid. She has had much less painful but similar episodes over the last 12 months but did not think much about it at the
time. Today she feel really sick and she had some shivering during the night.

O/E:
Mrs. Jones looks generally unwell, BP 110/75, P 88, T 38,3, RR 20. Localised tenderness (midpoint of the right
subcostal margin; MURPHY.s sign!). RUQ guarding and rigidity with rebound tenderness.

Differential diagnosis:
• Perforated or peptic ulcer
• AMI
• Pancreatitis
• Hiatal hernia
• Right lower lobe pneumonia
• Appendicitis
• Hepatitis
• ectopic
• Herpes zoster

Ix:
• FBE, LFT’s, Lipase and bilirubin
• URINE (bilirubin)
• ECG and CXR
• US shows evidence of thickened gall bladder wall and multiple gallstones in it
• HIDA nuclear scan: a substance called HIDA (hexa iminodiacetic acid) is injected into the patient
and then the activity of the gallbladder before and after the injection is measured.
• Pregnancy test

MANAGEMENT:
• Nil orally +/- NG tube for gastric suction
• Iv fluids and analgesia (morphine 2.5 to 5 mg. repeated prn,)
• Iv antibiotics (1 g amoxy/ampicillin,iv 6/24; plus gentamycin 4-6 mg/kg iv daily)
• SURGICAL OPTIONS:
1. Conservatively
2. ERCP
3. early open or laparoscopic cholecystectomy (if symptoms less than 72 hours)
4. delayed surgery (if symptoms more than 72 hours)

Complications:
• perforation
• subphrenic abscess
• gall stone ileus

! 28!
80
GASTROINTESTINAL TRACT

Pancreatic Pseudocyst
A 56 year-old female 2 years after laparoscopic cholecystectomy complained of recurrent abdominal
pain, bloating & discomfort.

Task: history, examination findings, interpretation of abdominal CT and discuss DD.

DDx:
Pancreatic pseudocyst (collection of fluid in lesser sac between stomach, liver, colon, pancreas & spleen)
Pancreatic cancer
Irritable Bowel Syndrome (can have depression)
Hypothyroidism (weather preference)

Hx:
PAIN Q – LOTS RADIO
Association: nausea, vomiting, diarrhoea, constipation, fever, jaundice
Any relation to food? Do you feel worse after spicy food?
Do you feel better after passing stool or wind? -- usually feel better
Any complications after your surgery 2 years ago
Depression Qs: How’s your mood recently? Appetite and sleep
Cancer Qs: Any recent tiredness; Weight loss; Any lumps somewhere
Thyroid Qs
Menopause Qs
General health: DM, Hypertension, Ischaemic Heart Disease
SADMA
Are you on stable relationship?
What’s your occupation?
FHx of any chronic condition and cancer

O/E:
- Any distension or visible mass ----distension (+), there is a mass
- Mass :Size, tender or not, mobile or not, pusatile or not
- Organomegaly
- Ascites
PR: normal

Ix: CT scan Pancreatic pseudocyst (Based on Hx & presentation, it’s most likely pseudocyst; If
necrosis, the symptoms will be severe. I’m looking for a cyst, probably it’s here, to confirm I will
discuss with the radiologist)

Mx:
Rules of 6: 6 weeks; 6 cm
If cyst < 6 cm  small cyst – most likely will disappear spontaneously
If cyst > 6 cm  symptomatic  drainage
< 6 weeks  present with immature pseudocyst with fragile wall, may disappear by itself
> 6 weeks  wall of the pseudocyst will have fibrotic tissue

Follow-up with US

Refer to surgeon for surgical assessment & opinion


Because the cyst is large, causing symptoms, and chronic, it needs drainage done laparoscopically
There will be connection between the cyst & the stomach
Fluid will disappear to the stomach gradually and cyst will close

! 29!
81
GASTROINTESTINAL TRACT

Liver Metastasis
Scenario 1: A patient has some symptoms and you ordered CT abdomen last time. Now, she
comes for the result. The patient is anemic (microcytic hypochromic) and has back pain and
lower limbs swelling. Pt is living in a farm for many years, has contact with dogs and sheep
Task: explain CT, further investigation & management

Scenarion2: 30 yo female c/o unwell for 1 month, O/E: liver 5cm below the costal margin.

The CT scan showed an enlarged liver and it has several hypodense focal lesions in both
lobes of the liver. The margin of this lesions are well defined. I’m sorry to tell you this is
what is called metastasis. This is a sort of Ca that results from Ca in other parts of your body.
The cells of that organ or tissue are out of control so they start to spread and in your case they
spread to your liver.

Are you alright?


Do you want me to continue my explanation? (Offer her tissue)
Do you want any member of your family to come here with you?
Do you want me to postpone this session?

Our aim now is to work out your liver problem and also to find out where is the primary
lesion (the first organ who give this cancer GIT: stomach and colon, breast, bronchus) I
will refer you to an oncologist.
He will take CT guided biopsy from your liver.
We would also do CXR, pelvic and abdominal CT, gastro+colonoscopy, mammography, and
we will do LFT and tumor markers as well.
Tumor markers are specific proteins that we check on these specific proteins for 2 reasons: to
check your response to treatment, and prognosis of your condition.
We have 3 types of tumour markers:
1. Carcinoembryonic Antigen – CEA – this is related to colorectal cancer
2. Ca 19.9 – related to pancreatic cancer and GIT malignancy as well
3. IF this patient is a man check PSA

I’ll refer you to the specialist for any possible treatment and pain management team
(palliative care)
There are a lot of things to do
Multidisciplinary approach
Palliative care – explain more
A team who helps patient with terminal illness to enjoy life as much as illness will let them.
It’s for patient, family, friends with terminal illness.
Palliative care can be given at home, nursing home, hospice care centre.

Treatment:
We will start you with chemotherapy - 5a Fluorouracil
Radiation therapy as well

It has 2 aims: It will improve her condition and a palliative care for her.
One of the options as well for treatment is: ligation of hepatic artery. It would help the
patient to reduce the size of the tumour.

! 30!
GASTROINTESTINAL TRACT

If the lesion is only in one lobe of the liver (esp right side), we can resect it.
If the lesion everywhere, we cannot resect it

Am I dying?
There is always a hope with the new medication

A CT scan of the upper abdomen showing multiple metastasis (cancer that has spread) in the
liver of a patient with carcinoma of the large bowel. Note the dark areas in the liver (left side
and center of picture.

! 31!
82
GASTROINTESTINAL TRACT

Peptic Ulcer (AMC 16)


A 48 year-old businessman came to your GP clinic with abdominal pain.

70% Duodenal ulcer caused by Helicobacter pylori

Hx:
PAIN Q – LOTS RADIO
Any particular time do you feel the pain?---2 am in the morning (pain in empty stomach)
Any relation with food?
Any heartburn, something travel up towards the throat, nausea and vomiting?
Bowel habits
Any blood in the stool, any black bowel motion?
Any similar episode before, how many times?
Do you have any problem after taking fatty food? (pancreatitis, cholecystitis)
Any difficulty of swallowing? (oesophageal problems)
Any dizziness, SOB
Weight loss
PHx of hepatitis and GORD
Family history of similar condition or cancer
Lifestyle: diet, stress
PMHx---Joint pain and is taking Neurofen for a long time
SADMA---smoking, alcohol and NSAIDs are risk factors

O/E:
GA: jaundice, anemia
VS: BP, T
Lymph nodes
Focus Abdomen
Inspection
- Any mass
- Distension
- If mass, does it moves with respiration or not
Palpation/Auscultation
- Where do you feel the pain  palpate the other side
- Check all, rebound tenderness, guarding
- Peritonitis  bowel sound (-), rebound tenderness (+)
- If there is mass  check site, size, move with breathing
Take permission to do PR, sometimes can find blood
Urine dipstick

Explanation:
From history & examination, most probably you have ulcer in your stomach or first part of
your small bowel, we call it peptic ulcer. To confirm I will order upper gastrointestinal
endoscopy. There is a mucosal lining in your stomach and duodenum that is protected by a
mucous layer that works as a defence mechanism. This layer protects the stomach from ulcer.
But you’re taking NSAID for a long time and also there are some bacteria called Helicobacter
pylori. Mainly these two things that break the integrity of this layer and more susceptible to
the bacteria and causes ulcer. H. pylori lives in the stomach and releases urea.

! 32!
GASTROINTESTINAL TRACT

I’d like to refer you to a gastroenterologist who will do endoscopy and biopsy. They will put
a tube with a mini camera through your mouth to your stomach to look at the lining. They
will take some samples from the (fundus) to look for H. Pylori. If they see there is an ulcer,
they will take a small tissue from there as well. If H. pylori is positive, we’ll give you triple
therapy that contains:
- Proton Pump Inhibitor which will reduce acid secretion
- 2 antibiotics: Clarithromycin or Amoxycyllin plus Metronidazole
Antibiotics are given for 1-2 weeks and proton pump inhibitor 4-8 weeks.

Urea breath test is done at 6 weeks to see if the H. pylori is eradicated or not.
Repeat endoscope for gastric ulcer.

Stop Neurofen and change to another painkiller (paracetamol or COX 2 inhibitor)

Lifestyle modification advice


- Reduce your weight
- Stop smoking
- Reduce your alcohol intake
- Maintain your diet regularly
- Relaxation therapy

It’s a common condition, not dangerous, easily treated, usually the ulcer will heal in 1-2
weeks, and success rate is very good – over 90%. With triple therapy, the chance of relapse is
low.

Ix: Endoscopy and FBE

Complication:
- Bleeding
- Perforation
- Gastric Outlet Obstruction
- Malignancy (only related to gastric ulcer, not duodenal ulcer)
- Iron deficiency anemia

If there is complication, patient may need surgery.


- Perforation
- Gastric outlet obstruction
- Severe persistent bleeding heater probe or injection of adrenaline

Minor bleeding stop automatically.

! 33!
83
GASTROINTESTINAL TRACT

Perforated Peptic Ulcer


A middle-age female presented with abdominal pain for 3 hours in ED. She had a back pain
last week & took NSAID (Neurofen 400 mg tds) to control.

Task: abdominal exam, ask investigation, diagnosis & management.

Critical errors:
Not asking if patient is haemodynamically stable resuscitate & give morphine
Not recognizing the free air under the diapraghm
Not admit
Not suggesting surgery

O/E:
GA: Pale, sweaty, clammy, distress by pain
BP 100/60, P 104, RR 20, T 36.7, a bit tachypnea, SaO2 normal IV canula
Abdominal exam:
Inspection: check abdomen & pattern of breathing
- Distension or
- Shelving abdomen
- Pattern of breathing : reduced abdominal movement with breathing  shallow breathing

Palpation: tenderness, rigidity: very painful  don’t continue


Percussion for any signs of peritonitis---very severe pain, generalised peritonitis

Auscultation: it’s acute abdomen, I’d like to hear the bowel sound no bowel sound

PR: check if any tenderness of Pouch of Douglas  tenderness


might see blood (melena)

At this stage, in my opinion my patient has acute abdomen most likely due to perforated
peptic ulcer based on short history (3 days of pain) & NSAID use. I want to do some Ix:

Erect abdominal X-ray and CXR, I’ll especially look for free gas intraperitoneal – abdominal
X-ray showed free air under the diapraghm, below the liver.

! 34!
GASTROINTESTINAL TRACT

- Admit
- 2 large IV cannula & give fluid immediately
- Put nasogastric tube and keep my patient nil by mouth
- Give morphine for pain control
- FBE, Urine, U & E, ECG, Coagulation profile
- It’s a surgical emergency, I’ll call surgical registrar to arrange for the theatre

What will the surgeon do?


- may be done by keyhole or open surgery
- You have a hole in your bowel, we need to close with an open surgery or 3 small cuts
with keyhole, the surgeon will talk in details which is better for you
- Surgeon will use (omentum) to fix this hole

Can I pick up my children, can I go home & come back in 2 hours later?
No, it’s a surgical emergency, we need to do this operation as soon as possible because of the
risk of intra-abdominal infection, infection will spread to your abdomen and blood
We’ll give you antibiotics to prevent the spread

How many days do I have to stay in hospital?


When your bowel function return to normal & can tolerate oral fluid, you’ll be free to go
home. Generally 3-7 days.

! 35!
84
GASTROINTESTINAL TRACT

Mesenteric Ischemia
A 65 year-old lady has been having pain for 3 hours in the morning and bloody diarrhoea,
nausea. She has a history of appendicitis and atrial fibrillation.

Task: take history, examination, investigation, diagnosis.

Hx: (Stable or not)


HOPC
- Where is the pain?
- Is it started gradually or suddenly?
- Is it becoming more severe?---yes
- How severe is it?
- Does it go anywhere else?
- Did you try anything to relieve your pain?
- Is the pain continuous or on and off?
- When did the diarrhoea start
- How many time
- What’s the nature of the diarrhoea
- What is the colour
- Is that smelly
- How many times
- Any vomiting
Are you on any medication for AF, what is the medication
Have you been diagnosed with any serious condition
Any similar condition in the past
Any similar condition in the family
What’s your blood cholesterol level
Any cholesterol medication
Any history of hypertension
Do you have any unexplained chest pain, any surgical history in the abdomen
Smoking
What’s your lifestyle like, exercise, diet
Travel, trauma
Ca

O/E:
GA: anxious, tense, sever abdominal pain, pale
VS: High blood pressure, tachycardia, T normal
Abdomen
- localised tenderness & rigidity in the centre of abdomen
- rebound tenderness in the central of abdomen (over infarct loop of bowel)
- absent bowel sound

From Hx and PE, your condition is called acute abdomen, there are several possible causes.
Ix:
- FBE
- ESR, CRP
- Abdominal X-ray, thumb printing sign (thickening of bowel wall), bowel is full of gas
- CT scan

! 36!
GASTROINTESTINAL TRACT

Explanation:
From Ix, your condition is caused by mesenteric ischaemia, the blood supply to your bowel
has been cut off due to blockage. B/O that, part of the bowel is not functioning.
We will admit you to the hospital
- IV cannula, give fluid, nil by mouth
- Give antibiotics intravenous, analgesics
- The surgeon will assess you and may do more Ix like CT and mesenteric arteriography
make definitive diagnosis
- You’ll need urgent operation, we’ll cut the affected bowel and re-anastomosis the healthy
segment, the earlier we do the operation, the better the outcome

Causes
- Occlusive : thrombus/embolism, arteriosclerosis
- Non-occlusive: shock, heart failure, sepsis

Where did I get this condition?


You have AF, your heart is not working properly. The blood clot will form and travel to the
peripheral circulation and cause occlusion of the small blood vessel supplying the bowel.

Ischaemic colitis  gangrene


Transient ischaemic colitis
Stricture of bowel due to fibrosis, cut off blood supply

! 37!
85
GASTROINTESTINAL TRACT

Coeliac Disease
A 32 year-old man presented with persistent diarrhoea for 6 months. He had a past history of
bronchitis for a few months. He has been taking Imodium and one of the other drugs
irregularly for 4 months.

Task: take history, examine the patient and explain to him what he has and manage his case.

HOPC: Bob has not been feeling well for the last 6 months. He noticed increasing tiredness,
generalized weakness and lassitude. His appetite is rather poor and recently he has developed
a sore tongue (glossitis). There has been mild and intermittent diarrhea with sometimes
bulky, pale, offensive, frothy, greasy stools which he found hard to flush down the toilet
(Steatorrhea) and a lot of flatulence.
Altogether he probably lost 4-5 kg over 6 months and now weighs 70 kg with a BMI of 22.
PHx.: unremarkable
FHx.: He is not sure but thinks his mother had some gastro-intestinal problems but she died
relatively young in a car crash.
SHx.: married school teacher, 3 children, no problems, no stress, non smoker, non drinker,
NKA, no medication.
O/E: all normal

Diarrhoea  watery, profuse, a little bit blood, difficult to flush, 4-6 times a day.
Feverish but not high

History of abdominal discomfort---yes


Anal effect  discomfort, redness, inflammation because of repeated diarrhoea
Vomiting ---no
Family history  anyone in your family has a special diet (gluten free)
Any joint pain---a little bit
Do you have any dizziness, loss of consciousness  feel a little bit tired
Any special food which makes your diarrhoea worse---bloating
Any history of peptic ulcer
Appetite---anorexia & weight loss, tiredness, low grade fever, tummy pain
---yes weight loss
Any medication---just anti diarrhoea but irregular
Seek help or not?
Any other investigation in the past---stool occult blood (-), colonoscopy normal
(exclude malignancy)
History of travel

O/E:
GA: looks tired, little bit bothered from his diarrhoea, BMI: decreased
VS: normal, low grade fever 37.5
Cardiovascular exam: normal
Abdomen: no organomegaly, gargling sound in the tummy, some flatulence and ballotment
Anal area: inspection  redness around the area

Explanation:
What is celiac disease: It’s a hereditary disease of small intestine caused by sensitivity of
some of the protein called gluten. A kind of protein found in the food, normally our intestine

! 38!
GASTROINTESTINAL TRACT

is fluffy. However in this condition, it’s flat, cannot digest and absorb gluten. Diarrhoea
passes all the food. Cause unknown.

Ix:
Stool exam
FBC (iron-deficiency anemia in children and folate-deficiency anemia in adults)
Coeliac screening: antigliadin antibody and antiendomisial antibody.
Duodenal biopsy definitive Dx

Diagnosis is by small-bowel biopsies showing characteristic though not specific pathologic


changes of villous atrophy that resolve with a strict gluten-free diet.

The classic triad is:


1. Diarrhea
2. Weight loss
3. Iron/folate deficiency anemia

Mx:
• gluten-free diet (avoiding foods containing wheat, rye, oat or barley). Gluten is so widely
used (eg, in commercial soups, sauces, ice creams, hot dogs) that a pt needs a detailed list
of foods to avoid.
• consult a dietitian and join a coeliac support group.
• Supplementary vitamins, minerals, and haematinics (Iron, Cobalt Co, Zinc Zn, Vit-B12,
Folic acid and Erythropoietin) may be given, depending on the deficiencies.
• Give pneumococcal vaccination
• Small-bowel biopsy should be repeated after 3 to 4 mo of a gluten-free diet

Mr Smith we have certain type of food you have to avoid and by avoiding this food, you will
be free from your symptoms and your diarrhoea as well. Try to buy your food from the
special section in the supermarket (gluten free). Also you may have anemia, we have to check
your blood picture to see if you have anemia. If there is iron deficiency, I will give you iron
supplement. Try to have complex carbohydrate, high protein diet and low fat. I will refer you
to gastroenterologist to do more investigation for you and he will do biopsy of the small
intestine lining to confirm the diagnosis. Also will refer you to dietician, he will help you to
choose a healthy food suitable for you.

There is a support group, celiac disease support group. You can contact them, they will give
you good support.

! 39!
86
GASTROINTESTINAL TRACT

Irritable Bowel Syndrome


Scenario 1: A 52 yo woman presents with a long Hx of constipation. She was thoroughly
investigated for the past 3 months. Gastroscopy, colonoscopy, and FBE were all normal. Now
she’s c/o bloating, wind, tenderness in the tummy and pain in the Left Illiac Fossa.

Task: ask relevant history, explain diagnosis & management.

Hx:
Could you please tell me more about your symptoms:
Bloating, wind, pain
- How long
- Present all the time or any particular time
LIF pain, please tell me more about this pain Pain Qs
Constipation
- How long? 3 months
- How often do you open your bowels?
- Have you noticed any recent changes in your bowel habits
- Have you noticed any recent changes in the frequency of stool & form of stool
- Have you noticed any blood or mucus with stool
- Do you need to strain when you pass stools
- Have you noticed any bleeding from down below or any blood on your toilet paper (to
differentiate from haemorrhoids)
- Have you noticed any recent changes in your appetite
- Have you noticed any recent weight gain or weight loss
How is your general health so far?
How is your mood (rule out depression and stress) recently? Any stresses at work or at home
Is there any condition that is affecting your life?
How is the relationship with your partner?
Menstrual history (52 year-old  close to menopause)
Any past history or family history of bowel disease or carcinoma colon
SADMA

Explanation:
Irritable Bowel Syndrome  investigations normal.
Not the same as Inflammatory Bowel Disease (ulcerative colitis, Crohn’s disease).

From the history and report of all investigations you’re most likely to be suffering from a
condition called Irritable Bowel Syndrome.
Irritable Bowel is the bowel that does not work smoothly & causes abdominal problems such
as colicky pain and disturbed bowel actions.
There is no known cause for this but emotional stress plays a key role and usually affects the
perfectionist.
It is a very common condition and people simply easily learn to live with this but there are
some aggravating factors such as infection, food, irritation, food allergy, lack of fibre,
overuse of laxatives, sometimes painkillers and antibiotics, smoking.

Mrs Smith since our mind and body are connected with each other, so when our mind is upset
or under stress, it can affect the effectivity of bowel & becomes more irritable. Please do not
worry, it is a harmless condition.

! 40!
GASTROINTESTINAL TRACT

Mx:
1. Avoid any stress
2. Develop a more relaxed lifestyle
3. Be less perfectionist
4. Do not bottled-up things (speak out)
5. Avoid excess of coffee, alcohol, smoking, fizzy drinks
6. Drink plenty of water
7. Eat fibery food, green leafy vegetables
8. Try to maintain a diary & write down the food that cause your symptoms

I can give you some anti spasmodics to relieve your symptoms.


If very stressful, some anti-depressants can be given for a short period.
If these medications fail, I can refer you to a psychologist.

Scenario 2: A young woman presented in a GP clinic, c/o diarrhoea, no blood, no fever,


always complaining of abdominal pain that is vague in nature. She is thoroughly investigated
and all investigations are normal. She is diagnosed with Irritable Bowel Syndrome.

Task: take a psycho-social Hx, manage the case, explain condition to the patient.

Hx: Address confidentiality


How is your general health?
Is this condition affecting your life?
Depression and anxiety Qs
- How is your mood?
- How is your sleep?
- How is your appetite?
- Are you still interested in the things in which you used to be?
- Do you have any stresses at work or at home?
- How is your relationship with your family?
- Are you sexually active?
- Are you in a stable relationship?
Past history of similar conditions
Family history of similar conditions
SADMA
HEADDSS

! 41!
87
GASTROINTESTINAL TRACT

Irritable Bowel Syndrome


You’re a GP. A 45 year-old female presented with a sudden changes of bowel habits.

Task: Hx, discuss DDx, manage the patient’s condition, answer the patient’s questions.

DDx:
Irritable Bowel Syndrome
Coeliac Disease
Inflammatory bowel disease: Crohn’s disease & Ulcerative colitis
Lactose intolerance
Thyroid problems
Colorectal cancer

Hx:
What kind of changes – I have diarrhoea & sometimes constipation
How often the diarrhoea?
Any mucus or blood, or hard to flush?
Big in volume?
Timing through day and night
Any fecal incontinence?
Any relationship of bowel action to eating?
Any pain?---cramping pain
How severe
Any relieving or aggravating factors---feel better after passing stool or winds (typical
symptoms of irritable bowel syndrome), more when eat spicy food
Associated symptoms: pain, fever, nausea, vomiting, weight loss, lethargy
Anemia – have you noticed any changes in your face color---my friend told me I look pale
Current medication, any antibiotics or laxative, codeine-contained pain killers
Any thyroid disease
Change of appetite or wt
PMHx Any chronic diseases in the past ---no
FHx Any bowels disease or cancer in your family? Anybody in the family on special diet?
Social historyStressors; Whom do you live with---married with 2 kids
Occupation---extremely stressful life, executive
SADMA

Explanation:
Your condition is most likely IBS because you have a stressful life, it affects your bowel
action. IBS is very common in your age group & it’s a functional problem, not organic.
Because you have FHx of bowel cancer, I’d like you to do some stool tests and after this test
we may need to do some scope to look at your bowels from the back passage (colonoscopy)
and check if there is any abnormality in your bowel.

If pt is concern  do FOBT, if blood test come back positive we need to do colonoscopy.

Red flag: you notice any blood in your stool or wt lossScreening program

Critical error Not demonstrated awareness of excluding bowel cancer

! 42!
88
GASTROINTESTINAL TRACT

Inflammatory Bowel Disease


A 25 year-old computer analyst complaining for diarrhoea for 3 weeks. This is the fourth
episode in the past 3 years. In the past he was treated with antibiotics but did not improve.

Another scenario of the case above:


o A patient came with a photograph of colonoscopy, he’s concerned about the management.
o A man in 30’s complaining of diarrhoea for a couple of days. It is bloody.

Task: history, PE, management

Hx:
- How long have you had the diarrhoea? -- for a couple of days
- How often? ---3-4 times a day
- What is the nature of your stool, soft or hard?---soft
- Have you noticed any blood or mucus in the stool? ---yes there is blood
(more blood: ulcerative; more mucus: Crohn’s)
- Have you noticed blood on the top of stool? (haemorrhoid)
- Is it smelly, explosive?(Giardiasis)
- Is it greasy and hard to flush? (Coeliac)
- Have you noticed any incomplete sense of evacuation? (colon Ca)
- Do you soil your underwear? (fistula on IBD)
- Have you noticed any tummy pain?
- Any fever, nausea, vomiting?
- Is it related to any particular food? (Coeliac)
- Is anyone in your family has the same symptoms? (viral gastroenteritis)
- Have you travelled overseas recently?
- Any palpitation, tremor, sweating? (hypothyroidism)
- Any history of bowel cancer, Coeliac disease?
- What is your occupation? ----Stress…..irritable bowel syndrome
- Have you noticed any joint pain, eye pain, mouth ulcers, rashes (complication of IBD)
- SADMA

O/E:
GA: any pallor, signs of dehydration, skin turgor, mouth ulcers, angular stomatitis, skin rash
VS: T, P, BP
I’d like to do the cardiovascular, respiratory and abdominal examination
Inspection: The perianal skin for any fistula, fissure, skin tag and haemorrhoid
Per rectal examination: Any mass, mucus, blood, prostate enlargement

Ix:
FBE, ESR, CRP, TFT, Stool microscopy & culture, Colonoscopy to confirm the diagnosis
Colonoscopy: look for any growth, polyp, ulcer, abscess, erythema, oedema, swelling,
granuloma.
How the colon looks on the colonoscopy
Ulcerative colitis  Ulcer
1.Involves only lining, confined to mucosa
2.No skip lesions
3.Ulcers found

! 43!
GASTROINTESTINAL TRACT

Crohn’s  Granulomas
1.Affect whole layers of bowel
2.Involves large/small intestine
3.Has skipped lesions
4.Granulomas are found

CRITICAL ERROR: Failure to do colonoscopy

Explanation:
From Hx, exam & Ix, most likely you’re having a condition called Inflammatory Bowel
Disease. There are 2 groups of disease, ulcerative & Crohn’s. It’s usually runs in families
The exact cause of this condition is unknown.
It’s more common in western society.
And can be associated with low fibre diet.
It’s a lifelong condition.
We can control the disease and stop the progression.
I’m going to refer you to a gastroenterologist.

Medications will be given by the specialist:


1. Sulphasalazine
2. Prednisolone
3. Azathioprin

If medication failed, then a surgery adviced.


Apart from this medication, please take a well-balanced diet, rest and follow-up regularly
with me and the specialist.

Complications:
Ulcerative Colitis
1.Toxic megacolon
2.Acute flare up
3.Bowel obstruction
4.Increase risk of carcinoma

Crohn’s
1.Acute flare up
2.Acute colonic dilatation
3.Fissure/fistula abscess
4.Risk of malignancy low

! 44!
89
GASTROINTESTINAL TRACT

Splenic Abscess
You’re an HMO. A 50 yo male came to you complaining of L upper quadrant pain and fever.

Task: take history, examination, investigation and management.

TRIAD
- Fever (90%)
- Left upper quadrant pain (35%)
- Splenomegaly (50%)

Pain started 3 days ago, younger son kicked into his abdomen when playing. Pain 5 out of 10.
Constant pain, no radiation, no history of DM, taking omeprazole 20 mg daily, smoke 10
cigarettes per day. TT

O/E:
GA: no pallor, no acute distress
VS: T 39.9, PR 83, BP 100/80, RR 18
Systemic exam normal except for abdomen
Abdomen:
- tenderness in left upper quadrant
- slight guarding
- no rebound tenderness
- no inguinal mass
- genitalia normal

Ix:
- FBE – WCC 20 000
- U & E normal
- BSL normal
- CRP increase
- USG abdomen
- CT scan – multiple area of low density in left upper quadrant

EXPLANATION
It looks like splenic abscess with gas forming organism. It’s common in people who drinks a
lot of alcohol, DM, splenic infarction secondary to leukemia, sickle cell anemia and trauma.

DDx
- Pyelonephritis
- Left lower lobe pneumonia
- Subdiaphragmatic abscess

You’re most likely suffering from splenic abscess. It’s like abscess in anywhere in the body.
It means there is a collection of pus. It could be due to secondary to trauma as a result of
emboli in the blood vessels affecting blood supply to spleen. It’s not very common. It’s risky,
it can cause septicaemia, peritonitis, splenic rupture.

! 45!
GASTROINTESTINAL TRACT

Management
- Admit to hospital. Refer to surgeon. Start IV fluid, IV antibiotics until condition
improved. Give analgesics.
- Surgical drainage may be required through USG or CT scan guided. Specialist will
decide.
- After acute attack, need to take oral antibiotics for 2-3 weeks. After discharge, follow up
in 2-3 weeks.

Antibiotics: Gentamycin + Metronidazole + Ceftriaxone

! 46!
90
HAEMATOLOGY

Thalassemia (AMC 12)


A 24 yo female presented with Hb 10, HbA2 10% (normal < 2%), MCV 70, Iron 4
g/L, FHx of anaemia. She’s getting married next month.

Task: explain the result and management


(She’s going to marry next month. From Mediteranean, her partner is Greek)

You’ve got some abnormal result in your blood test.


You have mild anaemia.
Your condition is called thalassemia minor.

In our blood, we have different types of blood cells.


You have a problem in your red blood cell
Inside the red blood cell there is Haemoglobin  Heme + globin
Globin has 2 chains, the alfa chain and B chain

You have a problem with the B chain.


It is a genetic condition, inherited, usually runs in family
Common especially in Mediteranean region. (She’s Mediteranean, partner is Greek)

Genetic disorder means gene disorder.


You only have half of your gene affected.
You received it from your father or mother.
Most likely you will have no problem with it, and will not require any treatment.
But folic acid 1 mg/day may be helpful.

When you plan to get marry, please come with your partner to check his gene for
thalassemia.
If he doesn’t have any abnormal gene, there will be no great risk for your children,
they can be carrier like yourself.
But if your partner has half of the gene affected, it carries risk for your children. Some
of them may have thalassemia major.
If the whole gene is affected  thalassemia major 25%  severe anemia, decrease
life expectancy.
Risk carrier = 50% thalassemia minor
25% chances of normal

I’ll give you written material


I’ll refer you to a genetic clinic, for all family
If you have brother/sister  refer to genetic clinic
Refer to haematologist

Do I need to do anything for my anemia?


My friend is taking iron tablets, do I need to take iron tablets too?
You don’t need to take iron tablets. Your iron is normal.

1
91
HAEMATOLOGY

Haemophilia
GP setting. A couple who have an 8 months old son with hemophilia want to have
another child.

Task: counselling.

Hx:
It’s very good that you have come. I know you would like to know more about
haemophilia which is a genetic condition.
It's a clotting disorder. The main symptom is easy bleeding.
We have genes and we have chromosomes.
We have specific type of chromosomes.
In female = XX and male = XY
This hemophilia genes are linked to X gene.
Because both of you are healthy, it means Bob does not have any abnormal gene 
XY, but Carol has one normal and one abnormal gene  XX•
XX• XY

XX XY X• X X• Y

X• X  she will be healthy but she will carry the X• gene


X• Y  boy will have effect

The chances of having the next child with this problem will be 25%.
All your girls will be normal.
We have some clotting factors in our blood.
Deficiency factor VIII = Haemophilia A
Deficiency factor IX = Haemophilia B

It is a bit upsetting information. But it is usually a mild condition.


I will refer you to a haemophilia clinic. These clinics are all over Australia.
With good care your son will have a good life.
We usually recommend avoiding contact sports.
In case if bleeding occur or bruises happen please see me or go to the hospital

Do you have any question about your son?

Now let's talk about your future pregnancy.


The risk of having an affected child is 25%. So it is a high risk pregnancy.
You need more close monitoring and antenatal check-up.
We can detect haemophilia before labour.
We can do :
1. CVS at week 11 (option to continue pregnancy or not)
2. Amniocentesis: week 15 or week 16
We can confirm or exclude haemophilia.
Both of these procedures carry risk of miscarriage. But it will give us 100% diagnosis.
Here are some written materials for you.
If you have any more questions or concerns please come back.

2
92
HAEMATOLOGY

Hodgkin Lymphoma
A 45-year old man came to your GP clinic complaining of a lymph node enlargement
in his neck. He was referred to an oncologist who did a FNAC and confirmed
Hodgkin’s lymphoma. The nodule was removed and further investigations showed no
involvement of other lymph node and spread to other organ. He is going to have
chemotherapy and radiotherapy.

Task: explain to patient what is lymphoma. Explain about the side effect of
chemotherapy and radiotherapy, answer patient’s questions.

Explanation:
What do you know about lymphoma?
Lymphoma is a cancer of the lymphatic system.
The 5th most common cancer in Australia
Lymphatic system is a part of the immune system, consists of lymph nodes and
lymphatic vessels. Spleen and bone marrow are considered as part of the lymphatic
system. In this cancer, the lymphocytes become abnormal and divide, grow out of
control and lead to develop a cancerous tumour in the lymph node.
It can develop anywhere else outside the lymphatic system.

There are 2 types of lymphoma:


1. Hodgkin’s
- Affects B lymphocytes
- Affects younger age group
- Increase the risk by EBV (glandular fever)
- 5-year survival rate  80%
2. Non Hodgkin
- Affects B cells  head, neck, throat, abdomen
- T cells  chest
- More common than Hodgkin’s lymphoma
- Risk increase with age
- It has extranodal involvement (organ involvement)
- 5-year survival rate  55%

Symptoms of both are similar


- Fever
- Night sweating
- Decrease appetite
- Weight loss
- Generalized itchiness
- Persistent tiredness
- Lymph node enlargement

Ix: Lymph node excision and biopsy.

You have a good prognosis because you don’t have other lymph node and organ
involvement.
Because dormant/silent cells can activate any time, so we need to prevent flare up
early  chemotherapy and radiotherapy.

3
HAEMATOLOGY

Multidisciplinary team include GP, oncologist, nurse and social worker.

Chemotherapy is a systemic therapy given by IV route for 4-6 courses. Not


monotherapy, there is > 1 drug.
Each course for 3 weeks (3 weeks apart).
Semi permanent device in the chest or arm.
Side effects:
- Pancytopenia
- Immune suppression
- Infection  manifest by sore throat in early stage
- Nausea, vomiting, diarrhoea
- Hair follicle toxicity  hair loss
- Infertility ask if family is complete or not. If not, sperm or egg can be put in a
bank before chemotherapy.

Radiotherapy is a local therapy to target area or organ, given as short shoots for 5
days a week over several weeks.
Side effects
- Tiredness
- Fatigue
- Skin reaction
- Dysphagia for thoracic radiation
- Diarrhea for abdominal and pelvic radiation
- Bone marrow suppression

Biological therapy is a sort of immunotherapy which depends on the body’s natural


immunity.

Prognosis:
Depends on the stage of lymphoma.
Ranging from stage Ia to IVb.
During the course of therapy, patient needs to be off work.
After finishing the treatment, it depends on the condition of the patient.
If patient has no one to pick him up at the hospital, arrange a social worker.

Will I get pain?


You’re in the early stage and in a good condition.
You’re going to have treatment early (don’t mention pain, just assure).
Re-assure about the good prognosis.

4
93
HAEMATOLOGY

Autologous Blood Transfusion (AMC 17)


A middle aged man is booked for a total hip replacement. The surgeon has arranged
an elective surgery. The patient is concern for a blood transfusion if it’s required.
He’d like to find out if he can use his own blood.

Task: explain the principle & indication for pre-op blood collection, answer his Qs.

I understand that you’ll be having a total hip replacement


Do you have any concern about that---he’s concerned about the blood transfusion.
When are you booked for the operation? ---4-5 weeks later
Do you want to know about the autologous blood transfusion?
We take 450 ml of blood (1 unit) every week and we can take 2-5 units depending on
the need (2L over 2-5 weeks)
The blood is stored under the patient’s name at 4 degrees Celsius in the refrigerator
for up to 5 weeks
If not used within 5 weeks, then it is discarded
It cannot be used for others

When can it be done?


- If the Hb >110 (for female >135)
- The date of surgery is fixed
- It’s sure that you’ll need the blood
- There are no medical contraindication for donation

If the patient requires more blood:


- Volume replacement is done (plasma expander)
- Intra operative blood transfusion (blood salvage)  collection of blood from the
bleeding site, filter the blood, add anticoagulant, retransfuse it.
- The site should be free from bacterial intestinal content and tumor

The equipment for this is expensive


It’s usually done in major trauma or vascular surgery
The blood is not screened for autologous transfusion
The volume is replaced within hours but the cells take weeks to replace haemoglobin

Benefits of autologous transfusion


- Minimizes risk of allergies
- Minimizes risk of infection
- There is no incompatibility risk
- Causes blood thinning (normal volume but Hb less  hemodilution)
- Decreases the risk of pulmonary embolism, DVT, stroke

Risk of homologous transfusion


- Mishandling or wrong labelling  can lead to blood transfusion reaction

Normal transfusion – increase risk of:


- HIV in window period
- Prion disease
- Hepatitis G

5
HAEMATOLOGY

Different types of donor blood transfusion:


- Autologous
- Heterologous
- Intra operative autotransfusion
- Recombinant erythropoietin – an injection to stimulate the body cells to produce
more RBC

Whole blood
Packed red cell
Platelet
Plasma

6
94
HAEMATOLOGY

Herpes Zoster (Shingles)


A middle-aged man working in a childcare came to you with a painful rash and the
picture was given.

Task: take history, exam finding and manage the case.

I read that you have a painful rash, can you tell me more about your rash
It started 3-4 days ago.
Regional lymphadenopathy
Unexplained symptoms to exclude underlying malignancy (lymphoma) like tiredness,
unusual blood film or blood result, recently any lumps or bumps
Any family history of any cancer

Shingles occurred by a contact with someone who has a chickenpox or herpes zoster
but most commonly with chickenpox. The virus stays in the ganglion and become
dormant. When this person has some stress factor or long illness, the virus can flare
up and appear as herpes zoster. Or if in contact with chicken pox, immune not good,
the rash appear. Reactivation:
1. Contact
2. Reactivation

Chicken pox during childhood, the virus became dormant. Stress factor  virus
appear.

Mx:
Avoid contact
Take simple pain killer and soothing agent (lignocaine cream) in addition to an
antiviral cream.
Be aware when you clean your lesion, clean your hands carefully, otherwise it will
spread to eyes and ears (facial nerve)  Ramsay-Hunt syndrome.
Antiviral tablet
Since the rash to appear, there’s 3 days to give Acyclovir (works within the first 72
hours when rash start to appear). Mode of action  prevent the patient to have post
herpetic neuralgia  give amytryptilline & local anaesthetic, painkiller.

Most common complication:


Meningoencephalitis
Post herpetic neuralgia

Herpes Zoster patient: Need to find out why patient became immunocompromised…

1. Leukemia- ask fever, night sweats, wt loss


2. HIV- ask abt sexuality
3. Exposure- occupation, family
4. any other malignancy
5. any meds that can cause immunosuppression

7
95
HAEMATOLOGY

CLL With Herpes Zoster


A 60 yo female c/o painful rash on the right side of abdomen and fatigue for 6 weeks.

Task: take history, physical examination, diagnosis, and management.

Hx:
I know you have a painful rash
When did you notice it---yesterday
Is that the first episode
Did you have chickenpox as a child, any contact
What is your occupation? ---childcare
Do you have fever, chills, night sweat, muscle, joint pain
Any weight loss, how many kilos?
Tell me about your fatigue
Is it getting worse
Any lump or bump over your body?
Menstrual history
Any vaginal bleeding, discharge  cancer
Have you had mammography, Pap smear? what was the result
Qs regarding other cancers
General health
Have you travelled recently?
SADMA
Any family history of cancer

O/E:
GA: pallor
VS: T, PR, BP, RR
All lymph nodes – cervical, axilla, inguinal
Chest, heart
Abdomen – liver and spleen enlarged
Rash – character, distribution

Explanation:
After physical examination, I found that you have 2 problems
First, the rash, this condition is called herpes zoster or shingles is common and caused
by virus.
In the childhood if you get the infection with chickenpox virus, it will remain in your
nervous system for a long time and symptoms will come back when our body is weak
due to some health problems, stressful situation
I’ll give you analgesia, paracetamol to control pain
Local Calamine lotion
Acyclovir if rash appears < 72 hours

Risk of post-herpetic neuralgia – pain still present after rash disappear (Tx:)
Meningoencephalitis – but rare

8
HAEMATOLOGY

Your second problem is your lethargic and lumps all over your body
I suspect you may have some growth in your lymph nodes.
I’ll urgently refer you to the surgeon to do biopsy of the lump
The surgeon will cut the skin, take a part of the lymph node and send for pathology
After that, you may need blood and urine tests
You may need to have CT scan of chest, abdomen depending on the results of the
lymph node sample.

9
96
INFECTIOUS DISEASES

Hepatitis A (AMC 147)


A 25 year-old male came to your GP practice complaining of malaise, nausea, fever which
has gradually worsen during the last 10 days. Yesterday he passed dark urine and pale stool.

Task: take further history, examination, diagnosis and management.

Symptoms of HepA: Fever, jaundice, abdominal pain. Usually pain at right hypochondrium
due to stretch of liver capsule and OK while take rest.

Hx:
When did it start? fever and loss of appetite 10 days ago, yellow skin for 1 week
Appetite, hydration status
Hepatitis Qs
Tattoo, piercing, IV drug, sharing needles, blood transfusion or donation, operation,
accidental needle prick injury, travel Hx, alcohol, sexual life, contact history of hepatitis.
Any previous history
Have you received any hepatitis immunization
If travel history (+), when did you travel, how long, did you go aloneWent to Thailand 2
wks ago with girlfriend.
Does partner have similar problem
Where did you eat?  outside
Did you stay at a good hotel? no
SADMA
What is your job (hospital, laboratory)

O/E:
GA: signs of liver failure  flapping tremor, altered consciousness, fetor hepaticus
Check hydration status
VS: T 38, patient has jaundice
Lymph nodes
Systemic exam, focus on abdomen

Ix:
- FBE
- LFT
- INR
- Hepatitis serology A, B, C  Hepatitis A Ig M (+)
- Epstein-Barr virus serology
- Urine dipstick  ketones (+) in severe dehydration

Explanation:
It seems that you are suffering from an infection in the liver due to hepatitis virus. It’s a
group of virus but in your case, it’s hepatitis A.
It’s self-limiting in nature and you’ll be better by the time
There is no specific treatment
Bed rest (medical certificate), ↑ fluid intake, avoid fatty food, take care of your hygiene
It’s communicable disease so you should wash hands before handling food and after toilet
For 10 days, it’s better to use your own utensils
Don’t take any medication especially Paracetamol, don’t drink alcohol

! 1!
INFECTIOUS DISEASES

I’ll review you after 2 weeks


If you have any concerns, please come and see me
It’s an infectious disease, so we’ll need to check your partner
I need to notify to DHS
If you have vomiting, will give vit B6

Critical errors:
- Give antibiotics
- Not telling patient to avoid alcohol

! 2!
97
INFECTIOUS DISEASES

Hepatitis C (AMC 58)


You’re in a GP clinic. A 26 year-old female goes for a blood donation & her blood result
showed that she had Hepatitis C. Now she comes to you to discuss her results.

Task: take history and advise her about subsequent management and likely prognosis.

In this case, the history is important to find the possible source of infection:
- Blood transfusion
- IV drug usage
- Unprotected sex

How are you feeling today since the last time I saw you.
Have you come alone or do you have anyone with you?
While discussing the result, do you want someone to be with you?

I’m sorry to tell you that unfortunately the test result showed you have Hepatitis C.

Have you ever had a blood transfusion before or have you donated blood (donating blood,
the source of infection may come from the nurse or the cannula)
Any tattoos or body piercing

Now, I’m going to ask you some personal and sensitive questions, is that OK with you?
Are you sexually active----yes
Are you in a stable relationship ----yes, I have a husband
Before that, have you ever had more than 1 partner or unprotected sex ----yes, 2 partners, one
for 3 years and the other for 2 years
Have you ever used intravenous drug
Did you use to share the needles

Asking for symptoms:


Any nausea, vomiting, how’s your appetite
Have you noticed any change in your skin, colour of your eyes, or urine
Any itchiness
Any bleeding or bruising (clotting factors  problem in the liver)

Past medical surgery


Dental surgery or any other kind or surgery
Medications
OCP
Family history of hepatitis
SAD Questions

! 3!
INFECTIOUS DISEASES

Management depends on the investigation


1. PCR (Polymerase Chain Reaction)
- to check if the virus is still in your body  if you have cleared the virus  it means
that you have a spontaneous recovery (occur in 15-50% of cases),
- 50-75 % of patient still have the virus

2. LFT, ALT  if normal, repeat every 2 months for 3 times (total 6 months)  even
though you have the virus but it’s not affecting your liver  carrier, no long term effect
on the liver
3. If LFT and ALT high  refer to gastroenterologist  who will treat you with Interferon,
or Ribavirin

4. 20% will cause cirrhosis  liver failure  transplant

5. 1-5% liver cancer

If your PCR is positive, there’s 5% chance that your child will have vertical transmission,
however you can still breastfeed (HIV patients cannot breastfeed their children).

You can get pregnant after 6 months from stopping Ribavirin.

Will my husband get this infection?  transmission during sex


It’s extremely low, unlikely (Hepatitis C is mostly transferred by blood)

How can I prevent the transmission?


You should not donate blood or organs, no sharing of needles, toothbrush, razors  if you
have a bleeding cover the cuts and injuries with a firm dressing.
Practice safe sex
Advise healthcare and dentist about it

As this is a notifiable disease  I have to notify DHS

It’s a good idea to discuss about your condition with your partner

! 4!
98
INFECTIOUS DISEASES

Liver Cancer Councelling – Hepatitis B


A 45 yo man comes for a general check up. His older brother recently died in Vietnam from
liver cancer. He wants to know if he is at risk of liver cancer or not.

Task: take history, examination, investigate and management plan.

Accountant; No stress at home, stress at work; Eat healthy diet; Occasional alcohol; Don’t
smoke; Not on any medication; no symptoms and signs of hepatic disease, FHx

GA: no signs of anemia, jaundice

Ix:
FBE normal: platelets 149
LFT:
- bilirubin 12 (<20)
- albumin 38 (38-48)
- ALT 45 (<35)
- AST 48 (<35)
- AP 60 (<130)
- GGT normal
Hepatitis B:
- HBsAg (+)
- Anti-HBc (+)
- Anti-HBs (-)
- HBeAg (-)
- Anti-HBe (+)
- Anti-Hcv (-)
- HBV DNA 20 000 IU/mL (100 000/mL)
Ultrasound

HBeAG (-) chronic hepatitis B with ongoing liver inflammation


There is a concern that he may have advanced fibrosis and cirrhosis
Concern of early portal hypertension because decrease of platelet count

Mx:
Refer to hepatologist & gastroenterologist
Liver biopsy to assess the stage of the disease

There are 2 antiviral therapy:


1) Oral antiviral agent including Nucleoside & Nucleotide analogues
- Effective therapy to suppress the viral replication
- Liver enzymes will go back to normal and liver histology will improve
- Use for life long
2) 180 microgram subcutaneous interferon alpha 2A every week for 48 weeks. If virus is
still active need to give the oral agent.

Vertically acquired who remain HBS(+) at risk of having hepatic failure and cancer
Should be screened for carcinoma 6 monthly USG and alpha feto protein and LFT

! 5!
99
INFECTIOUS DISEASES

Request for HIV Test


56 yo John requesting HIV test after travelling to Thailand and having sex with a prostitute.

Task: take history and management advise.

Hx:
When did it happen?
What type of sex?
Any use of condom?
What is your sexual orientation?
How many partners did you have in the past? Casual or stable?
With casual was it protected sex?
Any problem with sexual intercourse? Any previous STD?
Any IVDU history?
Any history of tattoos?
Any medical problem? DM HTN
Previous surgery or blood transfusion
SADMA
Social history

Explanation:
HIV is a virus which targets our immune system which reduces our ability to fight infections.
Spread of HIV can be by blood or bodily fluid.
This can happen by unprotected sex or sharing needles with other people with HIV.
There is a small chance of transmitting HIV by vertical method.
There is no cure but there is treatment to prevent the progression depending on viral load.

Signs:
Acute viral symptoms
Incubation period 2 months to 20 years. Average of 10 yrs.
30% of people are healthy carrier, they didn’t get it but they spread.
Test is done to look for antibody. It can be from 1 to 12 weeks.
Protein produced by our system they are antibody. Will repeat the test in 3 months time.

Also screening for other STD: chlamydia, gonorrhoea, hep C, hep B, herpes, syphilis.
No results are given on the phone. It may take about 2-3 days.
PCR from urine test - first stream of urine. Chlamydia, Gonorrhoea.
Blood test : serology . For syphilis VDRL
Don’t share toothbrush, needles, shaving blades.

! 6!
100
INFECTIOUS DISEASES

HIV Counseling
John 56, revisit for HIV test result. (+) ve for HIV and (-) ve for other STD.

Task: Explain the result. Advise further Mx

Breaking bad news. What are you expecting or do you have any expectation of your result? I
am sorry to say that your test is +ve. Confirmatory test western blot. And CD 4 viral load.
Reffer the pt to infectious specialist and recumbent treatment if needed.

Tablet: ART depends on viral load and CD4. Have good survival rate with medication and

Notify: sexual partner and DHS. Any health professional you are coming contact with you
should notify.

Metrosexual: sex with married persons.


Don’t share needle, shaving kit, tooth brush.

Advise:
Legal obligation: notify DHS with full name.
I am here to help you. I will help you with finding support and coping process.
Duty of care towards the patient - offer counselling and support.

If the test Is -ve:

Repeat the test in 3 months


Don’t share toothbrush, needles, shaving blades
Not to do jobs related to
Advice about harm reduction strategy.
Alcohol restriction
Healthy diet

! 7!
101
INFECTIOUS DISEASES

Ross River Fever


A 46 year-old male came to your GP clinic complaining of aches and pains in the joints
especially the shoulder and maculopapular rash. (Myalgia, Arthralgia, Rash).

Task: take history, physical exam from examiner, diagnosis and management.

Pain is present all the time, started 2 weeks ago, no variation, pain in the joints especially
shoulder and knee. He travelled to Queensland 3 weeks ago, rash appeared 2 days after
returning from Queensland.

DDx:
- Dengue fever
- Epstein-Barr virus
- HIV
- Malaria
- Rheumatoid arthritis
- Rheumatic fever

Hx:
When did the joint pain started
When did you notice the rash
Which joints were affected
Severity
Any swelling in the joint
Any muscle pain
Any fever
Headache
Pain behind the eyes (to rule out dengue fever)
Any metallic taste (to rule out dengue fever)

! 8!
INFECTIOUS DISEASES

Any change in the mood (in dengue fever there can be depression)
How does it affect your life
How’s your appetite
Any significant past history
Medical history
General health
Any medication
Allergic to any medication
Any recent travel
Any mosquito bite
Sexual relationship, are you in a stable relationship
Have you travelled with your wife
Any unprotected sex
SAD
Anyone in the family has a similar condition
Anyone in the family has rheumatoid arthritis
Any stiffness in the joint

O/E:
GA: Rash maculopapular mainly in the lower limbs, regional lymph nodes (+)
VS: fever
Joint swollen but no deformity
Abdomen: organomegaly

Explanation:
Due to the recent travel to QLD & mosquito bites, you’re likely to have a Ross river virus
infection. I need to do some blood tests:
- FBE
- U&E
- Serology to check Ross river antibody, Epstein-Barr, HIV (pretest counselling before
HIV)

Symptomatic treatment
- Bedrest
- Increase fluid intake
- Painkiller

This condition usually resolve in 2-4 weeks, if you want I’ll give you a leave certificate.

Ross river fever is a notifiable disease.

Dengue fever (Mosquito bite)


- Severe headache
- Retro-orbital pain
- Fever
- Rash
- Muscle and joint pain
- Abdominal pain
- Metallic taste

! 9!
102
INFECTIOUS DISEASES

Travel Advice
A 25 year-old Jimmy Jones who sees you for a travel advice come to your GP practice.

Task: Take relevant history and give advice.

Hx:
Where do you intend to travel
For how long you’re going to be there
Have you arranged for a good travel insurance
Are you going alone or as a group
How about your reservations and hotel bookings at your destination
Is there any significant problem before like past history of DVT
Have you been diagnosed with DM and hypertension (chronic health problems)
Are you on any medication like blood thinning product such as heparin or warfarin or any
other medication  give 1 injection before travel and 1 after arriving back from the travel
For women  oral contraceptive pills
Do you have any plan to make any tattoos or bushwalking
Are you aware of your vaccination status
Ask SADMA
General health: Any recent history of surgery or heart attack/troubles

Pre travel
Update vaccination status according to destination, may include typhoid, hepatitis A, B,
yellow fever, meningococcal vaccine
You need to have a booster of tetanus and diphtheria as well
Malaria prophylaxis  Doxycycline 100 mg daily 2 days before travel, during the entire stay
until 4 weeks after coming back or weekly tablets (SE: photosensitivity)
Avoid mosquito bite, apply skin repellent creams, wear long sleeve clothes, avoid wearing
dark coloured clothes
Medication travel kit  bandage, elastoplast, dressing, water purification tablets, sunscreen,
topical analgesics, topical antifungal cream, antacids, antibiotics, rehydration mixture
gastrolyte.
Dental check-up
DVT prophylaxis
I will give you written materials, if in any doubt come and see me.

During travel
There is a risk of DVT in long flights >6 hours, try to move around
Do not take too much alcohol
Drink plenty of fluids
Take care with food and water including ice cream, drink bottle water or boiled water
traveller’s diarrhoea
Always practice safe sex and use adequate protection
No tattoo
Never walk barefoot at night in snake areas, wear shoes
Always carry a vaccination certificate
Better to have and keep any document of any known health problems
Try to sleep and rest at a transit stop to reduce jet lag on arrival at your destination
Take adequate rest before taking up critical task

! 10!
INFECTIOUS DISEASES

On returning back
See your GP again
Get a blood test done – FBE, blood thinning, as required

Any further questions


Give written materials/pamphlets

! 11!
103
INFECTIOUS DISEASES

Pyrexia Of Unknown Origin


Fever, weight loss, tiredness, arthralgia, myalgia.

Causes:
Infection:
- Infectious Mononucleosis
- HIV (unprotected sex, IVDA)
- Hepatitis (sex, travel, blood, IV, tattoo)
- Cytomegalovirus
- Herpes
- Rubella
- TB (weight loss, travel)
- Atypical pneumonia
- Malaria (malaria, weight loss, fever)
- Infective endocarditis (chills, rigors)
Malignancy
- Lymphoma (pruritus, lumps bumps, night sweats, chills, rigors, rash)
- Leukemia (easy bruising, infection)
Autoimmune
- SLE (joint pain, muscle pain)
- RA
- Sarcoidosis
- Rheumatic fever
- Haemochromatosis

Hx:
Headaches
Unprotected sex
Hepatitis
Jaundice
Tiredness
Nausea
Childhood history
IV drugs
Tattoo
Piercing
Blood donation
Blood transfusion
Contact, weight loss, night sweats (TB)
Hot flushes
Any lumps or bumps
Any PV discharge
Appetite
Past medical history DM, Hypertension, surgical Hx
SADMAS

O/E: Examine all systems


Head
Skin

! 12!
INFECTIOUS DISEASES

ENT
Neck
Thyroid
Chest
Heart
GIT
Urinary

HIV: drug marks,


Hepatitis: jaundice, scratch mark, right upper quadrant tenderness
Herpes: rash
Lymphoma: hepatosplenomegaly, lymphadenopathy, scratch mark
Leukemia: bruise, lump,
SLE: malar rash, discoid, episcleritis
Hemaochromatosis

Ix:
- FBE
- CRP, ESR (inflammatory markers, CRP is faster & cheaper, CRP is more with infection,
ESR more with autoimmune, CRP is diagnostic for some diseases eg in temporal arteritis
to see the improvement, in hospital usually CRP is ordered)
- U&E
- LFT
- Septic work-up : blood culture, CXR
- HIV screen
- Sputum
- Blood film thin & thick  thick for malaria to see the parasite more obvious
- ANA, Rheumatoid factor
- Echo, blood culture (IE)
- Leukemia: blood film
- SLE: ANA, RF
- Sarcoidosis: X-ray chesta
- Haemochromatosis: Iron studies
- Hepatitis: serology (hep B, hep C)
- TB: Chest X-ray, Mantoux test
- Glandular fever

! 13!
104
INFECTIOUS DISEASES

Intermittent Fever
A 28 yo female c/o generalised muscle weakness, generalised tiredness.

Task: History, examination, investigations & treatment.

Positive findings:
- Intermittent fever for the last 1 month
- Night sweats usually occur in lymphoma, HIV, TB
- Losing weight within 3 months time
- Travel history to Bali 6 months ago
- Went with husband, stay at a very good hotel (no extramarital relationship)
- Examination finding: left cervical lymph node

Think of:
- Atypical pneumonia (came with generalised malaise – ask for sputum, nothing in the
chest)
- Glandular fever
- Infective endocarditis
- HIV

DDx:
- Lymphoma
- TB
- Malignancy
- Infectious mononucleosis
- Toxoplasmosis
- Cytomegalovirus
- HIV (need to do precounselling)

Hx:
When did you notice---1 month ago
How did it start
What do you mean by the tiredness---I’m not able to do work as before, I want to go to sleep
Any fever
How often you get the fever
When – morning or evening (pattern)
Low or high temperature
What do you take for the fever
Any rigor
Any vomiting
Headache
Any rash

! 14!
INFECTIOUS DISEASES

Any joint pain


Any cough
Any sputum
Any chest pain
Have you TRAVELLED anywhere
Whom did you go with
Did you go alone
Where did you stay
Any mosquito bite
Any unprotected sex
Any contact with animals (cats & dogs – toxoplasmosis)
Hygienic food, any street food
Any contact with TB
Any lumps in your body
How’s your partner, does he have similar problem
Did you receive any vaccine before travel
Any night sweat
Any change in your weight
How’s your appetite
How’s your bowel habit
Have you received any blood transfusion
Any tattoo
SADMA – any recreational drug, any IV drug use
How’s your general health
Any significant medical illness
Have you been admitted to hospital before
Any minor or major surgery before
Excessive drinking of water, passing urine (DM)
Any family history of DM
How’s your mood (Depression)
Any recent dental procedure (tooth extraction – to rule out infective endocarditis)

O/E:
- GA: alert, signs of pallor, jaundice, clubbing, rash
- Lymph node  consistensy: hard, rubbery, size, fixed or mobile
- Respiratory exam: any added sound
- ENT
- CVS
- Abdomen
- PR
- Joint examination

! 15!
INFECTIOUS DISEASES

Ix:
- FBE
- U&E
- Toxoplasma, Cytomegalovirus, Infectious mononucleosis serology
- Tumor marker
- Mantoux test
- HIV
- Chest X-ray
- CT abdomen

! 16!
105
INFECTIOUS DISEASES

Glandular Fever
A 20 year-old boy complained of sore throat and rash on the back.

Task: take history, examination, diagnosis and management.

DDx:
HIV
Hepatitis
Streptococcal tonsillitis
Lymphatic leukaemia

Symptoms: febrile, sore throat and glandular (with lymphadenopathy)

Positive findings:
Temp 38
Petechiae on the palate
Generalised lymph node enlargement
Slight enlargement of spleen

Hx:
How long have you been suffering from the rash & sore throat
- Sore throat first one week ago later I found by accident the rash on my back
- He looks unwell
Have you had any neck pain or stiffness (to exclude meningitis)
Any abnormality in your mouth like ulcer and spot
Any tonsil enlargement
Any sneezing, cough, block nose, any discharge from the nose, what’s the colour
Is the rash itchy---annoying but not itchy
Description of the rash (fine papular rash)
Any application on the rash, have you used any cream---no
Anyone in your family has this condition---no
Are you sexually active
What about your partner
Have you used any medication for your condition
Any similar attack before---my partner has the same condition last week
Ask rheumatological disease or inflammatory bowel disease because they can come with rash
I feel tired, fatigue, not like before
Any headache, vomiting
Any lumps and bumps in your body---I feel some lumps on my neck
Any bleeding disease

O/E:
GA: jaundice, myalgia, looks tired, voice (nasal tone because of blocked nose)
VS: T 38, BP normal, P 90
Rash: red, fine popular rash on the back and upper part of chest
ENT: palate – petechiae (small red spots), tonsil free
Pharyngs: can have exudative pharyngitis
Lymph node all over the body, majority post cervical but can extend to other lymph nodes
Chest, heart free

! 17!
INFECTIOUS DISEASES

Abdomen: can have hepatomegaly (+/-), 50% have splenomegaly

Ix:
FBE
- White blood cell increase (lymphocytosis)
- Blood film  atypical lymphocytes
- Paul Bunnell test (Monospot)  heterophyl antibody (specific for Epstein Barr virus)
- EBV antibody test (antibody for the capsule and for the core)
• VCA – virus capsule antigen antibody
• EBN-A  Nuclear or core antigen antibody

Explanation:
From the history & examination, you have glandular fever or infectious mononucleosis which
is caused by virus called Epstein Barr virus
Main symptom is fever with large lymph nodes and sore throat (triad glandular fever)
Incubation period is one month
Can have symptoms from 1-6 weeks with anorexia, headache, sore throat, nasal tone
Signs: pharyngitis sometimes exudative, petechiae in palate, lymph node enlargement,
splenomegaly, can have hepatomegaly

5% who have glandular fever have rash from the beginning


95% rash is precipitated by amoxicillin
Common in 15-30 years

Is it risky doctor?
You will have the supervision and good rest at home
It has some complications such as skin rash, depression, weakness, hepatitis, and in rare
condition ruptured spleen
Neurological manifestation like facial nerve palsy (treat with corticosteroid)

Mx:
Rest at home - indoor (Medical certificate for 4 weeks)
Take plenty of fluid
Painkiller for fever
Crush aspirin with sugar lotion and gargle
Avoid alcohol, fatty food, contact sport, no effort

May need corticosteroid if:


- There is neurological manifestation
- If he has some obstruction in his airway
- Thrombocytopenia

Prognosis is very good

! 18!
NEUROLOGY

LIST OF NEUROLOGICAL CASES


1. Epidural haematoma (same side haematoma as the pupil dilation, leg weakness on the opposite
side)
2. Subarachnoid haemorrhage
3. Pituitary adenoma (bitemporal hemianopia – unable to look on the side, tunnel vision (tonometry
normal or not to exclude glaucoma); loss of vision – examination with a red pin
4. TIA
5. Essential tremors
6. Bells palsy
7. Lateral Medullary syndrome
8. Cervical spondylosis
9. Chorda equine syndrome
10. Radiculopathy (L4, L5, S1)
11. Epilepsy
12. Medullablastoma (paediatrics)
13. Tension headache (paediatrics)
14. Migraine
15. Examination of the cranial nerve, upper and lower limb
16. Cerebral palsy (paediatrics)
17. Examination of unconscious, comatose, confusion (multiple sclerosis, dementia, Alzheimer)
18. Brain death – counselling case

! 1!
106
NEUROLOGY

Tension Headache
You’re intern in ED. A 35 yo engineer c/o increase headaches over the last 5 years. The pain
was dull, no aura, no vomiting, no local signs. Usually appearing when he’s tired. Improving
with Panadol. He’s a very busy man.

Task: Hx, examination, do relevant investigation if it’s needed and manage this case.

Hx:
Pain Qs
Where on the top of my hat, like someone is squeezing my head
Does it radiate elsewhere? (to exclude neck stiffness)
How often do you get the pain?
Anything make it worse?---when I’m tired, my headache increase
Have you seek any help? --- to GP 1-2 times, give me Panadol, sometimes help sometimes no
Any vomiting (esp. in the morning), blurring of vision, any other weakness in your limbs?
Did you have any history of trauma or injury to the head in the past?
Have you had any troubles with sinuses?
FHx of migraine or brain tumor
General health
Stressor job, home situation
Depression Qs
SADMA + caffeine

O/E:
GA, VS, systemic review
ENT: tenderness of sinuses, fundoscopy
Neurological exam

Explanation:
You have Tension Headache. Common condition. Not risky.
It’s a muscle contraction of the head, always associated with stress and tension
It’s caused by over activity of your scalp muscle, forehead and neck
Like a tight a band around your head, or putting heavy weight on top of your head

Mx:
1. Prevent the triggering factor  relaxation technique
2. When you have the headache attack, choose a quiet room
3. I will give you painkiller like panadol or aspirin
4. Sometimes Amitryptiline, Diazepam
5. Massage with your neck
6. Refer to physiotherapy to have more exercises

! 2!
107
NEUROLOGY

Temporal Arteritis
A 60 year-old male has recurrent headaches & pain in temporal area. He has unusual fatigue
and loss of appetite since one month.

Task: Ask history, investigations, management.

Headache DDx:
Subarachnoid haemorrhage (trauma, fall)
Temporal arteritis
Meningitis (neck stiffness, rash, fever)
Space occupying lesion
Migraine
Tension headache
Cluster headahce
Sinusitis

Hx:
PAIN Q
Where is the pain – can you point it with one finger
How long? – 2 weeks (need to treat immediately)
Continuous or on and off (migraine, tension)
Describe the pain ---dull, sharp, throbbing, burning
Is the pain going anywhere else
Are there any aggravating factors like loud noise, bright lights, perfumes, food, posture
(looking down  sinusitis)
Anything that makes it better  sleep, quiet room (clue to migraine)
Are you taking any medication for this pain? Did it help?
Is this the first time
Any other associated symptoms like blurring of vision, double vision, any fever, rash, nausea,
vomiting, any pain when you chew your food, any recent weight loss
Any resent history of trauma or fall
Stress (rule out tension headache)
For cluster headache  ask any watering of eyes, runny nose, congested nose
General headache
Morning stiffness of the shoulder and pelvic area (associated with polymyalgia rheumatica)
Any significant medical condition, any blood vessel disease, hypertension, diabetes
Family history (runs in the family) of similar conditions
SADMA

Explanation:
John, I think you have got a condition called temporal arteritis.
It’s an inflammation of blood vessel. This area is the temporal, the blood vessel there is
inflamed, that’s what causing your headache.
Uncommon. The cause is unknown.
It’s a good thing that you’re here today. I will prescribe you high dose prednisolone 60-100
mg before undergoing Ix to decrease the risk of eye problems.

! 3!
NEUROLOGY

Ix:
Full blood examination
C-Reactive Proteins
Erythrocyte Sedimentation Rate OPHTHALMOLOGY R/V URGENT
I’d like to refer you to a surgeon, he may do biopsy of the superficial temporal artery to
confirm the Dx.
- Outpatient procedure by the specialist), will leave small scar, parallel to the hair line (so
not obvious).
- Sometimes the biopsy (-) but doesn’t mean there’s no inflammation  skip lesion
(scatter)  do multiple bunch biopsy (collecting from multiple places)
Specific and sensitive test  MRI
Serological test to rule out rheumatic arthritis, connective tissue disease, myeloma.

I’ll give you high dose of steroid now, but once symptoms control and ESR levels falls, we
can gradually decrease to maintenance levels (5-10mg tid). If the Dx is confirmed, you need
to take steroid for 2-3 years.

Do you have any problem of indigestion or Hx of peptic ulcer?

Because the risk of develop visual impairment, I’d like to urgently refer you to an
ophthalmologist. Blindness is a serious complication of this condition, and if it occurs it’s
usually irreversible. Fortunately, this complication is preventable, the early we use steroid,
the better outcome.

The vascular surgeon may do additional tests like:


- MRI
- Serological test to rule out connective tissue disorder like polymyalgia rheumatic (20%)

I will be following you up in 48 hours


Frequent follow up for monitoring symptoms and serial ESR level and SE of steroid.

The long term complications of steroid: osteopenia, hypertension, DM and changed faces.
If the patient is >60 years old & on steroid treatment, has to undergo DEXA Scan, serum
calcium & vit D level.
If uncontrolled DM, cannot give steroids because steroid cause DM  give Methotrexate
(immuno suppressant).

! 4!
NEUROLOGY

Temporal Arteritis Counselling!(AMC 93)

A 60 year-old man came to your clinic for the second time to discuss his diagnosis with you.
He had visited your clinic last week and he was diagnosed with temporal arteritis.

Task: explain the Dx and advice about other Ix. Manage his condition.
Meningitis
108 MENINGITIS
A 32 yo man presented to ED & you’re an intern there. He has headache since yesterday.

Task: take history and examine, investigate & manage.

DDx:
meningitis
Migraine
tension headache
space occupying lesion
subarachnoid haemorrhage (very sudden – eg. coughing)
temporal arteritis (unlikely at this age group)

Hx:
Headache Qs “Is this the worst headache you have ever had?”
Associated symptoms: N/V, photophobia, fever, skin rash, visual changes, sensory or altered
sensation, weakness
Any recent head injury?
Recent flu-like illness
Any contact with a sick person?
Is it the first time?
Any FHx of similar condition?
SADMA
Stressor

O/E:
GA: conscious level
VS: pulse, BP, T (if there is hypotension, bradycardia for raised intracranial pressure)
Fundoscopy
Skin rash
Signs of meningitis:
- Neck stiffness
- Kernig sign (having the person lie supine (flat on the back), flex the thigh so that it is at a right
angle to the trunk, and completely extend the leg at the knee joint. If the leg cannot be completely
extended due to pain, this is Kernig sign)
- Brudzinski’s sign (one of the physically demonstrable symptoms of meningitis is Brudzinski's
sign. Severe neck stiffness causes a patient's hips and knees to flex when the neck is flexed.
Heart, lungs, abdomen

Mx: (call a registrar)


- Take blood for blood culture & blood tests
- FBE, coagulation profile, LFT, U&E
- Give Ceftriaxone (age >55 Ceftriaxone + Ampicillin), change to the culture later
- Admit patient for CT brain, if there is no increased intracranial pressure LP
- Lumbar puncture

! 5!
NEUROLOGY

- Notify to Health Authorities


- Prophylaxis for family members with Rifampicin (or Ciprofloxacin – but Ciprofloxacin is only
for adults)
- If shock  admitted to ICU & resuscitation
- If in country  Antibiotics and ambulance

109
Bell’s palsy (AMC 94)
A 40 yo male presented to GP clinic b/o sudden onset of paralysis of his face.

Task: relevant history, examination findings, diagnosis and management.

Critical error: Not to exclude stroke (cranial nerve and limb neurological examination)

DDx:
Bell’s Palsy
Stroke
Brain Tumor

Hx:
When & how did it happen? ---started this morning
Suddenly or gradually? ---suddenly (brain tumor: gradual onset, stroke: sudden onset)
Is it the first time?
Associated symptoms:
- Pain
- Speech problem
- Limb weakness
- Numbness
- Dizziness
- Swallowing problems
- Hearing problem
- Taste changes

! 6!
NEUROLOGY

- Morning vomiting (tumor)


- Weight loss (tumor)
Recent viral infection  flu-like symptoms
General health  DM, Hypertension, Hypercholesterolemia AF, THYROID
Family history of stroke or brain tumor

O/E:
GA: distress and anxious young man
VS: BP, P, RR, T – normal
ENT: Inspectionany unilateral drop; any vesicular rash around the ear
Cranial nerves examination  All nerves are normal, except for N7
Neurological examination of limbs to rule out stroke

Explanation:
Your condition is called Bell’s palsy
It’s an acute unilateral paralysis of 7th nerve (LMN paralysis)
It’s quite common
Most likely cause unknown, we call it idiopathic or maybe cause by:
- infection around the ear
- mastoid infection
- acoustic neuroma
- cholesteatoma
- parotid cancer

Doctor, do I need to do any investigation?


Not at the moment, it’s a clinical diagnosis, we need to start therapy which is Prednisolone
high dose 40-80 mg for 3 ds, then gradual ↓ in dose for the next 7 days (full course 10 days)
Antiviral agents
Refer to neurologist
Refer to physiotherapist

Are you sure that it’s Bell’s palsy and I don’t need to do a CT scan?
If you’re highly concern, the neurologist may do a nerve conduction study or CT brain to
exclude intracranial pathology, but it’s least likely in your case.

Is it risky, what’ the prognosis?


Prognosis quite good, 70% will have complete recover in 2 months, 20-25% full recover in 6
months, 5-10% do not recover after 1 years.

! 7!
110
NEUROLOGY

Vitamin B12 Deficiency


A 65 year-old complained of increased weakness of the legs and unsteady gait.

Task: history, examination findings, investigations, explanation & management.

DDx:
- Diabetic peripheral neuropathy
- Stroke
- Brain tumor
- Motor neuron disease
- B12 deficiency  Peripheral neuropathy & ataxia (40%)

Hx:
When and how did it happen?
One or both legs
Any weakness anywhere else
Abnormal sensation
Infection: any fever, joint problem, rash
Any recent trauma to the head and falls
Back pain
Bowel & urine habits
Tiredness, appetite, loss of wt
Normal diet: meat, vegetable intake
Hypertension, stroke, DM, Ischaemic Heart Disease, Cerebrovascular events
Any surgery in the past
SADMA alcohol
Any family history of similar problems
Social history
Whom do you live with? alone
Any support?  No
Any financial problem?  on pension
How do you feel yourself? (depression)

O/E:
GA: pale, unwell, distress, BMI high
VS: T normal, little bit tachycardic, BP normal, RR 18-20,
Carotid bruit, heart rhythm
Any peripheral oedema
System review
- Tongue problem
- Tonsil
Neurological examination of upper & lower limbs: tone, power, reflex, sensation,
coordination  ataxic gait, impaired sensation in lower limbs
Proprioception impaired

Ix:
- FBE
HB 92, MCV 108 (78-98)
RBC 3.1, WBC 3.1, Platelet 130

! 8!
NEUROLOGY

Macrocytic anemia megaloblastic


- Check B12 level and folic acid---B12 low (normal <148 pml/L)
- Anti parietal cell antibody and anti intrinsic factor antibody to rule out pernicious anemia
as a cause of B12 deficiency
- TFT
- LFT

Possible causes:
- Decrease intake in vegetarian
- Malabsorption
- Pernicious anemia absorption of B12 (can associated with hypothyroidism)
- Liver disease

Mx:
Refer to haematologist
How to treat – depends on cause
If poor intake (vegetarian)  give B12 oral
If pernicious anemia or malabsorption  give B12 IM

Folic acid deficiency – is usually seen in elderly lady with chronic alcohol use with
macrocytic anemia

! 9!
111
NEUROLOGY

Epilepsy Counselling
A 30 yo man is referred to you by his neurologist diagnosed with idiopathic epilepsy after he had 2
fits. He’s put on carbamazepine and is planning to get married in a few months time.

Task: explain the condition, counsel him accordingly and answer his questions.

Epilepsy is a disorder in which a patient is prone to having recurrent seizures. In your condition the
cause is unknown, called idiopathic, a common type of epilepsy. Exact cause is unknown but usually
result from increase electrical discharge or activity in a part of the brain. Usually is diagnosed after 2
fits.

Can be controlled by medications and most of the people with epilepsy can lead a normal life. Our
aim of treatment will be to achieve a complete seizure control, means fit free.
Most people can lead a normal life, can marry & have kids.

There are a lot of medications, different medication to be used. Your neurologist has started you on
carbamazepine. The treatment is usually preferred to be monotherapy, 1 drug at a time b/o the side
effects. The dose of medication can be increased gradually until getting control. If you’re still getting
fits, the dose will be increased until reached the maximum safe dose. When reaching the maximum
dose and still no control, another medication will be started until getting control. When there is no fit,
the old treatment will be stopped. No tapering.

While you’re on treatment, you’ll need frequent reviews. If you’re seizure free for 3 consecutive yrs,
there’s a chance your Tx to be stopped, the decision is made by neurologist.

Every medication has SE. Carbamazepine can cause nausea, gastric upset, dizziness, visual
disturbances, tiredness, skin rash, abnormal LFT, some neurological SE such as ataxia.

Important to avoid some trigerring factors include:


- Fatigue, exhaustion
- Lack of sleep
- Stress
- Excessive alcohol
- Prolonged or flashing lights especially if the patient is photosensitive
- Prolonged or excessive video games
- Fever

I understand that you’re working as a driver but I’m afraid that you might need to change your job
because it’s unsafe for yourself and other people when you’re driving. You can do most of the jobs,
but not operating heavy machineries, working on heights, working near deep water. I’m afraid that
you won’t be suitable to work as a police, in military, as a pilot and public transport driver. Office-
based job will be better. I can contact Centrelink, there is a job matching network that will help you.

For driving, I’m afraid you might need to stop driving for a while. Driving will be unsafe until you’re
fit free, I’ll check with VicRoads. According to the regulations, people with epilepsy has to stop for a
certain time, I will check. If you have a licence, you will be suspended until 3 months seizure free. If
you have no license, you cannot apply until 2 years seizure free

With sport activities, you can practice most of the sport except scuba diving, parachuting, rock
climbing, and car racing. Swimming is not forbidden if under supervision.

If somebody in the family has fit, the kids have a slightly higher chance of getting epilepsy, about 3
%. I can give you some contact numbers for epilepsy support group & some reading materials.

! 10!
112
NEUROLOGY

Multiple Sclerosis
You’re a registrar in ED. A 35 year-old female presented with a 2-week history of visual
disturbance, pins and needles in the left hand, and difficulty walking.

Task: focus Hx, ask examination finding, Dx, investigation and explain management.

Hx:
- Previous episode
- Accurate physical examination
- Loss of visual acuity, color disturbance, intermittent diplopia, double vision
- What kind of visual disturbance ---cannot identify color, no clear picture
- Pins & needles left arm – is it continuous or on and off, any associated weakness, any
other weakness & abnormal sensation, anywhere else
- Weakness in the right leg
- Difficulty walking, why, please describe---when walk, fall to the side and had a few falls
(ataxia)
- Any headache, nausea, vomiting
- Any pain anywhere when you move your eyes
- Any neck stiffness
- Urinary symptoms and bowel action---urinary urgency, hard to keep my bladder under
control
- Is it the first episode---I had the similar episode 10 years ago
- Family history of myasthenia gravis (autoimmune disease)
- Social history ---single, works as a freelance artist, smoking, drink alcohol

O/E:
GA: Distress, any abnormality in the face, any droop---only some ophthalmoplegia
VS: everything normal
Neurological exam:
Cranial nerve exam N2, N3, N4, N6– visual acuity (decrease), visual field (normal) &
funduscopy: atrophy of optic nerve, slight swelling of optic disc
Any abnormal eye movement ---ophthalmoplegia, nystagmus and double vision
Pupils normal
I’d like to check N5 to N12 ---no other abnormality detected
All other investigations normal
Neurological exam of upper and lower limbs
I’d like to do neurological examination which is tone, power, reflexes, coordination and
sensation---spastic paraparesis in lower limbs with increased reflexes, impaired coordination
in heel-shin test, impaired sensation in left upper limb
Could you please walk---ataxic gait

Findings:
Chaotic neurological changes with some of the brain affected:
- multiple sclerosis
- motor neuron disease

Explanation:
I suspected you have multiple sclerosis, to confirm we need to do an MRI (imaging of your
brain) and visual evoked potentials.

! 11!
NEUROLOGY

How common is multiple sclerosis One in 800 people

Age onset Between 17-35 years (young female reproductive)

Presentation:
Optic neuritis
Weakness of 1 or more limbs
Tingling sensation in extremities
Diplopia
Nystagmus
Ataxia
Vertigo
Bladder dysfunction

Onset maybe acute, subacute and insidious

CRITICAL ERROR:
Diagnose multiple sclerosis, counsel
To confirm with MRI

Explanation:
It’s quite common in your age
Visual disturbance, weakness or tingling sensation
It’s a risky condition
May give relapse & remission or some periods without symptoms
It’s very hard to say now what type
This condition is not curable but it’s treatable
We can slow the progression or increase the periods between relapse or free of symptoms by
giving some medications.
We need to refer to the neurologist

! 12!
113
NEUROLOGY

Recurrent Falls
A 50 year-old male presented to your GP clinic because of recurrent falls. He had stroke 4
years ago with weakness of right sided upper and lower limbs.

Task: take history, ask for examination finding, diagnose/DD, management.

DDx:
1) Neurological
- TIA
- Stroke
- Seizure
- Parkinsonism
- Vision (cataract)
- Ear (Meniere’s disease)
- Vestibulo cochlear (vertigo)
2) Cardiovascular
- Vasovagal attack
- Carotid sinus syndrome
- Arrhythmia
- AS
- HOCM
- MI
3) Medication  Polypharmacy
- Diuretic
- ACE Inhibitor
- Alpha blocker is more common than beta blocker
- Antipsychotic (Haloperidol)
4) Metabolic – hypoglycaemia (DM  hypoglycaemia, neuropathy – autonomic)
5) Alcohol – hypoglycaemia/alcohol itself
6) Environment

Hx:
Fall
- What were you doing at the time of fall?
Exercise  AS
Toiletting  aneurysm
- Any abnormal sensation
- Palpitation  Arrhythmia
- Chest pain  MI
- Sweating
- Visual changes  TIA
- Dizziness/lightheadedness  postural hypotension
During
- Where did you landed
- Did you get hurt
- How long did you come to yourself
- How long were you unconscious
- Did you lose control of your bladder/bowel
- Any shaking/tongue biting

! 13!
NEUROLOGY

- Any witnes

After
- Did you have headache after the fall
- How long did it take you to come to yourself
- Any trauma to head, shoulder or anywhere else
- Any numbness, tingling, weakness

PMHx
- Similar episode in the past
- Any risk factors  DM, hypertension, heart disease, high cholesterol, smoking, obesity
- Have you ever had epilepsy
- Drug: aspirin & heparin / warfarin may need CT
- Do you take medication regularly
- Any mental illness

DM
- When were you diagnosed with DM
- What are you taking for your DM
- Are you taking medication regularly
- Do you usually check your blood sugar level regularly
- Do you eat regularly
- Do you used to skip your meals

FHx
Alcohol  how much, how often
Social history  home environment, any changes, with whom do you live? Alone?

O/E:
GA
VS – BP (sitting/standing); P, irregular
CVS examination, Carotid pulsation
PR
Neurological examination

Ix:
FBE, BSL, U&E, Lipid, ECG, Echo, Doppler (neck), TFT, LFT, Urine dipstick, CT, MRI

Mx:
Depend on the cause
Medication problem  refer to fall clinic
Physiotherapy
Occupational therapist will come to your home to check home environment
If lighting is not good, they will come & arrange it

! 14!
114
NEUROLOGY

Subarachnoid Haemorrhage
A fit 36 yo female, no PHx of any illness, felt sudden severe headache, took Panadol, and lie down. 3
hours later partner brought her to ED b/o a problem in her vision and headache.

Task: focus Hx, examination finding from the examiner, provisional Dx & management.

Sudden, severe onset of headache, feverish, vomiting


 Subarachnoid haemorrhage (<40 years), positive history, sudden death of aneurysm rupture in the
family, certain kidney and liver cysts
 Temporal arteritis (>50 years): localized temporal tenderness

DDx:
1) Sign of increase of intracranial pressure Why?  haematoma, space occupying lession
2) Meningitis – photophobia, fever, rash
3) Seizure
4) Focal neurological signs – cranial nerve palsy
5) Cauda equine syndrome (spinal cord compression)

CT scan
CT angiography (including circle of Willis) – to exclude aneurysm of cerebral artery
FBE, coagulation profile, ESR (to rule out temporal arteritis)
CT scan normal  do lumbal puncture

Hx: (Is my patient haemodynamically stable?)


Are you in pain, do you want me to give you painkiller
Could you please tell me more about the pain?  character of pain
Shooting pain at the back of head
Location
Radiation
Severity Is this the worst headache you ever had?
Intensity 0 – 10 (no pain  excruciating pain)
Onset  sudden
Associated with  fever, rash, vomiting, neck stiffness
Past medical history
Past surgical history
Trauma
Family history
 is your parents still alive, sudden death, siblings
 polycystic kidney disease, liver disease
 aneurysm

Social history  employment (what do you do for living), any wife/partner, are you
coping well at home, are you able to look after yourself
Personal history  SADMA, medication (any allergies or adverse drug reactions),
sexual activity. Drug abuse – amphetamine.
Any previous psychiatry history

O/E:
GA: Drowsy, irritable, photophobic
VS: Pulse regular, no postural drop in BP, GCS - AVPU
(Alert 15; Responding to Voice 13 Pain  8-10; Unresponsive <8)
Pupil: dilated (increase intracranial pressure, herniation, any surgical emergency), fundoscopy
(subhyaloid haemorrhage 60-70% positive)

! 15!
NEUROLOGY

Speech
Gait
Neck stiffness
Cranial nerve 2-10  any cranial nerve deformity?
Peripheral nerve system: tone, power, reflexes, sensation, coordination
Cerebellar sign (hypotonia, tremor, pointing, nystagmus, dysarthria, Romberg test)
What about other system, any cardiovascular, abdomen  normal or not

Ix:
CT scan, CTA (circle of Willis), CT non contrast  if normal  lumbar puncture (after 12 – 48
hours): look for xantochromia
If patient doesn’t want to do a lumbal puncture  MRI/MRA
Cerebral angiogramCT scan positive (don’t forget to exclude aneurysm)
BSL
Urine
FBE, coagulation, ESR

Mx:
Stabilise patient – confirm diagnosis - liaise with neuro-surgical team
- Give Nimodipine
- Hypertension keep BP high (140-160)
- More fluid hemodilution
- Central Venous P 8-10

Complication:
Hyponatremia - cerebral salt wasting (low sodium  brain can swell)
Hydrocephalus
Vasospasm – stroke
Seizures
Re-bleeding

! 16!
115
NEUROLOGY

Subdural Haematoma (Extradural, Biconvex)


A 60 year-old female came to your GP clinic complaining of right arm & leg weakness.

Task: take history, examination and management.

DDx:
Cardiovascular disease – haemorrhagic or thrombotic
Brain tumor
Migraine
Multiple sclerosis
Infection – abscess, encephalitis
Head injury
Hypoglycaemia
Psychogenic

Hx:
When did it start?---a few days ago
One or both side---one side
Is it progressive, increasing or decreasing---becoming worse
Headache---yes
Any other problem like speech difficulty, blurring of vision, dizziness (vertigo)
Loss of sensation in any part of your body (numbness)---no
Nausea, vomiting
Is it the first time, did it happen before---no, first time
Recent head injury or falls (2 months ago)
Did you lost consciousness?
Any chest pain, palpitation
Risk factors: Smoking, alcohol, diet, exercise, DM, HPT, high cholesterol, AF, stroke
Medication: aspirin or warfarin INR
Any bleeding from anywhere of your body (nose bleeding, gum bleeding)
Family history of heart condition

O/E:
GA: posture, BMI, bruise; Gait: limping or not---problem on the right side
VS: pulse 90 regular, BP normal
CVS: Fundoscopy, carotid bruit
Neurological examination:
Check speech, orientation, gait
Cranial nerves
Upper and lower limb (positive findings tone & power reduced on the right side)
Proprioception, coordination, reflex---normal

! 17!
NEUROLOGY

Explanation:
From history & examination, most likely you have a condition we called subdural haematoma
because you’re taking warfarin and you have a history of trauma. I’d like to refer you to the
hospital by ambulance. In hospital, you will be assessed & managed by neurologist. They
will order an urgent CT scan of head CT showed clot & midline shift

Ix:
- CT scan
- FBE, Clotting profile (INR)
- BSL
- Lipid profile
- U&E
- ECG, echo, holter
- Doppler ultrasound

Mx:
Lifestyle modification, healthy diet
Quit smoking
Reduce alcohol
Check INR regularly

! 18!
116
NEUROLOGY

Hyperprolactinemia – Pituitary Tumour


A 20 yo female c/o not having her period for the last 12 months.

Task: relevant Hx, PE findings from examiner, further Ix if needed, explain Dx & Mx to pt.

The causes of secondary amenorrhoea


- Pregnancy
- Metabolic causes
o Uncontrolled/unstable DM
o Renal failure
o Hepatic failure
o Thyroid diseases
- Hypothalamic
o Eating disorder  anorexia nervosa
o Excessive exercise
o Emotional stress
- Pituitary
o Tumour
o Microadenoma
o Pituitary infarction (Sheehan syndrome) – any milk discharge
- Ovarian
o PCOS (acne, hair)
o Premature Ovarian Failure (hot flushes, dry vagina)
- Uterine
o Asherman syndrome (D&C)

Ix:
Pregnancy test
BSL, U & E, LFT, TFT, Prolactin, MRI, examine eye (bitemporal hemianopia), throat (thyroid),
ultrasound of ovary

The cases of primary amenorrhoea


- Sexually infantile (secondary sexual characteristic)
o Short stature: Turner’s syndrome, Hypothyroidism or cretinism
o Normal stature + anosmia: Kallman’s syndrome
o Normal stature without anosmia: True gonandal dysgenesis
- Masculinized
o True haemaphrodism
o Androgen secreting ovary tumours
o Congenital adrenal hyperplasia
- Normally feminised
o Mullerian developmental abnormalities
o Androgen insensitivity syndrome
Constitutional delayed and imperforated hymen

Hx:
Was your period stoped gradually or suddenly?
When was the last menstrual period exactly?
Since then any spotting?
Any lower abdominal discomfort or pain?
Menstrual Hx
Any similar problem in the family
Are you currently sexually active?

! 19!
NEUROLOGY

Stable relationship
Any previous STD
Any current vaginal discharge
Any previous pregnancies or miscarriages one time termination
What age of termination
The cause of termination
Any OCP, how long, what was the dose of contraception
Any current hot flushes, dry vagina, painful intercourse (premature ovarian failure)
How much exercise you do daily (exercise induced amenorrhoea)
Any stress at home or work
Any change in your appetite, any change of weight
Any abnormal hair distribution or growth or acne (lanugo hair for anorexia, hirsutism)
Any headache, blurry vision, milky secretion
Any thyroid disease or lumps or bumps or neck swelling
Any other systemic illness or DM, high BP, kidney or liver disease
Are you on any medication
When was the last Pap smear
SADMA
Did you receive your Gardasil vaccination

O/E:
GA: pallor, BMI, Distribution of hair, Acne in face
VS: examine from head to toe
- Vision field & acuity
- Neck thyroid enlargement
- Chest and heart
- Breast – milk and nipple secretion
- Abdomen – organomegaly, tenderness, mass, size
- Take consent & examine the pelvic area
- Pelvic - enlarged organ
- Inspection - secretion, abnormal feature of genitalia
- Speculum - size uterus  (if there is abnormal size of uterus  endometrial sampling by swab)
- Bimanual - cervical excitation
- Take consent for pregnancy test, urine dipstick, random BSL

Ix:
Pregnancy test, urine dipstick
FBE, U & E, LFT, TFT, LH, FSH and the LH/FSH ratio, serum prolactin (in this case ---3000), serum
androgen, testosterone, serum insulin (for PCOS), ultrasound, serum estrogen
CT or MRI of the brain
DEXA scan for osteoporotic changes (amenorrhoea for 12 months  liable for osteoporosis)

Explanation:
The problem in your absent period is a condition which we called secondary amenorrhoea. From my
exam & investigation, you have a high level of hormone named prolactin. The most likely cause in
your case is possibly due to a growth in the master gland which is located in the brain known as the
pituitary gland. We’re not sure if this growth is benign or malignant. Most likely it’s benign. For this,
there’s a specific test to be done by MRI. I will refer you to an endocrinologist who will decide upon
the size he found. Most likely he will start you on a medication called Bromocriptine, which will
reduce the size. Macroadenoma if the size is big. You need also to be assessed for osteoporosis as you
have no period for 12 months. For this, I will refer you to a gynaecologist who may start you on HRT
as needed. You may need to have a surgery through the nasal approach.

! 20!
117
NEUROLOGY

Alzheimer Disease: Counselling


A 35 yo woman came to your GP clinic asked about her father who has recently been
diagnosed with Alzheimer disease. She’s worried and concerned. CT scan showed atrophic
changes of the brain.

Task: counsel and answer patient’s daughter’s questions.

FIRST get consent from the father, ask examiner.


I’d like to discuss your father’s condition, what he can do.

It’s a kind of dementia, in which there is some wasting of the brain’s cells. The cause is not
certain.
Usually occurred after 65 years
Affects the brain, personality changes
Gradual memory disturbances
Forget family members
Might have malnutrition, poor personal hygiene
More prone to accident

Can I get it in the future?


Usually occur after 65 years of age
There is a rare type of Alzheimer, if it’s run in the family, it can happen before 65 years

How do you diagnose it?


Diagnosis is by exclusion, clinical diagnosis
CT scan also shows atrophic changes of the brain
But we can only confirm it by autopsy after death, we will take tissue and examine it under
microscope

Can it be depression?
Yes, it could be. We called it pseudo-dementia.
We can find out by talking to your father

Mx:
As your father is 65 years of age and has been diagnosed with Alzheimer disease, I’d like to
refer him to ACAT (Aged Care Assessment Team) to assess your father’s disability and what
kind of service is available for him.

He’ll be managed by a multidisciplinary approach with geriatrician, psychiatrist/psychologist,


physiotherapist, occupational therapist, nurses, social workers, GP, volunteers and support
group.
- The geriatrician who assess your father might prescribe him some medication, although
Alzheimer is untreatable, it can slow down the progress of the disease.
- Psychiatrist will assess whether he’s having a mental problem or not. If he needs any
medication, he will be given and just watch out for the side effects of the medication.
- The physiotherapist will assess the patient’s ability to walk, will provide walking aids and
show some exercise.
- Occupational therapist will visit your father and assess the house condition if there is poor
light, loose carpet, railing if needed, bathroom slippery or not.

! 21!
NEUROLOGY

- Social workers provide meals-on-wheels


- Nurses will keep medical records
- GP will do the regular home visit
- Support group

- Home environment is familiar to him. Better to manage him at home and he needs regular
home visit by sympathetic caring people, friends, relatives
- Respite care  if you’re unable to take care of him, you can get help from the respite care
people temporarily
- Lifestyle modification can improve quality of life and delay progression of the disease 
diet, exercise, personal hygiene
- 4R refer, reading material, review
- Need to assess the patient’s ability to drive

Any special medication can prevent this condition?


Unfortunately no

! 22!
118
NEUROLOGY

Posterior Inferior Cerebellar (PICA) Infarction (AMC 72)


A 50 year-old man presented to your GP clinic due to severe dizziness.

Task: take history, physical examination findings, discuss management plan.

Hx:
Dizziness, when did it start
What do you mean by dizziness? Is it lightheadedness or vertigo, spinning around
Vertigo  ask other symptoms  any tinnitus (ringing sounds in the ear), loss of hearing 
to exclude Meniere or Acoustic neuroma
(Vertigo is worse with positional change)
Vertigo without tinnitus and hearing problem is unlikely related to ear
Vertigo with ataxia is more likely to relate to cerebella
Associated symptoms
Headache, nausea, vomiting, balancing problem, gait, fever, neck pain
Cerebellar signs – loss of balance
Face – cranial nerve
Any visual problem, double vision, any numbness/weakness in face, any problem in
speaking/swallowing
Any weakness/numbness in limbs
Gait – any difficulty in walking or maintaining balance
Headache, fever, neck pain
(Lesion in the brain stem  lesion is on the same side of the face, sensation loss is on the
opposite side of the lesion)  contralateral face & body (cranial nerve – LMN)
Lesion above the brain stem (high lesion)  weakness & numbness on the same side of face
& body  facial (UMN)
No motor involvement in PICA (pyramidal tract OK)
Any head injury
System review chest pain, SOB, palpitation V
Past medical history – risk of cardiovascular disease: A
- hypertension
- DM N
- IHD
- TIA/Stroke I
- high cholesterol
- positive family history
S
- obesity H
- stress
- peripheral vascular disease E
- sedentary lifestyle D
- SADMA: smoking, alcohol, medication
- unmodifiable risk factor such as male and age

O/E:
GA: leaning to the left side, overweight, BMI
VS---normal
Full CVS examination---normal
Full neurological examination

! 23!
NEUROLOGY

Neurological examination
9th Cranial nerve: dysphagia, dysarthria
8th Cranial nerve: vertigo, nausea, vomiting, nystagmus
Inferior cerebellar peduncle – vertigo, ataxia
Horner’s syndrome (sympathetic fibre involvement) – ptosis, meiosis, anhydrosis
Loss of sensation on the same side of the face & opposite side of the body
5th Cranial nerve: absent direct & consensual pupil reflex, absent corneal reflex
Cornea reflex - 5th afferent, 7th efferent
(on the affected nerve, the direct pupil reflex and the consensual reflex are absent while on
the normal side everything is normal)
Sensation  Spinothalamic tract – sensory loss on the opposite side (cross pattern)

Explanation:
I’m afraid that you’re having a kind of stroke but not the classical one. It’s called PICA
syndrome. The blood vessel involved is the Posterior Inferior Cerebellar Artery:
 numbness but no weakness
So we need to admit you to the hospital to do more Ix and urgent assessment by neurologist
- CT scan to rule out tumor and MRI to confirm the type of stroke, to know if it’s
ischaemic or haemorrhagic stroke
- Blood tests: FBE, RFT, LFT, coagulation profile, ECG

Mx:
If ischaemic stroke, thrombolysis if there is no contraindication

Carotid bruit & Doppler  carotid stenosis is not a reason of PICA


It will block anterior, middle, posterior cerebral artery

DDx:
Stroke
Acute labyrinthitis
Benign Positional Vertigo
Meniere’s disease
Migraine
Cerebral tumor
Multiple sclerosis

! 24!
119
NEUROLOGY

Benign Essential Tremor (AMC 33)


!

A GP setting, a 40 year-old male consulting you for tremor.

Task: focus history, examination, investigations and management.

DDx:
Alcohol-related
Parkinson
Hyperthyroidism
Anxiety
Liver disease
Cerebellar disease

Hx:
Tremor
- When did it start
- Is it in one hand or both hands
- Other tremor or shakes during rest or during any specific moment
- What makes it better or worse
- Is it on and off or continuously present
- Are there any shakes in your head
- Do you think that your voice is shaky
- Are you aware (if someone has commented) if the tremor is present during sleep
- Can you control the tremors
- How are the tremors affecting your life
Parkinson
- Any muscle stiffness (to exclude Parkinson)
- Any difficulty in moving around (gait)
- Any slowing of movements
Thyrotoxicosis
- Do you feel unusually hot, more nervous than before
- How’s your appetite, wt
Cerebellar
- Any problems in balance
Sexual Hx
- Any change in your sexual life
- Any difficulty in maintaining erection
PMHx: thyroid or liver diseases
Stressors
FHx: essential tremor, parkinson
SADMA + caffeine

O/E:
GA: any icterus, any pallor, any nystagmus
VS: BP sit & stand, pulse, temperature
Thyroid examination: thyroid gland palpable
Any lymph nodes
Tremors – resting or on activity, fine or coarse, present anywhere else in the body
Power, tone, reflexes
Cerebellar signs
- Romberg test
- Heel-shin test (dysdiadochokinesis)

! 25!
NEUROLOGY

Cardiovascular exam
- Tachycardia (for thyrotoxicosis)
- Heart sounds, any advantageous sounds, any murmur
Respiratory & abdominal exam---normal

Explanation:
Most likely your tremors are benign essential tremors
We need to do some investigations to confirm:
- FBE – macrocytic anemia
- LFT
- TFT
Benign essential tremor runs in families (autosomal dominant). As the name suggests it’s benign, not
harmful.
The cause is unknown.
Usually the frequency & amplitude increases with age

The treatment, we can start you on:


- Beta blocker (propranolol) with a small dose.
- We can also think of physiotherapy for the muscles
- Advise safe level of drinking alcohol (Alcohol can seems to temporarily relieve the tremor) – DO
NOT ADVISE ALCOHOL TO HELP THE TREMOR it may stop the tremor now but you can get
addicted to it
- Prescribe tranquilizers (benzodiazepine) if very anxious, can’t walk, affecting his life
- Support group  Essential tremor foundation group

To differentiate from Parkinson:


- Control of the tremor  intentional tremor increase (BET), resting tremor (Parkinson)
- If patient distracted the tremor stop (Parkinson), looks at the hand and tremor increase (BET)
- BET coarse tremor, shaky voice while Parkinson fine tremor

Patient hold the cup and look at it, if the tremor stop, he can control the tremor  Parkinson
You are more likely to have the more serious Parkinson's disease if you have the tremor at rest, a slow
heart beat, rigid firm muscles, slowness to remember facts, difficulty speaking normally, depression
or sleep disturbances. Benign (Essential) Tremors usually do not need to be treated and early
treatment does not prevent this condition from becoming more severe. When benign tremors interfere
with daily activities, you can be treated with beta blockers, anticonvulsant, tranquilizers
(benzodiazepines), or diuretics (carbonic anhydrase inhibitors).
Both illnesses do cause shaking of the hands but in Essential Tremor the shaking is worse on using the
hands, whereas, with Parkinson's the shake is at it's worst when the hands are resting and, in fact,
Parkinson's Disease can be differentiated by the tendency of Parkinson's sufferers to a pill rolling
motion, that is to say, the movement is compared to rolling a small pill between the thumb and index
finger.

Symptoms of Parkinson’s
Tremor When At Rest
Rigid Movement
Balance Problems
Shuffling Movement When Walking
Forward Stoop
Lack Of Facial Expression
Freezing when Trying To Move

! 26!
120
! !
! ! !!!GENITOURINARY SYSTEM !

Pyelonephritis
A 40 year-old female feeling unwell and had a flu-like illness for the last 3 days.
Task: take history, physical exam, management.

Fever, headache, tiredness, arthralgia, myalgia


Loin tenderness
Intermittent fever
Weight loss

DDx:
- STD  HIV, cytomegalovirus
- Pyelonephritis
- Common cold, URTI, pneumonia
- meningitis

Skin rash  any contact


GIT
Respiratory
Urinary symptoms  ask for frequency, urgency, dysuria, haematuria, loin pain, suprapubic
pain
Sexual history
Travel history (malaria, tuberculosis)
Past medical history
SADMA

O/E:
Neck stiffness
Lymph nodes
Tenderness in the loin (+) knocking pain test, Ballotment in kidney
Skin rash
Genital area

Explanation:
It’s an Upper Urinary Tract Infection, if it’s not treated it’s a serious condition.

Mx:
Admit patient, fluid, analgesia, antibiotics, urine culture.
Amoxicillin and Gentamicin IV for 2 days.
Discharged on Trimethoprim or Cephalexin for 2 weeks.
!
Antimicrobial regimen (for non-pregnant women)
Multiple dose therapy preferred to single dose therapy.
use for 5 days in women (trimethoprim— 3 days).
use for 10 days in women with known UT abnormality.
trimethoprim 300 mg (o) daily for 3 days
or cephalexin 500 mg (o) 12 hourly
or amoxycillin/potassium clavulanate 250/125 mg (o) 8 hourly (preferred agent)or
norfloxacin 400 mg (o) 12 hourly for 3 days (if resistance to above agents proven) Follow-up:
MSU 3 weeks later!

1!
121
! !
! ! !!!GENITOURINARY SYSTEM !

UTI (AMC 95)!


A 40 yo man came with dysuria & urinary frequency which started gradually over the last 3 days. No
Hx of STDs. O/E: afebrile, no positive finding including PR of prostate. MCU & MCS showed large
numbers of leucocytes & bacilli. Urine (+) protein, leucocyte, nitrate. Pt is sensitive to Penicillin.

Task: Dx, immediate Mx. Discuss the condition & answer his Qs. Write prescription.

Explanation:
From the Ix, it’s most likely you’re having a urinary tract infection. Because you don’t have a fever
and loin pain, it’s in the lower urinary tract infection. Lower UTI means bladder, prostate and ureter.

Mx:
I’ll give you some antibiotics to treat your current infection.
Cephalexine 500 mg P.O BD for 2 weeks
or
Trimethoprim 300 mg P.O Once Daily for 2 weeks

Ural to alkalinize the urine 4 times daily


Drink extra fluids.

MCS – start treatment – change according to the sensitivity (Bacilli  E.coli)


After treatment for 2 wks, repeat MSU MCS if it’s treated or if the infection is still going on.

Chlamydia (take the first passing urine)


UTI (MSU  mid stream urine)

It’s unusual to have a UTI in male, I think it’s a good idea to investigate for underlying cause:
- malignancy
- calculus
- prostatic pathology

In child  vesicoureter reflux


Young adult  foreign body & STDs

Ix:
I want to refer you to a specialist (urologist) to do some more investigations:
- KUB X-ray
- PSA (? PR)
- FBE
- U&E
- CT Abdomen & Pelvis
- US of kidney, ureter, bladder
- Cystoscopy

Where does this bacteria come from? Coming from down below, they are found in the back
passage, probably come to your urinary tract from down below.

Is it the same as what my wife gets? Yes, it’s the same

Do you think that I have something seriously wrong? Not necessarily

Could it occur again? Yes

2!
122
! !
! ! !!!GENITOURINARY SYSTEM !

Post Operative Oliguria (AMC 102)


A 40 year-old female 6 hours after laparoscopic surgery due to perforated peptic ulcer.

Task: physical examination, management & write down any fluid required.

Assess the patient


GA: Assess dehydration level (ask pt if she’s thirsty or not)
Any drainage tube  nasogastric tube, catheter
VS: T 37.4, P 90, RR 20, BP 120/74 (postural drop), SaO2 99%
CVS and lung  signs of heart failure, JVP
Abdomen
- Any bleeding from the wound
- Feasible mass
- Tenderness
- Bladder – if cannot hear anything  percussion, bladder scan
- PR
- Ask pt if she wants to void or not

Can I have a look at the fluid balance chart


Fluid input for the last 6 hours – 400 ml (IV)
Urine output 50 ml
Nothing by mouth
Any nausea, vomiting for the last 6 hours
(Fluid loss can be from temp, tachypnea, nasogastric tube, vomiting, diarrhoea)

Put catheter in but if bladder is empty DON’T PUT CATHETER (perhaps it’s just lack of fluid)  do
the fluid challenge
If bladder is empty, risk of infection is very high

Fluid challenge (to see pre-renal, renal or post renal)


- IV access (cannula) with 16 or 18 gauge
- Normal saline 0.9% - volume 1 Liter stat IV
- 1 Liter 30/60 (1 Liter over 30 minutes) which means 500 ml in 15 minutes
(eg NS 4/24 means 1 bag of normal saline in 4 hours)
- Reassess patient in 30 minutes (BP, pulse, temp, urine output)
o If there’s urine  continue IV fluid  write NS 1 L V 4/24
(Need to rehydrate for normal urine output then give maintenance 2.5 L)
o If no urine  bladder scan (maybe something in the bladder), perhaps kidney failure, put
catheter  if just some drips or no urine call registrar, but if it’s good urine, continue fluid
o Signs of overload  SOB, increase JVP, crepitations on the bases of the lungs

Pre-renal oliguria due to lack of input or IV – continue fluid

Mx:
I’d like to do a fluid challenge and give:
N. Saline 1 L IV 30/60 ---if there’s urine 150 mL
Give the next bag:
N. Saline 1 L IV 3/24
Re-assess in 1 hour
Assess level of dehydration, urine output
Continue maintenance fluid (2.5 L) until she is able to take fluid orally
Give N. Saline 1 L IV 8/24

3!
123
! !
! ! !!!GENITOURINARY SYSTEM !

Haematuria (AMC p 222)


A 59 year-old male presented to your GP clinic complaining of colour changes of urine.

Task: manage the patient.

DDx:
Cancer Bladder cancer (70%), renal cell carcinoma, prostate cancer
Congenital renal abnormalities  Polycystic Kidney Disease, horseshoe kidney
Infection
Trauma
Stone
medication

Hx:
What do you mean by colour change?---red
Is it at the beginning or throughout the end
Constant or intermittent (is it there all the time or does it come and go)
Do you have problem starting or finishing urination (dribbling for prostate)
Any trauma
Recent cold or flu (acute glomerulonephritis)
Fever, Flank pain, Burning sensation.
Any blood thinners? Warfarin, clexane, heparin.
Weight loss, tiredness, night sweat, any lumps.
Any bleeding from any other site, any bleeding disorder?
Is it the first time or not
PHx of kidney stone
SADMA
Occupation (aniline dye)
Family history ---father died of subacute haemorrhage
- kidney stone
- prostate
- clotting problem bleeding disorder
- polycystic kidney problem

O/E:
GA, VS, Heart, chest and abdomen
Genital area for any sign of trauma, any discharge or visible blood
PR – look for size, shape, consistency of prostate
Urine dipstick  (+) for blood

Explanation:
You have a painless haematuria, which is blood in your urine. It’s abnormal, we need to do Ix to find
out the cause. I will refer you to the urologist.
- FBE
- Midstream urine  MCS (microscopic, culture, sensitivity)  look for fresh blood, casts
- US
- X-ray KUB (kidney, ureter, bladder)
- If there’s any problem with the prostate  PSA  transrectal ultrasound & biopsy

The urologist might do cystoscopy, he will put a flexible soft tube with a camera on the top through
your penis opening to see the lining of your bladder or inside of your bladder. He might take a piece
of tissue to confirm the diagnosis.

4!
124
! !
! ! !!!GENITOURINARY SYSTEM !

Horseshoe Kidney
A 50 yo female pt presented with sudden onset of lower abdominal pain since 3 days. Urine analysis:
blood (+++), X-ray: no renal stone was found. Helical CT was organised but report was not available.

Task: explain the CT to the pt & the most likely Dx, explain short term & long term Mx.

Is it assoc with cancer


Do I need to tell the doctor if I have an emergency situation
What is the incidence

The CT showed that you have a relatively uncommon condition which is known as horseshoe kidney.
Around 1 in 5000 person is born with this condition. A horseshoe kidney forms if the 2 developed
kidneys are connected to each other at the lower part and grow together. In fact, this adhesion does
not affect the function of the kidneys & about 1/3 of all patient with this condition will have no
symptom at all. However, some diseases are found to be more common among people with horseshoe
kidney than people with normal kidneys.

Approximately 10-15% will cause recurrent problems like UTI & kidney stones and approx 1/3 will
have medical condition like hydronephrosis (cause abdominal pain) which is a dilation of 1 or both
kidneys resulting from obstruction to the flow of urine which is caused in most cases by a contraction
of the junction between the kidneys & the 2 tubes taking the urine to the bladder known as the ureter.
In some cases, hydronephrosis will cause discomfort & pain in the site affected. It might be the cause
of repeated infections & sometimes the patient will have it with no symptoms at all. There are other
problems that could be associated, the horseshoe kidney is located differently than normal, it means
the ureters will be running in a different way.

Horseshoe kidney will more likely have complication such as double ureter, and a back flow of the
urine from the bladder to the kidney known as reflux.

There is no actual treatment to the kidney itself. The treatment is directed to the accompanying
disease such as UTI & hydronephrosis.

Let me assure you that the prognosis is good in the long term & usually the function of the kidney
with regular follow up will not be affected. I’ll refer you to a nephrologist to set up the plan of follow
up & to order investigation if needed. Repeat & serious infection might harm the kidney, you should
be careful to treat it immediately. Unfortunately long-term therapy with antibiotic might be
necessary.

The final warning in patient who’s diagnosed with a renal cell carcinoma, a type of kidney cancer, the
incidence is increased where 45% of tumour is found in horseshoe kidney.

Rarely patient will have undescended testicles (cryptorchidism).

Stone metabolic screening is one of the regular follow up to detect stone. Let me assure you that with
regular follow up it won’t affect the regular life span.

Waring sign: If there is a severe abdominal pain or severe blood in urine & trauma to the abdomen,
you should inform your surgeon or doctor  the kidney will press other structure in the abdomen.

5!
125
! !
! ! !!!GENITOURINARY SYSTEM !

Horseshoe Kidney
ED setting. 25 yo Matthew Ramm presented with 1st attach of renal colic with left sided loin
pain radiating into the groin. His urine was (+) for blood and the working Dx was ureteric
colic. A plain abdo X-ray is done but not reviewed.

Task: discuss X-ray with examiner, arrange Ix, explain Dx and Mx with pt.

Ix: Plain abdo X-ray shows calcific opacities in the region of the left lower renal pole. Note
the reversed axis of the kidneys, which suggests horseshoe kidney!!

6!
! !
! ! !!!GENITOURINARY SYSTEM !

Renal fusion occurs when a portion of one kidney is fused to the other.
The most common fusion anomaly is the horseshoe kidney, which occurs with fusion of one
pole of each kidney. > 90% fusion occurs at the lower poles

Symptoms: Horseshoe kidneys are much more frequently symptomatic than other varieties
of fused and ectopic kidneys. Up to 70 percent of children and adults with this abnormality
will have symptoms, which can include abdominal pain, nausea, kidney stones and urinary
tract infections. Although still rare, cancerous tumors are somewhat more likely to occur in
horseshoe kidneys than in normal kidneys. Blood in the urine, a mass in the abdomen and
flank pain can be symptoms of a kidney tumor.

The condition may increase the risk for:


• Kidney Obstruction - abnormal placement of ureter may lead to obstruction and
dilation of the kidney (hydroneprhosis).
• Kidney Infections - associated with vesicoureteral reflux.
• Kidney Stones - deviant orientation of kidneys combined with slow urine flow and
kidney obstruction may lead to kidney stones.
• Kidney Cancer - increased risk of renal cancer, especially Wilms' tumor, transitional
cell carcinoma, and carcinoid tumor. Despite increased risk, the overall risk is still
relatively low.
• Increased risk of trauma because of its position anterior to the spine.

Mx:
1. Manage the renal calculi as per usual renal colic plan
2. The horseshoe kidney normally does not require any treatment!

MRI

7!
126
! !
! ! !!!GENITOURINARY SYSTEM !

Benign prostatic hyperplasia - TURP


A 60 year old male presented in your GP clinic for an enlarged prostate. He was seen by a
surgeon who’s planning to perform a TURP but the surgeon did not explain the procedure for
the patient. PSA was done and came normal.

Task: explain to patient the procedure and possible complication. Answer the patient’s Qs.

Mr X, I know you’re here to discuss about the procedure of TURP. It’s the golden standard
for benign hypertrophy of the prostate. This procedure will give you back the good stream &
reduce the symptoms associated with enlarged prostate.

As you know, the prostate is a walnut size gland located around the opening of the bladder
surrounding the urethra and it produces substance that make up small part of the semen.
When enlarged it caused weak stream, night urge, difficulty in starting urination, dribbling at
the end. It’s a very common condition as nearly every men over 45 have some degree of
prostate enlargement. Very common in men >55 years of age (50% of men will have it).

This procedure will be performed under general or spinal anaesthesia. This will be done
through a tube with a source of light and a magnifying lens with a cutting loop at the end & a
diathermy part as well included through the tube to stop any bleeding which will be passed
through the urethra & the obstructing part of the prostate gland.

Before this a cystoscope, which is a scope to pass before the main one of the TURP to allow
direct visualisation of the bladder to detect any abnormalities. Bladder neck will be excised
during the procedure & the resectoscope will remove/excise the surrounding prostate or
enlarged prostate part around the urethra.

TURP is an easy procedure in the hand of the skilled surgeon & usually has very minimal
complication but as any other procedure, it has some possible complications:
1. General anaesthesia or specific ones like post operative haemorrhage or bleeding as the
prostate is a very vascular organ
2. Clot retention due to the bleeding
3. Urinary tract infection is common after this procedure
4. Low incidence of urine incontinence
5. Impotence
6. Unfortunately retrograde ejaculation also is one of the drawbacks due to the excision of
the bladder neck, one of the expected outcome.

A catheter will be left in place for 1-2 days, max hospital stay is 3-5 days. After hospital stay
you’ll gradually improved & within 4 weeks time you can go back to work and normal day-
to-day activities.

What about my sexual activities?


The procedure does not affect your desire but satisfactory ability is usually in 1-2 months to
having normal intercourse.

8!
127
! !
! ! !!!GENITOURINARY SYSTEM !

Benign Prostatic Hyperplasia (AMC 67)


A 55 year-old man come with generalised weakness, lethargy and nocturia.

Task: take history, examination, explain the condition, manage.

Hx:
Nocturia
- How long have you had these symptoms?
- How frequent do you have to go to the toilet at night?
- Is it becoming worsen?
- How much water, coffee do you drink?
- Have you experience any difficulty in urination?
Urgency after sitting long time
Difficulty starting
- Do you have any dribbling?
- Do you have any sense of incomplete emptying?
- Any burning sensation or pain during passing the urine?
- What’s the colour or urine, any blood, any smells?
- Any fever, malaise, chills and rigors? (to exclude prostatitis)
- Have you noticed any discharge from the urethra with or without urination?
- Any history of UTI, STD in the past
- Any scrotal or abdominal pain, back pain
- Any problem with intercourse?
Weakness & lethargy
- Would you tell me more about it?
- Where do you feel weakness, generalised or in specific area
- How’s your mood, sleep, Stressor (depression)
- Loss of appetite or wt (cancer)
- Did you experience any blackout, dizziness
SADMA – medication – diuretic
PHx of similar condition, or DM, thyroid problem, any surgery
FHx of similar condition or having a tumour
Travel history

O/E:
GA; VS; Chest, heart, abdomen
Anal area and PR exam
- Inspection – normal
- PR prostate – one lobe or 2, enlarged with medial sulcus intact, no abnormal nodule, soft, not
rubbery in consistency

Prostate is a gland situated below the bladder which secretes the fluid which nourishes the sperm. If
it’s enlarged it will obstruct the urethra and cause urinary symptoms
- Urine microscopy and culture
- Renal function test
- PSA if the level is abnormal proceed to prostatic biopsy
- Transurethral USG

This is a common condition in man after 50 After 55 yo, 50% will have BPH, 80 yo 80%
Refer to specialist, he will try medication
- to decrease the size of prostate (alpha reductase inhibitor- fenasteride)
- to ↓ the contraction of bladder and to ↑ the contraction of sphincter (alpha blocker – prasozin)
- if fail, surgical option TURP

9!
128
! !
! ! !!!GENITOURINARY SYSTEM !

Prostate Cancer
A 68 yo man with prostate cancer Gleason score 5 within prostrate, no spread, after surgery came to
you as a GP for explanation.

Explain prostate and the procedure.

The surgeon took some parts of your prostate and analyse under microscope.
It showed you’ve got a cancer, I know it’s not a good news but let me reassure you that the cancer is
only within the prostate. Very good news that it has not spread, and you’ve got here written Gleason
score not high, but we need to do some blood tests which is the tumor marker to follow-up the
progression. You’re in good hands because it was picked up early
It’s a very slow growing, you will need regular follow-up every 3 months and we’ll keep an eye on
your PSA level. (For young pt prostatectomy, chemo is not sensitive)
If it increase we will send you to the surgeon for other options such as surgery. It has some
complications. Other options may be general or local radiotherapy.

Gleason score is from 2 – 10


- 2-4  low
- 5-7  intermediate (within prostate)
- 8-10  aggressive (high score)
Gleason grade is from 1-5  according to morphological changes in the tissue
Gleason score is the total score from 2 separate areas of the prostate, eg:
- from area 1 = Gleason grade 1
- from area 2 = Gleason grade 3
 Gleason score 1 + 3 = 4

Outcome is good, because it has not spread


How to confirm?
We’ll do blood test and check the tumor marker Prostate Specific Antigen (PSA)
PSA is a very sensitive indicator for progress of disease or response to Tx, but not a screening test
PSA <4 – normal
PSA >20 – spread (advanced disease)
PSA >30 – most likely distant metastasis to the bones, liver

Can the cancer spread Doctor, where?


It can spread locally to the lymph nodes or to the bones and liver by blood.

BPH  TURP
Prostate cancer is a very slow growing:
1. Wait & watch
2. Observation & follow-up every 3 months – PR examination & PSA
3. PSA is a very sensitive indicator  if it’s raised  you’ll need surgical review and discuss
possible surgery which is radical prostatectomy  80% cured but also has some complications
like:
- urinary incontinence (10%)
- impotence (30%)
4. Other options
- Radiotherapy or brachytherapy (some kind of radiotherapy when we put radioactive seed into
the prostate)
- Hormone androgen deprivation therapy  flutamide (only in advanced stage with multiple
metastasis)  complication: feminisation
- Bilateral orchidectomy

10!
129
! !
! ! !!!GENITOURINARY SYSTEM !

Seminoma (AMC 108)


GP setting. A 28 yo university student who recently had his testis removed 3 weeks ago
because it was a tumor of the testis. The surgeon told him he would need further tests and
follow-up. He is moving interstate and you’re his new GP. You’ve his medical detail and
picture, and just finished examining him. He has a well-healed inguinal incision, the
remaining testis feels normal and other systems are normal. He is concern about his future
Mx.

Task: discuss his condition, follow-up and answer his questions.

Explanation:
From history & exam and the picture of the tumour sent by your specialist, I can tell you that
this tumour is a malignant tumour of the testis and its feature looks typically of one type
which is the commonest type known as seminoma. This type is commonest between the other
3 types of testicular tumor: seminoma, non seminoma (teratoma) and mixed.
The operation that was done is the best as you had removal of the testis and the cord with best
outcome. The incision that was done through the inguinal area it the best approach to prevent
spread of nasty cells to other lymph nodes. Usually this tumour travels to lymph node in the
tummy if the incision was done through the scrotum this opens the door for implantation of
the tumour cells to the skin of the scrotum and possibly spread to other inguinal lymph nodes
which means more possible spread to areas which was not supposed to have this nasty cells.

But my uncle had an incision in the scrotum.


Most likely he had another condition like collection of fluid around the testis known as
hydrocele.
We need to refer you to a radiation oncologist who will decide for you the next suitable
treatment and follow-up steps which usually includes abdominal radiotherapy as this tumour
is radio-sensitive which may be combined with chemotherapy if needed. This adjuvant
therapy after surgery increases what is known as survival rate of 5 years to up to 85% and
more. For future follow-up, a tumour marker test known as alpha-feto protein or beta HCG
will be done to detect any recurrence. A CT abdomen and chest if not done before the
operation will be arranged to detect the tumor staging and future management & treatment.
Mr X, early detection can completely cure you & with good follow-up you can have an
excellent outcome.
Before going to the radiotherapy you may donate sperm to the sperm bank for future fertility.
You have to be taught for self testicular examination to detect any lump.
In addition Mr X, there is also a plastic reconstruction surgery where they can have an
artificial testis.

Which chemotherapy if needed?


Four courses at 3 weekly interval of:
- Bleomycin
- Etopozide
- Cysplatin

Advice to do self testicular examination monthly and if u find pain,lump,contact your GP.
risk to another testes is 5%.Follow up should be at least 10 years.

11!
130
! !
! ! !!!GENITOURINARY SYSTEM !

Hydrocele
A 32 yo man presented with painless swelling of the scrotum.

Task: take history, investigation and management.

DDx:
Hydrocele
Hernia
Varicocele
Tumor
Orchitis
Trauma

Hx:
When did you first notice it?
Is it the same size since you noticed it?
Any history of trauma?
Any history of recent infection?
Do you have any fever?
Associated pain and discomfort?
Do you have any other swelling or any lumps or bumps anywhere in your body?
Have you ever been diagnosed with hernia before?
Are you sexually active?
Did you have any STD before?
How is your general health?
How is your appetite?
Have you lost weight?
Any family history of similar condition
Are you taking any medication for a long time?
What is your occupation?
Any possibility of radiation exposure?
Do you smoke?
SADMA

O/E:
GA; VS ; Lung, Heart
Abdomen: any mass? any hernial orifice intact or not?
Scrotum:
- Skin changes
- Any bruise?
- Temperature
- Palpate the swelling: soft or hard? Is it possible to get above the swelling?
- Cough impulse positive or not?
- Any change in size in lying down or not? Reduce the swelling when lying down?
- Transilluminate or not?
- Tenderness present or not?
Urine dipstick

12!
! !
! ! !!!GENITOURINARY SYSTEM !

Explanation:
From history and examination, your condition is most likely to be hydrocele, where there is a
collection of fluid in the layer of the cover of the testis.
Hydrocele at your age is most probably secondary due to trauma, infection or tumor. It can be
benign or it can be a nasty condition as well.
So we need to do some investigations to rule out and find out the cause.
I will arrange for the blood tests.

Ix:
FBE, CRP
Tumor marker - B hCG, CEA (carcinoembryonic antigen), Alfa Feto Protein
US (No aspiration because if due to malignancy, it may spread to the skin)

If it is a benign condition, we do not need to do anything to the hydrocele. If it is getting


bigger and very much uncomfortable the management can be decided between you and the
surgeon.
If it is a malignant condition they will take the testis out with or without the radiation or
chemotherapy.
I will refer you to the surgeon. The best management will be decided between you and the
surgeon.

!
!

13!
131
! !
! ! !!!GENITOURINARY SYSTEM !

Ureteric Stones (AMC 90)


A 45 year-old man complaining of abdominal pain, presented to ED.

Task: take Hx, PE and MX

Do you want any pain killer?


PAIN Q
Have you had this before----last week about half an hour, it disappeared by itself
System review: N/V, SOB, chest pain, bowel motion, urinary symptoms
Medications
Any medical condition
Family history of kidney stones
Social history
SADMA
Diet, drink less water, eat specific diet

O/E:
GA: normal

Ix:
Urine – dipstick, MC + S
CT Scan
US
Examine the stone
U & E, calcium
Parathyroid hormone level
Hyperuricaemia

From CT scan there is a stone, in the ureter which is the connection between kidney and
bladder. It’s most likely small, less than 5 mm which can be managed conservatively. I’d like
to call the urologist who will also have a look & will decide which is the best treatment. If it’s
< 4 mm, most likely we send you home with a strong painkiller and drink a lot of water. If it
doesn’t relieve the pain after > 48 hours or worsen, please come back. Low in the tube, we
can do endoscopic removal or ultrasound destruction. If big, remove by open operation.

Do I need to be admitted to the hospital?


No, not at this stage
Will the pain come back?
Possibly, if the stone is still in the ureter
Why did I get the stone?
How will I know if I have passed the stone?
You will strain your urine
What happens if the stone does not pass?
An instrument may have to be inserted to retrieve it (endoscopic removal).
When do I see you again?
Follow-up in a couple of days.

14!
132
! !
! ! !!!GENITOURINARY SYSTEM !

Mumps Orchitis Counselling (AMC 8)


Your next patient in your GP clinic is a 25 yo man who’s c/o fever and testicular pain. He
visited you 5 days ago because of painful swelling on the side of his face. You diagnosed him
as mumps 5 days ago. Left testis enlarged, twice the normal size, tender and hot.

Task: advice about the diagnosis, treatment, prognosis and answer the patient’s questions.

Son had mumps recently

Because you had mumps 5 days ago, this condition is most likely called mump orchitis,
which is an infection of the testis. It’s one of the common complications of mumps infection,
occur 20-30%. Usually it starts 3-4 days after mumps and resolves within 1 week time. Very
painful, I understand that.
To ease the pain :
- Scrotal support
- Elevation of the scrotum will ease the pain (because of the gravity)
- Rest
- Good painkiller like Paracetamol and Codeine
- Increase oral intake especially fluids
- Heat application
- Light diet
- Review

He’s still infectious to kids but to adults less likely.


Usually resolves within 1 week time, there’s a 50% chance to get testicular atrophy
(permanent) - size may get smaller. Fertility will not be affected provided the other testis is
normal. No chance of increase malignancy.
Each testis has separate blood stream and sac, the other testis will not get affected.
Cause by paramyxovirus that spread by droplets, incubation period 2-3 weeks. Swelling last
for 2 weeks.
Patients are infectious 6 days before and 9 days after the mumps ocurs.
(Child should be excluded from school for 9 days)

Complications:
- Orchitis
- Aseptic meningitis

Rare complications:
- Encephalitis
- Pancreatitis
- Arthritis
- Oophoritis

15!
133
! !
! ! !!!GENITOURINARY SYSTEM !

Renal Failure
Mr Smith, 68 yo, came to your GP clinic as he’s feeling unwell for a while. His BP has been
hard to control. He works as a storeman but has been on sick leave for 1 month. You referred
him to physician as he has been retaining fluids. The physician & you diagnosed renal failure.
According to the specialist he will need dialysis. He missed the return visit to the specialist
because he had to see the dentist urgently for painful tooth abscess. He comes to see you
today to find out the results of his tests. Creatinine very high, GFR very low.

Task: explain to him the diagnosis & the available option.

Explore the cause


Kidney failure caused uncontrolled BP, maybe caused by drugs
Mr Smith abused analgesics in his younger days, he took Panadeine, Oxycontin

Explanation:
What did the specialist told you---renal failure
How much do you know about it---my cat has renal failure & the vet shot her down
I’m sorry for your cat but your condition is different
Your kidney is not working as it should be
How do you feel about it (explore the patient’s knowledge & attitude about kidney disease)
Explain his condition in easy terms.
Blood supply – kidney not working good – a lot of waste products return to the bloods – less
urine – swelling – BP high – it may continue more if not treated – anemia, uremia,
encephalopathy & death.
It’s risky

Explain about frequent blood tests (on and off) hoping his kidney function improves
He’s worried HIV & Hepatitis B ---more blood tests such as baseline including HIV,
Hepatitis B & C

He needs frequent follow-ups


A common condition, I know it upsets you
I’ll refer to the specialist for a dialysis
Follow-up regularly
One of the risk is catching infection
We clean the machine regularly but that’s one of the risk

Offer to inform his family


With whom do you live
How much they knew about your condition---my wife doesn’t know
I can explain to your wife & family

What is expected from the candidate?


What blood test should be done
Complications of the shunt & renal failure
Family involvement
Explain about kidney transplant
Diet & nutrition

16!
! !
! ! !!!GENITOURINARY SYSTEM !

Let patient understand the diagnosis


What’s the causes of renal failure
Hypertension
Medication
Unknown

What’s the risk of renal failure: check Murtagh


- HF
- Resistant
- Electrolyte disturbance
- Anemia
- Vit D def & Ca

Needs regular visit for the dialysis


Regular frequent blood tests
Need to check medication

AV shunt:
- bleeding & infection
- improves renal function & get rid of the fluids

Plan for patient:


- Patient FU
- Regular dialysis
- Blood tests
- Control BP
- Healthy diet
- Exercise if he/she can
- Low salt or no salt
- Smoking & alcohol cessation
- Depression & psychological effect
- Vaccination

Financially what will he do  Centrelink

17!
134
! !
! ! !!!GENITOURINARY SYSTEM !

Erectile Dysfunction
Mr Smith, a 60 year-old man came to your GP clinic for “Levitra”.

Task: take history, examine, investigate and outline management plan.

Injury to the spine 15-20 years ago


CT spine OK
Hypertensive
DM more than 20 years – Diabex SR 1 tablet/day
Taxi driver
Any surgery---no

GA: well, not in any distress, not pale, overweight BMI 32, no dysmorphic features
VS: normal
Heart, Chest, Abdomen normal
PR: normal
Office test: urine dipstick, BSL normal
FBE: normal

DM well controlled

Drug related
Beta blocker, thiazide
Nexium, H2 antagonist (GORD)

Test prolactin, thyroid hormone, testosterone, LH


On Nitrates & past history of heart disease

Patient with erectile dysfunction


- Psychogenic - depression
- Neurogenic – multiple sclerosis (parasympathetic)
- Vascular – intermittent claudication
- Diabetes
- Hormonal
o Decreased thyroxin
o Decreased androgen
o Increased prolactin
- Drugs: beta blocker, thiazide, alcohol, cocaine, smoking, anti depressant, anti psychotic, H2
antagonists
- Aging
- Unknown

HISTORY
Patient may have erection but can’t maintain it or no erection at all
Morning erection – is a good sign
How long have you had this erection problem
Have you seen a doctor before
Did they investigate you
Any medication before?
Relationship with his partner
All medications especially the nitrates
Past history of heart problem, stroke, hypertension

18!
! !
! ! !!!GENITOURINARY SYSTEM !

Eye issues (retinitis pigmentosa – contraindicated for Viagra)


Past history of ischaemic heart disease 10 years ago but now OK, no SOB, no chest pain, can go to the
hill without any stop – no contraindication for Viagra
What’s his condition now – do ECG and stress test (if in doubt)

EXAMINATION
Check pulse everywhere
Neurologic examination
PR
Check genitalia, cremastric reflex (scratch in the inner thigh, his testicles will go up)

INVESTIGATION
Testosterone
Prolactin
LH
BSL
TFT
LFT
UEs
Electronic computerised test to different between psychogenic from organic

TREATMENT
Prostaglandin E (vasodilator) injection intracavernous, if erection improves – psychogenic or
neurogenic
If not improved – vascular cause
Include partner
Psychogenic – CBT and psychotherapy
Blood test hormonal problem – treat
Testosteron low (low androgen) – give hormonal oral, IM, S/C
Prolactin high – give Bromocriptin

CONTRAINDICATION
Phosphodiesterase inhibitors (Viagra 25-50-100, Cialis 5 mg everyday)
- Recent stroke
- Recent MI
- Unstable angina
- Recent intake of nitrates

MANAGEMENT
- Psychogenic
- Hormonal replacement
- Idiopathic and no contraindication try Levitra, Viagra, Cialis
- May need surgery – vacuum constriction, intraperineal injection

One hour before intercourse/sexual activity, take 1 tablet.


Sexual stimulation is needed before sexual activity
Side Effect:
If erection >4hours  ED (give general anaesthesia)
Visual changes – blurred vision
Headaches
Gastrooesophageal reflux

19!
135
! !
! ! !!!GENITOURINARY SYSTEM !

Urethritis (Chlamydia)
David, 21 years old came to your GP practice. He’s a new patient, seemed to be anxious. He
has been having 3 weeks history of painful micturition.

Task: take history, ask examination finding, investigation, diagnosis and management.

Hx: (Assure patient SENSITIVE & CONFIDENTIAL)


When did it start---3 weeks ago, suddenly
Painful micturition, when I wee, start and after
Any change in color of urine---no, not blood in urine
Nocturia---yes 3-4 times
Lower abdominal pain on and off
Other associated symptoms, fever, N/V, constipation, diarrhoea ---no
Discharge from penis
Past history of medication
What’s your job
Sexual Hx 3rd girlfriend for 6 months
Neither had STD before
Feeling tired
Sex preference, male or female—not homosexual
Which kind of sex, oral, rectal vaginal
Protected or unprotected
Have you had sex overseas -- no
Any problem other than the micturition, erection, premature ejaculation, fail to maintain
erection
SADMA

O/E:
GA: looks anxious, average body size
Testicles normal
Epidydimis normal, not tender
No varicocele
No abnormalities in penis
No visual sign of STD
Urine dipstick normal

Explanation:
David, you had urethritis, could be UTI or STDs.

Midstream urine sample analysis to see if he has UTI together with STD (coexisting).
First pass urine – PCR for Chlamydia & Gonorrhoea
Blood test – screen for:
- Hepatitis B,C
- HIV (with consent)
- Syphilis

HIV unlikely (common in homosexual), Syphilis less likely but should be ordered if IV drug
user, have sex overseas, has risk factors

20!
! !
! ! !!!GENITOURINARY SYSTEM !

Young people age <25 who has changed sexual partner, should be tested annually for
Chlamydia even if asymptomatic.

Genital herpes if there is rash (swab from fluid of the vesicle), vaginal swab for PCR

David, I’m sorry that you have Chlamydia. It’s an STD, comes through unsafe sex. Contact
partners are difficult to find.
Chlamydia has 30-50% transmission rate per act of unprotected intercourse.
David’s recent partner has >70% chance to have the infection.
According to the Australasian Contact Tracing Manual, at the time of diagnosis, sexual
contact should include from the past 6 months.
Contact tracing is voluntary and required the patient’s cooperation.
Chlamydia is a notifiable disease within 5 days of receiving the result by lab and the doctor.

Tx:
Antibiotics
Rapid treatment is essential to avoid complication
Azithromycin 1 gram oral (2 tablets of 500 mg together) single dose or
Doxycyclin 100 mg oral 12 hourly for 7 days.

Advice David to avoid sex during treatment. His partner should be treated at the same time to
stop reinfection.
I need your cooperation to bring your partner to come and see me, I need to treat her too.
Verbal approval from David to talk about his STD to his partner (take consent).
Give David request to repeat urine test in 6 weeks:
- Urine MCS
- Urine PCR
If still positive, repeat treatment

You have the obligation to contact David (Recall system) send mail & contact by phone 
document this.

Complication:
Very rare  infertility
Prostatitis
Spreading the risk to girlfriend & affecting her fertiligy

My girlfriend is cheating me?


I don’t think so (don’t be judgmental), we don’t know exactly what’s happening, our aim
now is to treat you.

21!
136
! !
! ! !!!GENITOURINARY SYSTEM !

Renal Stone!
ED setting. 35 yo Mr. Smith presented with a 2nd episode of severe R sided flank/groin pain.
The pain came on suddenly, was 10/10, reduced to 6/10 after a morphine injection.

Task: Brief Hx, Ix, Dx and DDx, explain Mx and give advice to the pt.

Hx & Ex:
A very similar episode happened 2 months earlier which resolved spontaneously and he did
seek help. Today the pain started in the Rflank and radiated into the right groin and testicle.
VS were normal, abdomen was tender on the affected side (as palpation increases pressure in
the already-distended ureter), but peritoneal signs (guarding, rebound, rigidity) were lacking.

Ix:
1. urine dipstix test for blood
2. catch stone for identification (sieve, sock; sent to the laboratory for crystallography: ca,
oxalate, uric acid)
3. MSU (?infection)
4. radiological investigation (size and location of stone / 5 mm border line)
• KUB (not very diagnostic, often confusing, about80% radio-opaque – calcium oxalate
and phosphate, whilst uric acid and cysteine radio lucent!)
• U/S Ultrasound  detect any significant hydronephrosis, cannot be used to find small
stones < 5 mm and does not help in the evaluation of kidney function.
• Spiral / helicalCT (noncontrast) of the abdomen and pelvis
• IVP: the main advantage is the clear outline of the entire urinary system pick even mild
hydronephrosis and show nonopaque stones as filling defects. !
5. Blood for WCC, U+E, uric acid and calcium (be mindful of hyperrcalcaemia secondary to
primary hyperparathyroidism / parathyroid adenoma!).

DIAGNOSIS: URETERIC CALCULUS / RENAL COLIC


Urinary calculi are solid particles in the urinary system. They may cause pain, nausea,
vomiting, haematuria, and, possibly, chills and fever from secondary infection. Diagnosis is
based on urinalysis and radiologic imaging, usually noncontrast helical CT. Treatment is with
analgesics, antibiotics for infection, and, sometimes, extracorporeal shock wave lithotripsy or
endoscopic procedures.
Eighty percent of patients with nephrolithiasis form calcium stones, most of which are
composed primarily of calcium oxalate or, less often, calcium phosphate. The other main
types include uric acid, struvite (magnesium ammonium phosphate), and cystine stones.

General risk factors include disorders that increase urinary salt concentration, either by
increased excretion of Ca or uric acid salts, or by decreased excretion of urine or citrate.
Urinary calculi may remain within the renal parenchyma or renal pelvis or be passed into the
ureter and bladder. During passage, calculi irritate the ureter and may become lodged,
obstructing urine flow and causing hydroureter and sometimes hydronephrosis. Common
areas of lodgment include the ureteropelvic junction, the distal ureter (at the level of the iliac
vessels), and the ureterovesical junction. Typically, a calculus must have a diameter > 5 mm
to become lodged. Calculi ≤ 5 mm are likely to pass spontaneously.
On examination, patients may be in obvious extreme discomfort, often ashen and diaphoretic.
Patients with renal colic may be unable to lie still and may pace, writhe, or constantly shift

22!
! !
! ! !!!GENITOURINARY SYSTEM !

position. The abdomen may be somewhat tender on the affected side as palpation increases
pressure in the already-distended ureter, but peritoneal signs (guarding, rebound, rigidity) are
lacking.

DDx:
• AAA (even with haematuria when the AAA presses on the ureter and causes bleeding!)
• Pyelonephritis (fever, leukocytosis, pyuria)
• Renal abscess (chills, fever)
• Appendicitis
• Herpes zoster
• Ectopic pregnancy
• Endometriosis
• DRUG SEEKING!

Mx:
1. ANALGESICS:
a) NSAIDS orally or iv. (ketorolac)
b) Narcotis (morphine, fentanyl)
2. ANITEMETICS (eg maxolon, ondansetron)
3. FACILITATING CALCULUS PASSAGE:
a) α-receptor blockers
b) Ca channel blockers (smooth muscle relaxation)
c) high fluid intake although not in the acute pain phase
4. REFERRALto urologist if stone not passed within 48 hours for possible intervention like:
a) Lithotripsy
b) ureterscopy and basket removal
c) open surgery
d) stents
A urologist must know the exact size, shape, orientation, radiolucency, composition, and
location of the stone and must know about overall kidney function, the presence of any
infection, and other clinical information before making the decision.
5. PREVENTION
In a patient who has passed a first Ca calculus, the likelihood of forming a 2nd calculus is
about 15% at 1 yr, 40% at 5 yr, and 80% at 10 yr. Recovery and analysis of the calculus,
measurement of calculus-forming substances in the urine, and the clinical history are needed
to plan prophylaxis:
a) Hypercalciuria  thiazide diuretics to lower urine Ca excretion, ↑ their fluid intake. A
diet that is low in Na, high in K and low in Ca is recommended.
b) Hypocitruria  K citrate enhances citrate excretion, normal Ca intake is recommended
c) Hyperoxaluria prevention varies. Patients with small-bowel disease can be treated with a
combination of high fluid intake, Ca loading (usually in the form of Ca citrate 400 mg po
bid), cholestyramine, and a low-oxalate, low-fat diet. Hyperoxaluria may respond to
pyridoxine 5 to 500 mg po once/day, possibly by increasing transaminase activity,
because this activity is responsible for the conversion of glyoxylate, the immediate
oxalate precursor, to glycine.
d) Hyperuricosuria, intake of meat, fish, and poultry should be reduced. If the diet cannot
be changed, allopurinol 300 mg each morning lowers uric acid production.
e) To prevent recurrent cystine calculi, urinary cystine levels must be reduced to < 250 mg
cystine/L of urine. Any combination of increasing urine volume along with reducing

23!
! !
! ! !!!GENITOURINARY SYSTEM !

cystine excretion (eg. with α-mecaptopropionylglycine or D- penicillamine should reduce


the urinary cystine concentration.

24!
137
RHEUMATOLOGY

Rheumatoid Arthritis Counselling

A 40 yo female who is a player in orchestra c/o stiffness, sausage shape swelling in


her hands. ESR, Rh factor, ANA done which showed that she has early rheumatoid
arthritis. Her mum has RA and is on corticosteroids. Today she came for results.

Task: counsel the patient.

Breaking bad news: Have u come alone, do u want someone to be with you?

From the blood test I’m sorry to tell you that you have a condition called early RA
But I assure you that your prognosis is excellent because we have diagnosed it at an
early stage.
RA is a chronic, inflammatory disease which usually affects symmetrical peripheral
joints. It affects about 3 % of the population, women 3 times more commonly than
men.
The aetiology is unknown but some auto-immune processes seem to be involved.

It is a condition which we cannot cure completely, but we can control the progression
of the disease, so that we can prevent the disability.
Because I understand that your mother has disability due to RA.

It is an autoimmune disease in which the body produce substance called antibody


which attack the joints and other organs. In the joint it will cause stiffness and pain.

At this stage, we won’t start you on medication.


Refer to physiotherapist who will recommend the best exercise for you. Swimming
and hydrotherapy is also needed.

Can I continue my job?


Yes, you can. But when you have an attack you need to take a rest.

Multidisciplinary team: GP, rheumatologist, physiotherapist, occupational therapist

To treat acute attack


- Take a rest at home
- Take simple analgesics - Paracetamol, NSAID

1
RHEUMATOLOGY

- But between the attack, you can do all your activities and do the exercise

How about diet and weight?


No special diet for RA. But it is better to keep ideal body weight for yourself.

How about stiffness and swelling?


Apply hot water bottle or hot pad to the joint as soon as you get up from the bed to
reduce the stiffness. In the evening, you can use cold pad that reduce the swelling.

How to manage the pain?


Paracetamol, Aspirin, NSAID
If not help we can start DMARDS (disease modifying anti rheumatoid drugs)
chloroquine, methotrexate, sulphasalazine, steroid, but this will be decided by
rheumatologist.

Steroid has many side effect


I understand you are concerned about steroids, but we use it only in acute flare up and
use for short term only. Side effects are less likely with short term use.

Give information sheets.


Give referral letter and follow up.

Ix of RA:
• Normochromic and normocytic anaemia
• Elevated ESR
• Rheumatoid Factors (IgM), 85% positive
• Antinuclear antibodies
• Synovial fluid: cloudy, sterile, reduced viscosity and high WCC.
• X-ray: erosion of joint margin (subchondral erosions/‘mouse bitten’), joint space
narrowing, periarticular osteoporosis, subluxation / ankylosing

2
138
RHEUMATOLOGY

Osteoarthritis Counselling
A 72 yo man c/o pain in the R knee especially at night. He was diagnosed with
osteoarthritis. He has been taking some medication but now it does not work.

Task: answer pt’s question, explain the nature of osteoarthritis, and management.

Hx:
Apart from the R knee, is there any other joints been affected? ---only R knee
When were you diagnosed?---1 year ago
Which medication are you taking? Any SE?---Panadol and Voltaren
Is your pain affecting your daily life?---yes, it’s getting worse, can’t squat
Any swelling, fever, color change
Social Hx: occupation, support, SADMA
Any previous injury or surgical procedure to the knee?  Few years ago my doctor
did arthroscopy on my knee. Osteoarthritis aggravated after injury. Pain for 5 years
but diagnosed for osteoarthritis for 1 year
Any significant medical condition
History of fall

Explanation:
I can see that you have been diagnosed with Osteoarthritis, do you know what is that?
Osteo means bone, arthritis means inflamed joint. So it’s an inflammation of the bone
joint. It’s a problem of wear and tear due to excessive use over the years and old
injuries in the affected joint (repeated injuries). It’s a degenerative process that
usually happens with aging.

Draw a picture of the long bone, cartilage, wear & tear between 2 bones which cause
pain.

It’s a controllable disease but not curable. We can control it with good Mx plan.

Mx:
Non pharmacological
- Reduce weight if obese by healthy diet, offer healthy diet chart & exercise
(walking, cycling, swimming)
- Physiotherapy will give strength to your leg muscle and increase ability to bear
weight. You can use knee support and hot pack.

3
RHEUMATOLOGY

- Occupational therapy: Evaluate room situation at home & car – put rail, provide
walking aid, check car
- Social worker – arrange community health, meals on wheels, help with shopping
and normal daily activities
- Support group

Pharmacological
- Analgesic
o Panadol & NSAID
o Codein
o Opiate (morphine and pethidine)
- Steroid
- Glucosamine + chondroitin

If not improved, I’ll refer you to an orthopaedic surgeon for knee replacement (partial
or total).

4
139
RHEUMATOLOGY

Gout (AMC 99)

GP setting. A 40 yo male c/o painful left foot (ankle joint) and swelling (first attack).

Task: Hx, Exam, Investigations, Mx.

Hx:
PAIN Qs
How many joints affected  Do you have pain in any other place
Any fever
General health
History of trauma
Joint pain or swelling  ask for the history of gout  musculoskeletal, arthritis,
varicose veins
Occupation  stand for long?
Travel history  rule out DVT
Family history of similar complain
Any medical condition  hypertension, heart disease, DM
SADMA
Any medication  aspirin & diuretics

O/E:
GA
VS: BP
Systemic review
Local exam  picture

Ix:
FBE, LFT, RFT, Uric acid, U&E, BSL, X-ray of the foot

Explanation:
Gout, is a metabolic condition. The kidney cannot get rid of some substances, which
goes back to the blood and gets deposited in the joints or tissues in crystal form
(deposition of urate crystals). Often runs in families.

Risk factors:
- Alcohol

5
RHEUMATOLOGY

- Dehydration
- Minor trauma
- Diuretics (thiazides, beta blockers)
- Eat a lot of seafood, red meat, beer (purine diet)

How does it get deposited?


Deposited in the tissue- called tophi formation
Kidney or bladders  stone formation
Gouty arthritis
Cardiovascular system

Acute: Red, swollen, tender (NO JOINT ASPIRATION  because it will increase
pain and discomfort)

Immediate Mx
- Stop Aspirin and Diuretics
- Start NSAID: Indomethacin, Ibuprofen
- Increase fluid intake
- Elevate and rest foot for 24-48 hours
- If using diuretics & beta blockers----stop and change to ACE Inhibitors

Preventive
- Adequate water
- Exercise
- Reduce alcohol
- Weight loss
- Decrease purine rich food (seafood)

After acute attack is recovered, start allopurinol


Check uric acid level every 4 weeks
Regular FU with GP
Increased uric acid  cardiac problem  cardiovascular risk assessment done

6
140
RHEUMATOLOGY

Polymyalgia Rheumatica

GP setting. A 60 yo retired account presented with gradual worsening of aches &


pains in his body. He also c/o weight loss, stiffness of his shoulder & neck and back
and tiredness. He had difficulty getting up from the chair.

Task: Hx, PE, Ix, Dx, DDx and Mx

Characteristics  tiredness & weight loss, difficulty getting up from the chair. They
will have hot & slight sweating at night. Joint problems.

Because of the weight loss  check for underlying malignancy (prostate, lung, breast
cancers), shingles, diabetes

Hx:
Weakness – when did it start
Tired – all the time?
When you wake up do you feel tired?
Any problem in sleeping (obstructive sleep apnea  tired)
Mood (depressed)
Fever
Weight loss – what period of time
Aggravating & relieving factors
Assess the severity  does it affect your daily activities?
Does it wake you up at night?
Any headache or change of vision?
Joint problem (when, where, what, which)
- Any limitation of movements
- Any swellings
- Which joints are affected
System review
- Cough?
- Lumps and bumps in the body
- Chest pain
- Waterworks
- Any change in bowel habits
Past medical history

7
RHEUMATOLOGY

SADMA

O/E:
GA
VS
Pain on the forehead
Jaw claudication
Musculoskeletal system
- Range of movements for shoulder, neck, wrist, elbow, knee
- Power, tone, reflex
Lymph nodes
Abdomen
PR ---check for prostate cancer

Explanation:
I think you might have polymyalgia rheumatica which is a condition with an unknown
cause, sometimes caused by autoimmune. Poly means more than 1, myalgia means
pain in the muscle. Rheumatica means pain in the joints. To confirm, I’d like to order
some investigations to make the diagnose before starting the treatment.

Ix:
FBE
ESR, CRP (100 or above is diagnostic)
LFT, BSL

Common medications used and can cause polymyalgia rheumatica  statins.

Tx:
Steroid  start with high dose (40-60 mg) then continue with low dose maintenance
(10-15 mg) for years

- Review after getting the results back


- Refer to rheumatologists
- Frequent follow-up with inflammatory markers (ESR, CRP)
- Reading material

8
RHEUMATOLOGY

DDx:
Myositis  weaknessk, more severe pain, muscle wasting  increase CK, check
antibody, biopsy
Polymyositis
Dermatomyositis  purple discoloration around the eye
Fibromyalgia  everything will be normal (ESR, CRP normal  no inflammation)
but will have skin tenderness (pain on touching the skin)
Chronic Fatigue Syndrome

Red flags
- Sometimes come with temporal arteritis (giant cell arteritis) and underlying
malignancy. Please come & see me if you have severe headaches, blurred vision
(can cause blindness), pain in the jaw while eating (jaw claudication).

• Symptoms of polymyalgia rheumatica include severe muscle aches and stiffness,


unexplained weight loss and fatigue.
• The cause is unknown, but genetics and environmental factors are believed to play
a role.
• Approximately one in five people with polymyalgia rheumatica experience giant
cell arteritis, which is inflammation of the arteries. If left untreated, giant cell
arteritis can cause blindness in one or both eyes.

9
141
RHEUMATOLOGY

Systemic Lupus Erythematosus

GP setting. A 32 yo female c/o 3 months history of joint stiffness & pain in fingers.

Task: take further history, examination, investigation, diagnosis & management.

Confirmation of SLE if there are 4 or more criteria positive from the below 11 points:
- Malar rash
- Discoid lupus
- Photosensitivity
- Painless oral ulcer
- Non erosive arthritis, joint involvement
- Serositis (pleurisy and pericarditis)
- Renal involvement (proteinuria)
- CNS - nervous system involvement – seizure, psychosis, severe headache
- Haematological: haemolytic anemia, leucopenia, thrombocytopenia
- Anti nuclear antibody
- Immunological features: double stranded antibody, lupus antibody, anti
phospholipid antibody

MD SOAP CHAIR
M - Malar rash
D - Discoid rash

S - Serositis
O - Oral ulcer
A - Arthritis
P - Photosensitivity

C - CNS
H - Heart
A - ANA
I - Immunological
R - Renal issues

Positive findings: Malar flush; Congestion in the eyes; Painless ulcer in mouth;
Pericardial rub; Splenomegaly; Rash in chest; Anti DNA antibody (+)

10
RHEUMATOLOGY

Hx:
When did you first notice the pain and stiffness?---3 months ago
Any difficulty to move your fingers?---yes, in all joints of my fingers
Apart from the fingers or other parts of the body too---other joints not involved
Any swelling, rash?---swelling and pain but no redness
Any particular time or all day? (if morning RA, if with activity OA)
Do you feel feverish?
Tired
Rash anywhere else in the body?---rash on both cheeks & chest
Are your skin sensitive to light? (check for photosensitivity)
Any color change in your hands (to exclude Raynaud phenomenon white, blue, red)
Do you have muscle pain? (dermatomyositis)
Is this the first time?---yes
Any chest pain, shortness of breath, cough (pleuritis, pericarditis)
How is your appetite, any nausea, vomiting
Any difficulty of swallowing food (scleroderma)
Any weight change recently (MS)
Any tummy pain
How is your waterworks and bowel habits (haematuria in SLE)
Any muscle weakness, numbness (peripheral neuropathy)
Any bleeding from your skin (purpura)
Any significant past medical condition
Has anyone in your family has similar problem like you?---no
SADMA

O/E:
GA: malar rash present, dark scaly patches on the chest
VS: pulse, BP, temp, RR
Hands:
- Color  any sign of inflammation and deformity (RA – sausage shaped fingers;
OA and psoriasis – distal joint involvement, pitting nail)
- Swelling & pain in both hands (+)
Head: alopecia, hair loss or not?
Eye: scleritis, redness, any kind of ingestion?
Mouth: ulcer  painless ulcer
Neck: cervical lymph nodes enlargement (+)
Chest: any rub, pleurisy present or not (abnormal sound)?
Abdomen: check organomegaly splenomegaly

11
RHEUMATOLOGY

Neurological exam check for peripheral neuropathy

Explanation:
I think you may have some rheumatoid problem, I’d like to do some tests for you:
- Dipstick: haematuria, proteinuria
- FBE: anemia, leucopenia, thrombocytopenia
- ESR, CRP  ESR is increased, CRP usually normal
- LFT
- RFT
- Antibodies
Anti nuclear antibody
Anti DNA antibody (+)
Rheumatoid factor
Anti cardiolyptin antibody
Coagulant antibody
lupus antibody
Anti phospholipid antibody

Explanation:
From Hx & PE, you’re suffering from a condition called Systemic Lupus
Erythematosus.
It’s not uncommon, it’s a connective tissue disease which causes inflammation &
damage of tissue. It can affect lots of systems in our body, bone, skin, lung, heart, and
other organs. The exact cause is unknown but it’s thought to be an autoimmune
disorder. That means the body’s immune function attacks its own tissue, most
common in female.
For treatment, I’d like to refer you to a specialist. Mx depends on the severity:
- First line treatment is NSAID
- Second line - Anti malaria: hydroxychloroquine
- Steroid
- Immunosuppresant – azathioprine, MTX

Usually there is no curative treatment. Treatment is lifelong and controlable.


You’ll need regular follow-up.
There will be relapse & remission.

I’d like to give you contact number for SLE support group and some reading
materials.

12
142
RHEUMATOLOGY

Dermatomyositis

A 17 yo girl c/o rash and oedema on the dorsum of the fingers and upper eyelids.

Task: take Hx, exam finding from examiner, Dx, Mx.

DDx:
Polymyalgia rheumatica (no rash, only weakness & tiredness)
Lupus
Rheumatoid Arthritis

Hx:
When did it start?…..2-3 months ago
Rash on the upper eyelids, which one start first….fingers…change of colour, later
became purple with reddish spot (red oedematous, scaly rash)
Is it itchy?…..yes, sometimes I had to scratch
Does it spread anywhere else?….V-shaped area of the neck, upper arm, elbow, knees
From the beginning oedematous?….no
Do you any other symptoms?…pain in some joints and foot
Any weakness in your body, any pain?….weakness & pain in the muscles of my
shoulders.
Have you experienced any increase in temperature?….low grade fever (under 38
because of inflammation in the muscle) for 1 month at least
Did you seek any help, any medication?….Panadol but doesn’t help
Your muscle power, any decrease, carry things, do activity? ….muscle weakness
especially in my arms and upper legs (proximal muscles)

Which organ can be affected to have malignancy with this condition?


Lung, gastric, pancreas, ovary
Breast, nasopharyngeal

Any weight loss


Any other symptoms like unexplained cough, breast pain, any lumps and bumps, any
indigestion, any pain in your back, unexplained medical condition.

What about your period, regular or any recent change?


Any lumps in your tummy

13
RHEUMATOLOGY

SADMA
Any joint pain (arthralgia in the finger and toe joints)
Any family history of this condition

O/E:
GA: looks tired, in pain, BMI: decreased
VS: T (37.8)
Rash: distribution & nature  Reddish, lilac colour, heliotropic, violet oedematous
scaly rash over the dorsum of the fingers and V-shaped area, over the shoulders, and
on the upper eyelids.
Joint: Arthralgia in the finger and toe joints.
Lymphadenopathy
Neurological exam: decrease in the power & tone of the proximal muscles
Heart, chest, abdomen for any organomegaly or masses

Explanation:
From Hx & exam, you have what’s called dermatomyositis. It’s a connective tissue
disorder characterised by muscle weakness and inflammatory changes in the muscles
and the skin. Derm means skin, myo means muscle and it is means inflammation 
skin and muscle inflammation.

I will refer you to the rheumatologist, he will do some Ix to confirm the diagnosis:
- CK (+) and high
- Rheumatic factor (+) and high
- Antinuclear antibody (+) and high
- Electromyography  changes of the inflammation in the muscle affected
- Muscle biopsy  muscle fibrosis or necrosis
- MRI for the affected muscles detect myositis (active inflammation)

Screening test for malignancy:


Chest X-ray
Abdominal & pelvic ultrasound
Serum tumour marker
Urine analysis
Mammography

14
RHEUMATOLOGY

Treatment:
Steroid 40- 60 mg daily  muscle enzymes will improve before the clinical
symptoms subside  as soon as muscle enzyme improve (check every 1-2 months),
decrease to 15 mg daily.
Some of the cases start to become more weakness  from the beginning have
physiotherapy.
Use splint for the muscle support and refer to physiotherapy

Prognosis
Young without malignancy, she has good prognosis
If there’s malignancy and old age  not good prognosis

What’s wrong with this woman?


A 52-year-old woman presents with a purple rash on her eyelids, purple bumps on the
back of her hands, rash on the neck and elbows with proximal muscle weakness
(difficulty getting up from a seated position and combing hair)

Gottron’s Sign (violet papules on dorsal hands predominantly over joints- individual
bumps called ‘Gottron’s papules’)

15
RHEUMATOLOGY

Heliotropic (purple/violet) rash on eyelids

“shawl” sign = purple patch in the distribution of a ‘shawl’ one would wear

16
RHEUMATOLOGY

psoriasis-like (mild) patches on elbows

Diagnosis:
Dermatomyositis
Dermatomyositis has a presumed autoimmune etiology. There are essentially three
variants of the condition or three variants or closely related manifestations.
1. Polymyositis. This is an inflammatory disease of the proximal muscles. This
occurs in the absence of any type of cutaneous rash.
2. Dermatomyositis. This presents with a violaceous rash, often known as a
poikiloderma rash, that occurs on the posterior neck and upper back known as
a shawl sign, violaceous or heliotropic rash on the upper eyelids, violaceous
patches on the elbows (sometimes mistaken for psoriasis), and a condition
known as Gottron’s sign with violaceous papules occurring in an articular

17
RHEUMATOLOGY

pattern on the dorsal hand. The bumps themselves are known as Gottron’s
papules, and the overall rash on the back of the hands is known as Gottron’s
sign. In dermatomyositis, in addition to the cutaneous rash, there is proximal
muscle weakness secondary to proximal muscle inflammation. Oftentimes,
patients will complain of difficulty in reaching for objects over their head,
combing their hair, and getting up from a seated position such as a couch or
commode. In fact, as a resident, I had a patient whose primary complaint was
he was upset because he was having difficulty getting cans of Campbell’s
chicken noodle soup from the upper shelf in his cabinet. It was his favorite
soup and he had it every day for lunch; however, his shoulders had recently
developed discomfort when he was reaching for the cans. I looked closely, and
he had a very fine rash on his eyelids and on the backs of his hands. He was
eventually discovered to have dermatomyositis.
3. Dermatomyositis sine myositis. This is where the cutaneous eruption occurs
without any proximal muscle weakness.
So, once again, the three variants are dermatomyositis, polymyositis, and
dermatomyositis sine myositis. The occurrence is bimodal. Oftentimes, juveniles
around the age of 10 can get it, with the second peak occurring in adults around age
50. Children also have an association with collagen vascular disease, vasculitis and
calcinosis cutis. Dermatomyositis may also occur in association with other collagen
vascular disease such as lupus erythematosus, Sjögren’s syndrome, rheumatoid
arthritis, scleroderma, and mixed connective tissue disease. Of interesting note, in
adults with dermatomyositis, approximately 25% are associated with internal
malignancy, so dermatomyositis certainly can be seen as a cutaneous marker for
internal malignancy. The types of malignancies seen reflect those commonly seen for
that sex and age. It has been noted that patients with leukocytoclastic vasculitis are at
higher risk for having internal malignancy than adults without leukocytoclastic
vasculitis. The connection between these two is unclear. Children with
dermatomyositis are not at increased risk for developing internal malignancy. The
ratio of women to men with dermatomyositis is 6 to 1. There are also associated
antimicrosomal antibodies reported with the condition that will be discussed in detail
in the accompanying linked articles.

Treatment
In my experience, I have used a combination of prednisone and methotrexate.
Generally speaking, I use prednisone starting at 1 mg/kg, tapering to half that amount
over the first six months and then taper down to zero over additional 1½ to 2 years. I
also use methotrexate low dose as a steroid-sparing agent, and may reduce the

18
RHEUMATOLOGY

prednisone more rapidly with methotrexate in place (10-20 mg/week). I have even
had a few patients who have done quite well on methotrexate alone after an initial
prednisone taper. Additionally, there have been other immunosuppressive agents used
to treat the condition, which will be attached in the attached links, such as high dose
IVIg, cyclophosphamide, chlorambucil, and cyclosporine. In the six to eight patients
I’ve treated with dermatomyositis, one had an occult malignancy and other ones were
managed quite well with prednisone and methotrexate. Diagnosis can be supported by
muscle enzyme studies, cutaneous biopsy showing an interface dermatitis, and EMG.
For additional detailed discussion on dermatomyositis, polymyositis, and
dermatomyositis sine myositis, please refer to the articles linked below.

19
143
RHEUMATOLOGY

Osteoporosis
A 60 year-old female presented to your GP practice. Bone scan showed T score –3.
Calcium level normal, ESR normal.
Task: history and management.

T score
-1 to -2.5  osteopenia (Everything will be normal)
< -2.5  osteoporosis
DDx:
- Osteoporosis
- Secondary metastasis
- Multiple myeloma
- Hypothyroidism
- Renal failure

You’ve done densitometry


We found you have a condition called osteoporosis. In this condition the bones are
like sponges, weak and fragile, that’s why you had the compression fracture.
This is a common condition in your age group and it’s caused by lack of calcium.
Because you have menopause for some years you don’t have the estrogen hormone,
you’re losing mineral, the bone become porous.
Usually asymptomatic until fracture occurs
I’d like to ask you a few questions To assess the risk:
- Diet
- Milk product
- Coffee
- Smoking
- Alcohol
- Medications corticosteroid
- Exercise
- Sun exposure
- Early menopause

Recommend 10 minutes everyday direct sun exposure without sunscreen and


sunglasses

Hx: (to differentiate from other conditions)

20
RHEUMATOLOGY

Ask menopausal symptoms (if positive  give HRT)


Weight loss
Night sweats
Appetite
Vaginal discharge
When was the last Pap smear and mammogram
HRT
(Possible cervical, endometrial, and bowel cancer)
Any changes in bowel habits recently
Any bloods in stool

Can you tolerate cold weather (hypothyroidism)


Have you noticed increased tiredness recently
Have you noticed any changes in your weight

PMHx: Any history of DM, HPT, Kidney or Thyroid disease


FHx of osteoporosis and cancer
Social history

Mx:
Lifestyle modification
- Diet
- Exercise
Medication
- Calcium
- Vit D
- Alendronate (Biphosphonate)
- Raloxifen
Sun exposure
Refer to occupational therapist to assess living condition
Fall prevention program for elderly
- Eye check – decrease vision
- Occupational therapist will ensure safety at home
- Low heel shoes

21
144
RHEUMATOLOGY

Osteoporosis With Secondary Compression Fracture


GP setting. A postmenopausal female complaining of back pain. X-ray shows L4
compression fracture, T12 compression fracture and bone densitometry –3 SD.

Task: explain the diagnosis and manage the case.

I know you got some back pain. How is your pain know, do you need any painkiller?

We’ve done some tests, I’ll explain the results


Your X-ray showed you have a fracture called compression fracture
The second result showed the bone densitometry, how strong is the bone
Your bone is not strong, it’s fragile because there is not enough calcium inside, we
call this condition osteoporosis.
When bones are fragile, the fractures often happen
It’s a common condition in your age group
The most common presentation is bone fracture of spine, arms or legs

Do you have any previous fracture


Ask social history

I’d like to ask you several Qs just to check if you have risk factors for this condition:
1. Diet (ask calsium intake)
- Can you tell me if you like dairy food, cheese, milk
- Do you like salty food
- How often do you eat meat

2. Exercise  how much exercise do you do, how often do you go out for sun?

3. Smoking, Alcohol and coffee

4. Are you on any regular medication (steroid)

5.Early menopause (<53 years of age)


- When was your last period
- Have you been on HRT after your menopause

6. FHx of osteoporosis, anyone in your family with weak bones

22
RHEUMATOLOGY

You have a condition called osteoporosis or we called it bones with holes due to
calsium loss. Very often there are no symptoms until fracture occurs.
Now we need to talk about management, how to deal with your osteoporosis and your
fracture.

For osteoporosis, I recommend you to adjust your diet


Increase calcium intake, I’ll refer you to a dietician
Also you need to take calcium tablets
If it’s possible, reduce salt intake, protein intake, coffee & alcohol

I suggest you to increase your physical activity, you need more exercise
Walking at least 30 minutes twice a day

Stop smoking

Medications – you need:


- Calcium
- Vit D, it helps to keep calcium in your body
- Biphosphonates (Alendronate) to prevent calcium loss from your bones
- Or we can use Raloxifen (SERM - selective estrogen receptor modulator)

Occupational therapy
- I’ll refer you to an occupation therapy, to assess your living condition in order to
prevent further falls
- Fall prevention programme

Fracture Mx:
We need to admit you for bed rest in recumbency position for a week plus analgesia.
When pain is under control, we can use physiotherapy, extension exercises.
Most likely you will go home in a week time.
In some cases, we recommend back braces.

Jennifer, do you have any question? Are you with me? Is it clear so far?

Now, to summarize everything, we will admit you for your fracture.


We need to manage your osteoporosis.
After discharge, please come back for follow-up.

23
145
RHEUMATOLOGY

Osteomyelitis

A 5 year-old boy was brought by his father with a temperature of 39 degrees and pain
in the left tibial hip, just below the knee. Take history, ask for investigation, manage.
Source of infection:
- Direct from trauma
- Indirect from blood

Therapy Flucloxacillin IV and oral until 6 weeks

FBE: high leucocyte, ESR high, X-ray not specific  bone scan showed
osteomyelitis

Movement of knee - restricted


Septic arthritis – cannot move the knee

Complication:
- Abscess
- Sepsis
- Cellulitis

Osteomyelitis means an infection of bone, which can either be acute (of recent onset)
or chronic (longstanding). Bacteria are the usual infectious agents. The two likely
access methods are by primary infection of the bloodstream (including secondary
infection via the blood following an infection somewhere else in the body), and a
wound or injury that permits bacteria to directly reach the bone. In adults, the pelvis
and the spinal vertebrae are most vulnerable, while bone infections in children tend to
target the long bones of the arms and legs. Without treatment, the infection and
inflammation block blood vessels. The lack of oxygen and nutrients cause the bone
tissue to die, which leads to chronic osteomyelitis. Other possible complications
include blood poisoning and bone abscesses. Treatment options include intravenous
and oral antibiotics, and surgical draining and cleaning of the affected bone tissue.
Symptoms
The symptoms of osteomyelitis include:
• Localised bone pain
• Reduced movement of the affected body part
• The overlying skin may be red, hot and swollen

24
RHEUMATOLOGY

• The overlying skin may contain pus


• Spasms of associated muscles
• Unexplained weight loss
• General malaise
• High temperature
• Excessive sweating
• Chills.

Events that can cause osteomyelitis


Bones are infected by blood-borne micro-organisms. In most cases, the micro-
organisms are bacteria such as Staphylococcus aureus, but fungi can also cause
osteomyelitis. Some of the conditions and events that can lead to osteomyelitis
include:
• Bacteria introduced during bone surgery.
• Bacteria introduced by trauma to bone.
• Infection of bone fractures.
• Infection of prosthetic implants (such as an artificial hip joint).
• Infections elsewhere in the body that reach the bones via the bloodstream.
• A primary infection of the blood (septicaemia).

Risk factors
Some of the risk factors that may increase a person’s susceptibility to osteomyelitis
include:
• Long term skin infections.
• Inadequately controlled diabetes.
• Poor blood circulation (arteriosclerosis).
• Risk factors for poor blood circulation, which include high blood pressure,
cigarette smoking, high blood cholesterol and diabetes.
• Immune system deficiency.
• Prosthetic joints.
• The use of intravenous drugs.
• Sickle cell anaemia.
• Cancer.

Acute osteomyelitis
The main categories of acute osteomyelitis include:
• Haematogenous osteomyelitis - primary infection of the blood or infection
from somewhere else in the body is delivered to the bone via the bloodstream.

25
RHEUMATOLOGY

Children are at increased risk. The bacteria are drawn to areas of rich blood
supply, which is why the infection tends to target the growing parts at the ends
of the long bones.
• Direct inoculation osteomyelitis - bacteria are delivered direct to the bone
tissue via surgery or trauma.

Chronic osteomyelitis
An acute attack of osteomyelitis can lead to chronic osteomyelitis, characterised by
dead areas of bone. This condition can fail to respond to treatment and recur for a
long time. In many cases, chronic osteomyelitis is polymicrobial, which means more
than one infectious agent is involved.

Complications
Some of the complications of osteomyelitis include:
• Bone abscess (pocket of pus)
• Bone necrosis (bone death)
• Spread of infection
• Inflammation of soft tissue (cellulitis)
• Blood poisoning (septicaemia)
• Chronic infection that doesn’t respond well to treatment.
Diagnosis methods
Osteomyelitis is diagnosed using a number of tests including:
• Physical examination
• Medical history
• Blood tests
• X-rays
• Bone scan
• Computed tomography (CT) scan
• Magnetic resonance imaging (MRI)
• Bone tissue biopsy.

Treatment methods
Treatment for osteomyelitis depends on the severity but may include:
• Hospitalisation and intravenous antibiotics.
• A long term (four to six weeks or more) course of antibiotics, either oral or
intravenous.
• Pain-killing medication.
• Lifestyle changes, such as quitting cigarettes to improve blood circulation.

26
RHEUMATOLOGY

• Treatment for underlying cause, such as diabetes.


• Replacement of the infected prosthetic part, if needed.
• Surgery to clean and flush out the infected bone (debridement).
• Skin grafts, if necessary.
• Amputation, in severe cases.

Long term outlook


Acute osteomyelitis is easier to treat than chronic osteomyelitis. The earlier the
diagnosis and start of treatment, the better the outlook. If dead and diseased tissue
needs to be surgically removed, the bone regenerates in a matter of weeks. Prevention
of acute osteomyelitis includes proper management of wounds, and prompt medical
attention for infections.
Things to remember
• Osteomyelitis means an infection of bone, which can either be acute or
chronic.
• Bacteria are the most common infectious agents.
• The two likely access methods include primary blood infection or secondary
infection following an infection somewhere else in the body, and a wound or
injury that permits bacteria to reach the bone.
• Treatment options include antibiotics and surgery to clean and flush out the
infected bone (debridement).

27
146
RESPIRATORY SYSTEM

Typical Pneumonia
A 40 yo man c/o generalised aches and pains all over his body & also feeling unwell.

Task: take history, ask exam finding from examiner, diagnoses & DD.

DDx:
Infection
- Pneumonia, atypical pneumonia, TB
- HIV, viral
- Malaria
Thyroid  Hypothyroidism
Autoimmune
- SLE
- Polymyalgia rheumatica
Malignancy
Depression

Hx:
You have aches & pains all over your body. Can you tell me more about that?
When & how did you get it?
Is this the first time?
Pain Qs 
Any other symptoms fever, chills and rigor, rash, joint pain and stiffness?
Cough Qs  how long, dry / bring out any phlegm, color (rusty), blood, amount
Chest pain, night sweats, loss of wt and appetite
Any change of skin color?
Have you noticed any lumps in your body?
Have you travelled recently? Malaysia 3 wks ago (Symptoms started before travel)
Did you take any malaria prophylaxis? ---doxycyclin and finished
Any contact with animals? ---atypical pneumonia
Did you have unprotected sex? (r/o HIV)
Who did you go there with? Did she have the same symptoms?
General health
SADMA

O/E:
GA: ill, tired, weak
VS: T increase, other signs normal
ENT, LN, CVS and Abdominal exam normal
Respiratory exam:
- Inspection: reduced movement on right lower zone
- Percussion: dullness on right lower zone
- Auscultation: reduced breath sound on right lower zone

Ix:
FBE, ESR, CRP
Sputum culture
CXR
Urine analysis

! 1!
RESPIRATORY SYSTEM

Blood film for malaria


Dengue serology
Hepatitis serology
HIV serology (need patient serology)
If suspect cancer  CT chest, bronchoscopy, biopsy

Explanation:
From the Hx & exam, the most likely Dx is pneumonia, it is the bacterial infection of the
lungs. To confirm the diagnosis I want to do some more tests: blood tests and send your
phlegm to the lab, chest X-ray.
Other DDx could be atypical pneumonia, TB infection, nasty growth in your lungs.
If pneumonia is confirmed (CXR showed consolidation), I’ll give you antibiotics.
Amoxycillin + clavulanic acid ± Doxycyclin.

Patient is stable, no need to send to hospital.

CURB 65
• Confusion ------------------------- 1
• Urea > 7 mmol/L ------------------ 1
• RR 30/min or more-----------------1
• BP: DBP < 60, SBP < 90 -------- 1
• Age > 65---------------------------- 1
• TOTAL ---------------------------- 5 (If 2 or more  admit, if >3, urgent)

! 2!
147
RESPIRATORY SYSTEM

Atypical Pneumonia
Scenario 1: A 45 yo female came with generalised muscle weakness & joint pain and feeling
lethargic, wt loss.
Task: history, examination.

Scenario 2: ED setting. A young male presented with SOB for the last 2 days.
Task: full Hx, ask examiner about examination findings, Ix if needed, management.

Atypical Pneumonia Dry cough, flu-like illness, tiredness, weakness, diarrhoea


Chest exam: no consolidation, nothing. CXR: no consolidation, diffuse infiltration.
(Mycoplasma; Legionella; Chlamydia; Coxilla)

DDx:
Lung cause:
- Pulmonary embolism
- Pneumonia
- Pneumothorax
- Pleural effusion
- Lung cancer
- Asthma
- TB
- COPD
Heart cause:
- Angina
- MI

Hx:
When did you start have the SOB? ---2 days ago
Did it start gradually or suddenly?
Is this symptom becoming worsening or improving?
Do you need to wake up at night for the difficulty in breathing? (nocturnal dyspnea)
Are you aware of anything that increase or reduce your SOB?
Any chest pain? Any relation with position?
(Pain in right lower chest, aggravate with deep inspiration)
Do you have cough?
Phlegm Atypical: dry; Typical; productive
How much, color, smelly or not?
Fever, chills and rigors
Any confusion or loss of concentration
Do you feel your heart is racing very fast?
Any previous diagnosis of asthma
Any recent long flight / travel? Recent surgery? Calf pain?
PMHx of clotting problem, asthma
Do you have any contact with TB patient?
Appetite, wt loss
SADMA – alcohol (aspiration pneumonia)
Occupation  air condition office  prone to Legionella; mines, birds
STD (HIV is associated with atypical – mycoplasma)

! 3!
RESPIRATORY SYSTEM

O/E:
GA: confuse, cyanosis, pallor, jaundice; Work of breathing; BMI
VS: BP sitting & standing, SaO2, T increase, RR
ENT, LN
Respiratory exam:
- Inspection: Chest movement reduced on the right lower side
- Palpation: Trachea not deviated
- Percussion: Dullness on the right lower side of the lung
- Auscultation: Bronchial breath sound, R basal crackles (vesicular breath sound is normal)
Heart: check murmur
Abdomen: organomegaly – sometimes splenomegaly

Ix:
- FBE, ESR, CRP,
- U&E
- Arterial blood gas
- LFT
- Blood culture
- Sputum culture & sensitivity
- CXR
- CT chest
- ECG
- Coagulation profile
- Legionella: blood serology & urine test (antigen for Legionella)
- Q fever: mycoplasma serology
- Bronchoscopy if malignancy suspected

Explanation:
Mr John, from history & examination most likely you’re suffering from a condition called
pneumonia. You need antibiotics. I’ll refer you to chest specialist.

Clarithromycin + Amoxycillin
For penicillin allergic  Ceftriaxone +/- Erythromycin

?Tx: Doxycyclin 100 mg bid 10-14 days

If patient has complications:


- Erythromycin IV or
- Erythromycin + Ciprofloxacin or Rifampicin

! 4!
148
RESPIRATORY SYSTEM

Pleural Effusion
A 40 year-old male c/o shortness of breath for 3 months. He has been a smoker with 25
cigarettes per day for 25 years.

Task: relevant history, examination findings, diagnosis and management.

DDx:
Cancer
Pneumonia
Pulmonary embolism
Congestive Heart Failure

Hx: (can offer O2 before taking Hx)


How did it start?
Suddenly or gradually
Associated symptoms: chest pain, cough, sputum, fever, rash, night sweats
Any joint pain? (Atypical pneumonia may present with joint pain and skin rash especially
with recent travel)
Any lumps, loss of appetite and wt?
Bowel habits
Travel Hx
Any TB exposure or contact
General health and PMHx
FHx
SADMA

! 5!
RESPIRATORY SYSTEM

Occupation (eg mining  asbestosis, mesothelioma  cancer)

O/E:
GA: distress, comfortable/uncomfortable
VS: T normal, SaO2 (94), RR a little bit up (22), slight tachycardia, BP normal
System review
- Any palpable lymph nodes
- Lungs
o Inspection: wall movements equal or not
o Percussion: any dullness
o Vocal fremitus: decrease or may be normal (consolidation with fluid  cancer with
pleural effusion)
o Auscultation: breath sound decrease, wheeze & crepitation (-)
- Heart: JVP, heart sounds, any additional sounds, murmur
- Abdomen: liver size, spleen size, signs of ascites, bowel sounds

Ix:
- FBE
- U&E
- Urine dipstick
- Chest X-ray  pleural effusion
- ECG

Explanation:
John, can you see this abnormal part of your lung, there is some fluid in your pleural cavity.
This is abnormal. This your lung & chest wall. Normally just a little bit of fluid to make the
lung movements comfortable. There maybe a few causes for this fluids, may be severe
pneumonia or some nasty growth.
Our next step is to drainage this fluid and do cytology to check if there is any abnormal cells
in this fluid and for infection.

Doctor, I don’t have any fever. I have some pneumonia in the past and took antibiotics but
nothing similar as now?
It’s difficult to say now, after pleural drainage most likely you need to do a CT scan, if there
is any suspicious for lung cancer, we need to do biopsy, take some piece of tissue for
investigations to confirm it.

I’ll refer you to hospital for the drainage and to the specialist for biopsy.

! 6!
149
RESPIRATORY SYSTEM

Pleurisy With Effusion


A 55 yo lady presented with a few days Hx of R sided chest pain. She had a pelvic operation
for sigmoid abscess 6 weeks ago. A few days ago when you examined her, her VS were
normal, there was a reduced respiratory movement, reduced breath sound & dullness on the R
side. CXR showed pleural effusion on the R side (small amount).

Task: take Hx, explain the X-ray finding and discuss further Ix and management.

Causes of pleural effusion:


Transudate:
- Heart failure
- COPD
- Nephrotic syndrome
- Hypoalbuminemia
Exudate :
- Pneumonia
- Malignancy
- PE
- TB
- Trauma
- Pancreatitis
- Oesophageal perforation

Hx:
Are you still having the pain?
Pain Qs
Associated symptoms: SOB, cough, fever, pain in the calf, night sweats
Any precipitating factor? Any relieving factor?
Any weight loss? Loss of appetite?
Have you had any clotting problem before?
Any complication from your operation?
Do you have any long standing period of sedentary or prolong bed rest?
General health? Do you have any heart problem?
Any travel Hx recently? Contact with TB?
SADMA

Explanation:
The chest X-ray showed you have a condition called pleural effusion which is a collection of
fluid between the 2 layers of the covering of the lungs.
There are many underlying causes
So I need to refer you to the hospital to do some Ix
Because you had an operation 6 wks ago, you have an increase risk of clot formation in the
vein which can travel to the lung. So they will do basic investigations such as:
- FBE
- ESR, CRP
- Blood culture
- U&E
- LFT
- CTPA if PE suspected.

! 7!
RESPIRATORY SYSTEM

Depending on the physician, they can do aspiration and send it for cytology, biochemistry test
and culture & sensitivity.

If they find any clot in the lung, they will start with the blood thinning medication.
If it’s due to infection, give antibiotics.
If malignancy, multidisciplinary approach

! 8!
150
RESPIRATORY SYSTEM

Spontaneous Pneumothorax
A 20 year-old male presented to ED with shortness of breath.

Task: history, physical examination, investigation and management.

Hx: (Stable? Pain? Put oxymetry, offer O2 and pain killer)


When & how did it happen?  3 hrs ago while driving, feel SOB and chest pain
Suddenly or gradually?
Is it the first time?
Pain Qs  any fever, cough, any pain in the calf?
Any trauma to the chest?
Any long trip recently?
Are you generally healthy? Do you have asthma? (complications of asthma)
SA DMA
Occupation (probably flight attendant, diving instructor)

O/E:
GA: oriented, confused, distress, is SOB progressive or stable
VS: BP 97/65, pulse, RR  tachypnea, decrease saturation, JVP increase
Respiratory exam:
- subcutaneous crepitation in the neck (one of the signs in the subclavian area of tension
pneumothorax)
- Inspection: chest movement - decrease on the left side
- Palpation: position of trachea
- Auscultation: reduced breath sound, decreased wall movement on the side of the lesion,
hyper resonant, decreased vocal fremitus (increase only on consolidation)
- Percussion: hyper resonant on the left side

Ix: CXR (Measure the pneumothorax)

Explanation:
Your condition is called spontaneous pneumothorax, air in your pleural cavity. The cause is
unknown.
There is a cavity in between chest & lungs called pleural cavity. Normally there’s no air in
there, but b/o ruptured of air sac, you’ve got air in there, not enough space for your lung to
move, this cause SOB.
I’ll call the surgical registrar to assess you but most likely we will put a small tube to this
pleural cavity to evacuate or suck the air out, it will help with your breathing.
- If pt symptomatic- chest tube (or needle thoracotomy as emergency procedure)
- If pt is stable (depends on the condition of the pt)  observe and do serial CXR

There is a chance it may happen again (30 % will have recurrence and the highest risk within
first 12 months).
You need to stop smoking, avoid some activities such as diving, flying for 6 months
If it happens again come back or see your local doctor

I’m physically active, can I continue swimming, bicycling?


We don’t want to change your life dramatically, keep your physical activity

! 9!
151
RESPIRATORY SYSTEM

Inter Costal Catheter (ICC) Insertion


The following steps are the most common method employed to place a chest tube:
• Place the patient in the supine position (45 degree if possible) with the arm of the
involved side over the head.
• Select and mark the site (4th or 5th intercostals space in mid-axillary line, upper level of
the lower rib)
• Prepare the skin around the area of insertion with 10% povidone-iodine solution or
chlorhexidine and drape with sterile towels.
• Using 1% lidocaine, anesthetise a 2 to 3 cm area of skin and subcutaneous tissue one
intercostal space below the intercostal space that will be penetrated. This will allow for
the development of a subcutaneous tunnel through which the chest tube will be placed to
prevent air entry after the chest tube is removed. 10-20 ml of lidocaine is needed for
optimal analgesia.
• Make a 2-4 cm skin incision parallel to the intercostal space, immediately above the rib
below in order to reduce the risk of injury to the neurovascular bundle.
• Using blunt dissection with a Kelly clamp, create a subcutaneous tunnel from the incision
site cephalad towards the intercostal space through which the chest tube will be inserted,
passing just above the rib to avoid the neuromuscular bundle and through the parietal
pleura (characterisitic “pop” and a hiss of air in pneumothorax). Open the Kelly clamp to
spread the intercostal muscles and parietal pleura.
• Insert a finger through the tract into the pleural space to confirm proper position and
make sure there are no adhesions between the lung and the pleural surface.
• Clamp the chest tube at the insertion end with the Kelly clamp. With the aid of the clamp,
insert the chest tube through the tract into the pleural space and direct it either apically for
a pneumothorax or inferiorly and posteriorly for a pleural effusion. Advance the tube to at
least 3 cm beyond the last lateral hole!
• Remove the Kelly clamp and confirm the location of the chest tube by the visualizing
condensation within the tube with respiration, or by observing pleural fluid drain through
the tube.
• Close the skin incision with mattress or interrupted sutures, and tie one of the sutures to
the chest tube to anchor it. Cover the site with sterile gauze and surgical tape.
Alternatively, cover the site with sterile petroleum gauze.
• Connect the chest tube to the pleural drainage system. Make sure that all connections
between the chest tube and pleural drainage system are tight and taped securely. Make
sure that the underwater drain is bubbling or swinging – ask the patient to cough to
confirm position.
• CXR to confirm tube position and assess lung expansion in case of pneumothorax or
drainage of haemothorax. Make sure that the gap in the radiopaque marker in the chest
tube, marking the most proximal drainage hole, is within the pleural space.

Complications of chest tube placement, excluding recurrent pneumothorax:


• Blockage or failure to drain due to blood clots, position against the chest wall, multiple
adhesions or links.
• Subcutaneous placement
• Bleeding
• Puncture of solid organs (lung, diaphragm, liver, spleen, heart, aorta)
• Infection.

! 10!
RESPIRATORY SYSTEM

• Empyema

Skin preparation and marking.

Local anesthesia.

Skin incision.

Blunt dissection down to the intercostal muscle

Palpation of the selected intercostal space and the superior margin of its inferior rib.

A closed and locked Kelly clamp is used to enter the chest wall into the pleural cavity. Make
sure to guide the clamp over the upper margin of the rib. Once the Kelly clamp enters the
pleural cavity, the clamp should be opened to further enlarge the opening.
A finger is used to palpate the tract and feel for adhesions before insertion of the chest tube.

The proximal end of the chest tube is held with a Kelly clamp that is used to guide the chest
tube through the tract. The distal end of the chest tube should always be clamped until it is
connected to the drainage device.

Connection of the chest tube to a drainage system

A 0 or 1-0 silk or nylon suture is used to secure the chest tube to the skin.

Apply petrolatum (eg, Vaseline) gauze over the skin incision

Apply support gauze dressing around the chest tube and secure it to the chest wall with 4-in
adhesive tape.

! 11!
RESPIRATORY SYSTEM

Chest tube drainage system

Three functional chambers are generally a part of most chest tube collection devices. From
right to left, the first chamber (ie, collection chamber depicted with three sub-sections)
accepts air and fluid from the patient via the chest tube; the fluid accumulates in this
chamber. The air rises and enters the second chamber (ie, water seal chamber) which contains
water at the bottom. Air from the patient enters this chamber below the water level bubbling
through the water seal preventing return of air to the patient. The air enters the third chamber
(ie, suction chamber) connected to wall suction, and is discharged through the hospital
collection system. The height of water in the suction chamber indicates the amount of suction
applied. Suction pressures are typically between -10 and -40 mmHg. An atmospheric vent
prevents the application of excessive suction; manual venting through a pressure relief valve
rapidly equilibrates the collection chamber with atmospheric pressure. Modern devices vary
in appearance, method of suction regulation, and volume of the fluid collection chambers;
this basic design in common to most.

! 12!
152
RESPIRATORY SYSTEM

Pulmonary Atelectasis
You’re examining a patient in a hospital who had laparoscopic cholecystectomy one day ago
for gallstone and she’s having fever of 38.5.

Task: assess overall condition of patient and provide diagnosis & management.

Hx:
I understand that you had a cholecystectomy operation & you’re having a fever. I’d like to
ask you a few more questions.
Was it an elective or emergency operation?
Have you noticed any difficulty in breathing?
cough or chest pain?
Any calf pain?
Abdo pain?
Any smelly discharge from the wound?
How is your general health?
Any other medical issues in general?
Are you allergic to anything?
Are you on any medication?
Do you smoke? -- 20 cigarettes a day.
Do you drink alcohol?

O/E:
Now, I’d like to see the operation record, drug chart, drainage chart, observation and wound
chart, U/O
GA: She looks well and moving freely
VS: T 38.5, P & BP, Sats
I’m looking at the drainage tube for any discharge, what’s the colour
Wound site normal, no discharge
Respiratory: crackles on the base of lungs
CVS: normal
Abdomen: soft
Urine dipstick

Explanation:
Mrs Smith, most likely you’re having a condition called Pulmonary Atelectasis. It’s a very
common condition after operations and in this condition a part of lung is collapsed and not
functioning properly. We need to do CXR to confirm the Dx. Others Ix: FBE, CRP, Blood
culture & sensitivity.

You need to be in the hospital till you get better.


You need to increase your mobility. Physiotherapist will tell you the respiratory exercises to
cough and spit out sputum. Use of incentive spirometry. We also need to measure the pulse
oxymetry. If needed, we can give humidified oxygen & bronchodilator. If you do not get
better we’ll give you oxygen by face mask and ventolin bronchodilator (Salbutamol). If
everything goes well, you can go home in 2 days time. Full recovery in 2 weeks.

! 13!
153
RESPIRATORY SYSTEM

Supraclavicular Node (Examination)


(Pancoast Tumor)
A 65 yo man came to GP clinic after having a swelling in the R supraclavicular region which
was biopsied by a surgeon and sent to pathology. FNAB showed SCC.

Task: exam and management.

Primary focus
1. Skin – squamous cell carcinoma
2. Lung
3. ENT: Oesophagus; Nasopharyngeal; Parotid
4. Breast
5. Stomach and testis go to supraclavicular lymph node although not SCC

Small cell carcinoma – 15%


80% present with distant metastasis at the time of diagnosis  chemotherapy &
radiotherapy; if there is distant metastasis no operation just chemotherapy & radiotherapy.

Non-small cell carcinoma


- Squamous Cell Carcinoma has the best outcome  treat with surgery + chemotherapy.
- Adenocarcinoma

O/E:
GA – dyspnoeic
Scalp and Skin all over the body  r/o SCC of skin
Face: Horner syndrome  temperature of face (anhydrosis), eye (ptosis and meiosis)
Lymph node: cervical, supraclavicular, axilla, inguinal
ENT (mouth, nose, parotid gland, gag reflex  ask pt to swallow, speak a few words)
Chest exam properly
Abdomen including inguinal area, testis

Ix:
Do chest X-ray and refer to specialist
Might need CT scan

Procedure:
I notice your R eye is dropping, was it like it before? No
Any double vision?  Yes
I can see your R pupil is constricted.
I’d like to do fundoscopy, it just a shining light, won’t hurt you.
Can I palpate your face?
Have you lost weight?
Do you smoke?
Have you noticed any change in your voice?
Can I see inside your nose and mouth?
Could you pls have a sip of water? Cough?
Any nasal bleeding?
I’d like to palpate your LN?

! 14!
154
RESPIRATORY SYSTEM

COPD (AMC 132)


How are you feeling right now?
Your lung function test which we have done recently does not show a good result, most likely
it’s a chronic obstructive pulmonary disease.
There is a small obstruction in your air vessels.
Also there is some mucous and phlegm.
It causes difficulty in passing air, difficult for air to come in and out.
As from your note, you’re a smoker for the last 35 years.
There are a lot of reasons, but that might be the reason for this, because cigarette contain
some ingredients that irritate the airway and also cause inflammation of the airway. This
irritation and inflammation is chronic now.
As you’ve stopped cigarette 2 days ago, I’m very happy to hear that. It’s never late.
How are you coping right now, we’ll talk about this later.

This disease is progressive and chronic, sometimes it runs in families. It has remission and
exacerbation. But I’m here to help you and you’ll be followed up by a respiratory team in the
hospital.

Mx:
C – Confirm the diagnosis asthma or COPD  give Ventolin
O – Optimise function
- stop smoking (continue stop smoking)
- pulmonary rehabilitation by multidisciplinary approach to improve the symptom &
quality of life
o exercise training
o psychosocial support
o chest physiotherapy
o nutritional advice
o patient and carer education
- symptom reliever  SABA (short acting beta agonist) – anticholinergic  LABA
(long acting beta agonist) – steroid inhalation
- If FEV1 < 50%  adequate rest, don’t go to work, fresh air, avoid irritant, pollutant
and people with flu

P – Prevent deterioration
- stop smoking
- influenza vaccine every year
- pneumococcal vaccine every 5 years
- regular review to monitor complication
- check medications
- oxygen therapy (low concentration oxygen)

D – Develop a self management plan supported by specialist, GP and other healthcare worker

E – Exacerbation  if you have SOB, audible noise, chest tightness, change of sputum
colour, increase volume, you need to contact your GP or call 000. You should have Ventolin
& corticosteroid at home. When you have this kind of symptoms, you might need to take it
and call 000. Also I will give you a COPD action plan.

! 15!
RESPIRATORY SYSTEM

X – Management of exacerbation for acute COPD


- Oxygen
- Nebulised Ventolin + Ipratropium
- Steroid oral 30-50 mg for 7-14 days + 1 dose IV
- Amoxycillin if evidence of infection
- Chest physiotherapy
- C-PAP (Continuous Positive Airway Pressure)

! 16!
155
RESPIRATORY SYSTEM

Spirometry (AMC 103)


A 22 yo male with a history of mild asthma until the age of 11 years. There has been no
symptoms in recent years and no need for treatment. Respiratory system normal.

Task: You’re asked to evaluate the patient’s lung function using a spirometer. Also to
determine the Forced Expiratory Volume in 1 second and Forced Vital Capacity and compare
these with normal values. Then explain the results to the patient.

Critical errors:
Failure to recommend maximal inspiration
Failure to recommend maximal expiratory effort
Failure to calculate the ratio FEV1/FVC

Tell patient:
Spirometry is the most helpful test for assessment of your lung function by measuring the
volume of air that you can expel from the lungs after a maximal expiration. I need to do it 3
times and take the best result.

First is your position


Please sit upright in the chair and touch your foot on the floor
1) I will put a clip on your nose to prevent air escaping through the nose
2) I will insert a clean mouth piece
3) Take breath as deep as you can
4) Sealed your lips around the mouth piece
5) Blow out as hard as possible and as long as you can
6) Continue for 6 seconds until I stop you
7) Let me know if you feel any distress during the procedure
8) We need to do it 2 more times
9) In between 2 attempts, the normal gap should be <150 ml

Avoid cough and leaning forward, if you cough we need to take it again

Important factors taken into account in determining predicted normal values: age, sex, height,
weight.

The spirometer calculates a comparison of the two values—called the FEV1/FVC ratio (or the
FEV1/FEV6 ratio)—which tells you what percent of the total was exhaled in the first second.
The machine also compares the patient's FEV1 and FVC with values of a healthy person of
the same age, gender, and height. These "normal" values are also called "predicted" values.
The spirometer calculates the patient's values and the predicted values as a report of the "% of
predicted" values. Some spirometers also display "best values," selecting the highest value of
each parameter from all the patient's exhalations.

FEV - Forced expiratory volume in one second: The volume of air expired in the first
1
second of the blow

FVC - Forced vital capacity: The total volume of air that can be forcibly exhaled in one
breath

! 17!
RESPIRATORY SYSTEM

FEV /FVC ratio: The fraction of air exhaled in the first second relative to the total volume
1
exhaled.

Spirogram pattern: Normal, Obstructive, Restrictive, Mixed

What are abnormal spirometry results?


Spirometry results are expressed as a percentage, and are considered abnormal if less than
80% of the normal predicted value.
An abnormal result usually indicates the presence of some degree of obstructive lung disease
such as asthma, emphysema or chronic bronchitis, or restrictive lung disease such as
pulmonary fibrosis.

Patient’s questions
Do I have asthma?
Spirometer only detect lung function test

How do I diagnose asthma?


I will give you Salbutamol (bronchodilator) 2 puffs and wait 10 minutes, if the difference is
>12% (pre and post Salbutamol), that means asthma
Example pre 80, post 100  difference 20

If the difference is <12%  restrictive


If the difference is >12%  obstructive (COPD, asthma)

OBSTRUCTIVE RESTRICTIVE MIXED DISORDERS


FEV  N 
FVC N/  
FEV1/FVC  N/ 

If results decrease it means obstructive


If increase or normal it means restrictive

! 18!
156
RESPIRATORY SYSTEM

Hoarseness Of Voice
(Lung Cancer With Recurrent Laryngeal Nerve Palsy)
Patient came to your GP practice complaining of hoarseness of voice.

Task: examination findings, diagnosis and management.

DDx:
- Acute laryngitis
- Cancer
- Ca of lungs
- Cancer of laryngs
- Thyroid cancer
- Hypothyroidism
- Vocal abuse
- Myasthenia gravis
- Motor neuron disease
- Foreign body
- Allergy

Hx:
How long
Started gradual or suddenly
Any viral infection before
Any cough, fever
Neck swelling; Weather preference; Dry skin
For female – irregular cycle or PV bleeding
Weight loss
Any trauma
Surgery
Smoking
Occupation
Any arm pain, weakness, or vision problems (motor neuron disease)
PMHx: Thyroid
Medication: corticosteroid, antipsychotics
Allergy

O/E:
GA: features of hypothyroidism and Horner’s syndrome
VS:
Neck: swelling, cervical lymph nodes
ENT: Careful oropharyngeal; Indirect laryngoscopy
Respiratory exam: dullness in upper lobes (percussion), decreased breath sound, increased
fremitus.
Heart normal, Abdomen normal

! 19!
RESPIRATORY SYSTEM

Ix:
- CXR
- Basic blood test: FBE, U E C, LFT, TFT

Explanation:
Unfortunately Mr X, we found some problems in your lung. Your X-ray showed some kind
of mass in the lung, most likely it’s a lung cancer. It’s a cause of your hoarseness of voice
due to laryngeal nerve palsy. I need to refer you to an oncologist because I suspect a nasty
growth. The oncologist will do a biopsy, take a small piece of this growth and do a
bronchoscopy, a small tube is put into your wind pipe.
We need to do staging with CT scan to check if there is any spread of this growth.

Can it be operated?
It depends on the type of cancer and stage of the condition. The oncologist will discuss with
you the possible treatment options when all results come back. Possible treatment options for
people with lung cancer are:
- Surgery
- Chemotherapy
- Radiotherapy

Incurable at presentation
- Liver metastasis
- Malignant pleural effusion
- Involve cervical lymph nodes
- Recurrent laryngeal nerve palsy

! 20!
157
RESPIRATORY SYSTEM

Pain Management
You’re a junior doctor in ED and a patient just came in with chest wall pain and shoulder
pain while playing footy with his son. Trauma to shoulder 2 days back.
(A 45 year-old man with NSCLC with metastasis to the liver and other sites.)

PAIN MANAGEMENT
Panadol
Panadeine forte
NSAID
Opioid
- 2.5 – 5 mg orally
- Side effect nausea (30%) give Maxolon; constipation give laxative
- If severe nausea swap to different opioid such as hydroxymorphine (5X stronger than
morphine) or oxycodone (1.5-2X stronger than morphine)
 Antiemetic counter with Maxolon
 Haloperidol (metabolic related nausea, vomiting)
 Ondansetron  chemotherapy related nausea, vomiting
 Cyclizine or Promethazine (motion related)
 Levomepromazine
 Nozinin

Morphine 2.5 – 5 mg q4h SC (give every 4 hours)


- If given for the first day and patient is fine with the dose  the dose can stay or change
- If change, see if the next day is better, see the trend
- If Morphine is given 20 mg in 24 hrs, it can be changed to 10 mg bid (long acting) 
total 20 mg
- If patient still have pain and is having more peaks of pain  give a breakthrough dose of
5 mg (short acting) prn  8 times a day  40 mg
- Total dose the patient is having 20 + 40 = 60 mg per day
- Short acting morphine (oral) in average last for 4 hours  q4 hourly (6 times/day)
- For the pain in between – give breakthrough dose 10 mg q4h prn
- If the patient’s pain is controlled with the increase frequency of morphine, check the pain,
where is the pain now, it might came from the mestasis in the leg (bone pain)
- Radiotherapy is the best treatment for bone metastasis

TYPES OF PAIN
- Somatic pain is relieved by morphine
- Some pains have to be added with other medications eg radiotherapy or medications like
NSAID (Naproxen SR 75-100 mg bid), Dexamethasone 4 mg/day, Ketamine for bone
pain
- Neuropathic pain  tricyclic, gabapentin, steroids, NSAID (for the inflammatory
component around the nerve)
- Liver capsule pain (capsular because of stretching or solution compression)  opioid +
NSAID or steroids.
- Back L1-L2 metastasis  bone metastasis, spinal cord compression (80%) mixture of
bone + neuropathic (if miss the metastasis patient can end up with paralysis)  give high
dose steroids 60 mg, MRI

! 21!
RESPIRATORY SYSTEM

Patches
- Norspan patch (buprenorphine 5 = 5 mg MS Contin/day) for 7 days for osteoarthritic pain
- Fentanyl patch 25mcg/hour for 3 days  t1/2 = 12-18 hours
 25 mcg = 2.5 mg morphine
 30 mg morphine S/C in 24 hours or
 60 mg morphine orally in 24 hours

Fentanyl patch usage


Static pain relief
Head & Neck, Oesophageal, Gastric cancer
Renal failure (if give morphine it accumulates in the system)
Renal impairment give hydromorphine
Renal failure give fentanyl and methadone
If constipation is an issue  patch can be given eg for bowel obstruction

Methadone
- Renal failure
- Neuropathic pain

! 22!
158
RESPIRATORY SYSTEM

Pulmonary Embolism
GP setting. A 35 yo female presented with sudden onset of SOB.

Task: take history, examination, investigation and management.

DDx:
- Tension pneumothorax
- Pneumonia
- Pulmonary embolism
- Heart problem
- Pleural effusion

Hx: (stable or not, offer O2)


When and how did the shortness of breath start?
Any precipitating factor?
Any sore throat, cough, fever, sputum, palpitation?
Any chest pain, calf pain?
How is your general health DM, hypertension, asthma, any clotting disorder
Any medication and OCP?
Recent surgery, travel, fracture
Any gynaecological procedure, miscarriage?
Any recent weight loss, night sweat, bowel habit changes, lumps
SADMA
FHx of clotting problem
How is your mood, appetite, sleep?
Anxiety question

O/E:
GA, VS: BP normal, RR increase, SaO2 low, Pulse increase
JVP, Heart , Chest, Abdomen, Legs and arms

Ix:
- Coagulation profile, FBE
- ESR, CRP, ABG
- Thrombophilia screening
- CXR, ECG
- V/Q scan or CTPA

Mx: (In GP: IV line, O2, heparin, warfarin  send to hospital )


- IV fluid
- Morphine if chest pain
- Heparin (subcutaneous) or clexane – start blood thinning medications

Recurrent Pulmonary Embolism or DVT  give warfarin life long


If no cause is found - give for 6 months

Travel advice for previous PE or DVT:


- Stocking compression
- Heparin 40 mg subcutaneous before travel, can repeat in 24 hours if long flight

! 23!
RESPIRATORY SYSTEM

Warfarin education counselling

What’s warfarin
What does it do, how does it work
How to take the dose and how much
What if I miss the dose, what to do
What things should be aware when using this medication (precautions)
Any blood test necessary
Red flags

I’m sorry to hear that you have this unfortunate condition (PE)
Probably your doctor has explained to you your treatment
Warfarin is an anticoagulant medication taken for 6 months
Anticoagulant is a substance that prevents blood from clotting and stops the clot or plug from
forming in the blood vessels

It’s important while on Warfarin to have a regular blood tests which measures your INR
which indicates how long your blood takes to form a clot or a plug

Your INR levels are very important as they help your doctor to maintain the most suitable
dose of Warfarin for you.

You must take your tablet at approximately the same time each day. If you forget for a longer
time, do not take the tablets to catch up but take your next dose when it’s due. Do not take a
double dose.

Different things in your life affect how Warfarin works in your blood such as:
- Diet  the way you eat, eating habits
- Medicines you’re taking
- Amount of alcohol
- Illnesses

The most important thing to remember if there is a change in your life concerning the above,
please come and discuss it with me.

Maintain a well-balanced and consistent diet to avoid crash diet or binge eating
Avoid foods contain too much vitamin K such as in green leafy vegetables such as spinach,
lettuce, broccoli, parsley – take in moderate amount
Alcohol – avoid binge drinking

Avoid pregnancy

When gardening, use gloves; Use soft toothbrush, electrical shaver; Wear alert bracelet
Things patient should look for:
- Obvious bleeding from cuts and takes a long time, nose bleed, very heavy periods, any
bleeding from the gum
- Change color of urine to dark red or brown
- Dark or black bowel motions

! 24!
159
RESPIRATORY SYSTEM

Common Cold
A 20 year-old girl presented with sore throat and runny nose.

Task: take relevant history, ask exam finding, discuss management.

How long have you had this problem?----2 days


Has it improved? ----Today I had difficulty in swallowing, it was bad this morning.
Any cough, hoarseness of voice?
Fever, rash, N/V, joint pain, did you feel lumps in your neck or any part of your body?
Just general malaise, body ache but not joint pain.
Are you sexually active?
Have your partner have symptoms like you? --- He has common cold last week.
You said you have antibiotics before, is that repeated? --Last 2 times I had for tonsillitis
When was it? --- 2-3 months ago
Between this attack were you free?----yes
Have you travelled recently?---no
Are you on any medication?
Have you been diagnosed with any serious condition like hypertension? ----no
Are you working?---yes
Does anyone in your work has the same problem---no
SADMA --- smoking can cause pharyngitis

O/E:
GA: looks tired, little bit dizzy
VS: low grade fever, 37.8, pulse & BP normal
ENT: oral cavity  a mild pharyngitis, tonsil free, ear drum is normal, blocked nose
Any joint pain, arthralgia, lymphadenopathy, rash
Chest, heart, abdomen  all normal

Explanation:
From the Hx & exam you have a viral infection or common cold.
I won’t give you antibiotics because it doesn’t affect your condition.
I advise you to have a good rest, will give you medical certificate, try to sleep in good
amount, also you have to increase your fluid intake. You will have analgesics like
paracetamol. If you have some problems with your nose, there is a steam inhalation. Vitamin
C powder or tablet.

Ix:
FBE
Monospot test  atypical lymphocyte (for glandular fever)

Complication  pharyngitis, otitis media, pneumonia, sinusitis

! 25!
160
DERMATOLOGY

Alopecia (Hair Loss)


Mrs Smith is 63 year-old, came to your clinic complaining of hair loss.

Task: Hx, ask for examination finding, order Ix and explain final diagnosis & management.

DDx of hair loss:


- Telogen effluvium
- Androgenic alopecia
- Iron (anemia), Zinc deficiency
- Endocrinal abnormalities (hyper, hypothyroidism)
- Drug induced
- Diffuse alopecia areata
- Infection
- Radiation, chemotherapy
- Fungal or ringworm infection
- Psychological

Hx:
Since when---3-4 months ago
Acute or gradual
Where is the hair loss – front, all around, in the middle
Any hair loss anywhere else in the body
Period  irregular period, midline hair loss, acne  androgenic
Virilisation
Tiredness and thyroid disease
Drugs
Hair dye – contact dermatitis

Explanation:
Nothing abnormal except hair loss
I don’t know actually what’s causing it, it could be:
- Iron and zinc deficiency  investigate (if given iron & zinc and improved then it’s iron and
zinc)  usually aggravate not precipitate; correction of iron storage, if not improved, seek
alternative causes
- Thyroid disease – gradual hair loss and loss of outer third of eyebrows
o (AMC RECALL  a 35 year-old female with hair loss and loss of right eye brow)
o If due to thyroid disease, hair loss will stop after a few months of treatment
- Telogen effluvium
o Acute excessive dramatic hair loss 2-3 months after stress (any chronic condition), physical
or emotional stress
o Self-limited after 3-6 months
o Thinning all over the scalp
o Hair pull test  equal in vertex & occiput  (+)
o Can be chronic  causing hair block in the shower
- Androgenic (most common cause of hair loss)
o Genetic predisposition related to androgen receptor

! 1!
DERMATOLOGY

o Gradual
o Site: no central widening, bitemporal recession (for male), the crown loss, front hairline
remains (for female)
o +/- manifestation of virilisation  acne, irregular period
- Drug-induced
o Oral contraceptive, Testosteron hormone, Danazol
o Anabolic steroid
o Carbimazole, PTU, Thyroxin
o Lithium
o B-blocker, ACE inhibitor (can cause telogen effluvium); Amiodarone
o Anticoagulant – heparin and warfarin (can cause telogen effluvium)
o Oral retinoid
o Allopurinol
o Levodopa
o Sulfasalazine
o Interferon
o Amphetamine

Ix:
FBE
Iron study
Zinc
TFT, LFT, RFT
If clinically indicated, androgenic screen – check for serum DHEAS
LH, FSH
If clinically indicated check antinuclear antibody (ANA)
Biopsy from the scalp

Mx: Depends on the etiology


Androgenic alopecia
- Conservative: camouflage (paint the area of alopecia) or wig
- Transplantation
- Medical
o Spironolactone (for female, to prevent further loss)
 Should not be given in patient with renal disease
 RFT & potassium should be done before treatment and check annually
 Should not become pregnant, use OCP because can cause feminising of a male child
o Cyproterone acetate (to prevent further loss)
 Should be combined with OCP
 Will take 3-6 months before improvement
o Minoxidil topical
 2-5% apply twice daily for at least 6-12 months
 May notice initial hair loss at the beginning of treatment

Alopecia areata
- Depends if it involves local or extensive area
- Use topical steroids twice daily for 12 weeks
- Intralesion steroid (inject in the hair loss area)
- Topical Minoxidil 5%
- If extensive  send to support group, use wigs, camouflage, send to dermatologist to have a
topical immune therapy, or systemic steroid

! 2!
161
DERMATOLOGY

Acne / Folliculitis
Nadia, 20 years old presented to you with skin rash on her face and upper back.

Task: take history, examination, diagnosis, management, and answer her questions.

Skin rash
When
Where
Go where
Itchy or not---not itchy but painful
Rash in the past
Contact with anyone with rash before
Any advice from any doctor
Previous treatment---OCP but not really helpful
Rash flare up with some food like chocolate, spicy food

Explanation:
Acne very common, 95% of population at some point had it during their live. Different from
one person to another from mild to very severe. It’s a disease of adolescents and adults due to
hormonal changes in the body. Caused by abnormal & excessive production of sebaceous
gland called sebum. Increase of sebum in the gland. This bacteria can lead to obstruction of
the gland, Proprionibacterium acnes (P Acnes). The neck of the gland is obstructed by this
bacteria.
Very common
Risky if you squeeze or scratch it because it can leave a scar.

Classification:
According to papules, pustules and nodules.
According to severity – mild, moderate, severe.
Papule/pustules Nodules
Mild Few to several None
Moderate Several to many Few to several
Severe Numerous or extensive Many

! 3!
DERMATOLOGY

Tx:
First-line treatment:
-Benzoyl peroxide – bactericidal drug over-the-counter
Panoxyl presented as gel and cream
Benzac acne wash
-Keratolytics: most effective topical treatment for early Acne
-Topical retinoids: vitamin A creams, most effective for comedolytic agent (Retin A creams)
 side effect skin redness, hyperpigmentation and photosensitivity (apply at night) and wash
it in the morning. In the morning put sunscreen. Initially flare up, initial exacerbation is
normal. Start with lowest strength and gradually change the strength. Build up over weeks.
Start with 0.025% and increase to 0.05%. Contraindicated in pregnancy. (do pregnancy test)
-Topical antibiotics: Clindamycin or Erythromycin act on the bacteria, both equal in
effectiveness.
-Combination: topical antibiotic in the morning and topical retinoid at night.

Second-line treatment
- Oral antibiotics: Doxycycline 50 mg daily for 12 weeks (if no improvement don’t continue).
Can cause thrush (vaginal candida).
- Hormonal treatment: reduce the production of the gland
- OCP Danazol

Third-line treatment (Referral treatment)


Retinoids (Isotretinoin)
- Reserved for the use of severe acne or moderate acne with resistant to treatment. This
prevents the scarring. It can only be prescribed by dermatologist.
- Can cause dryness of eyes, lips, skin, and mucous membranes. Nasal bleeding from
dryness of anterior nares. Arthralgia, myalgia. Can flare up acne in early treatment.
- Tetracycline is contraindicated with retinoids due to the potential for increased cranial
pressure and an increased risk of pseudotumor cerebri (benign intracranial
hypertension).
- Also contraindicated with oral vitamin A
- Can cause elevated tryglyceride
- Should not be used in pregnancy

! 4!
162
DERMATOLOGY

Psoriasis (AMC 92)


Jessy, 23 year-old, came to your clinic because she has a red plaque, red elevated rounded
lesions, on her arm. Also another red plaque on the back of her neck close to the hairline.
Both of them itchy, annoying her.

Task: Hx, examining finding take from the examiner, tell pt the Dx & manage her condition.

Reddish scaly, rounded on the back of the elbow


Extended the hairline on the back of the neck
Fine, silvery skin all over the lesion

I read from your notes that you have lesion in your arm and neck.
Can you tell me when did you have the lesion?
2 months ago in the arm, on the back of elbow, red, little bit raised, later some scale on the
lesion. On the back of neck started later, reddish thick, covered by silvery scale. Is that
itchy?….most of the time is itchy.
Normally not itch, but some people stress of the lesion.
Any other lesion in your body?

It’s a chronic condition.

Anyone in the family has this condition? ---- her grandpa has lesion like that….

Do you have partner


Sexually active
Any children
Happy family
Partner supporting
Any financial problem
What’s your job, is it stressful or not.
Outdoor or not, affect of the sun
Any serious medical condition
Chronis disease
On any medication
Smoke, Drink Alcohol
Any history of allergy (seborrhoic dermatitis)
Have you been diagnosed with allergy
Any family history of allergy
Did you try any cream or seeking any help -----cream but didn’t help

O/E:
GA
VS
Rash  location, lymph node (infected),
Back elbow, back of the neck near the posterior hairline
Scaly rash covered with silver lamellated scales all over the lesion

Psoriasis can be found in


- Scalp near anterior & posterior hair line

! 5!
DERMATOLOGY

- Around navel
- Gluteal
- Back of elbow
- Down genitalia
- On the front of the knee
- Buttocks

ENT, heart?

Mrs Smith, from the hx & exam, you have psoriasis. This is a chronic skin disease, can be
mediated with T lymphocytes because it’s thought that it’s an autoimmune disease. It’s a
common disease, around 2-4% of people suffered from this. Your lesion comes from
Our skin has 2 layers, epidermis & dermis. Epidermis has 5 layers. As the skin is renovating.
The normal period is 120 days. In Psoriasis the renovation is in 29 days, renovation
accelerated. Accumulation of the cells. Thickness of the skin increase. Keratin accumulated
on top. It’s focal. Two separate lesions can comes together. DNA of the people has
acceleration. No actual known cause, perhaps stress, chronic debilitating condition.
When recurrence, give methotrexate (aggressive treatment).

It’s chronic
It’s common, 2 % in the world.
It’s risky, you come here in good time, you’ll be follow up by me and I’ll refer you to the
specialist. Carries risk 10-15% become psoriatic arthritis.
Remission and exacerbation

Give corticosteroid, withdraw gradually.


Avoid stress
Supporting group for this disease

Treatment refer
1. Mainly corticosteroid cream started from 1% until lesion improved. Gradually withdraw.
2. Coal tar preparation, include shampoo. In the scalp, has to peel the lesion.
3. Narobent
4. Frequent sitting (2-3) per week, in the light  phototherapy.
5. Vit D3 derivatives  Daivorex cream (Calcipitriol).
6. Complicated cases use methotrexate

Sun exposure is good but put sunscreen and avoid the sun when it’s harming between 9am –
3 pm.

Red flag: secondary infection


!

! 6!
163
DERMATOLOGY

Contact Dermatitis (AMC 42)


A 32 year-old man came to you presented with hand lesion (there is a rash on his forearm and
the photo handed when you start asking examination finding from the examiner).

Task: take relevant history, examination finding and explain what you see in the photo,
explain to patient the diagnosis and management.

How long have you had the lesion?


Was it gradual course
Improved or deteriorated
How did it start---start as a redness pimples then form vesicles with yellowish spots in the
lesion
Is this the first time or is it a repeat incidence
Did you try anything to relieve the pain ---body cream but doesn’t help
Any itchiness
Any spread of the rash ---no, but my other hand had started to be red too
Is it painful
Do you have other symptoms like weakness, fever, vomiting
Do you have any allergic history in the past, asthma, hay fever
Any contact with animals, pets at home or plants
What’s your hobby, do you like gardening
How many times a week
Maybe with construction work or oils or planting or chemical hobbies
Did you recently try any perfume or cream on your body
Any allergy from synthetic material or clothes

What do you do for your living (speak in detail about the job)---work in construction
(engineer or worker)
Are you involved with irritant substance or not ---some substances like cement, concrete

Mr Smith, do you wear any gloves or protecting clothes for your work---no
How long is your sleeves
Do you wash your hands after work

Are you on any medication


Are you allergic to anything

! 7!
DERMATOLOGY

FHx of atopic conditions


Any itchiness ask mental Qs How’s your mood

O/E:
GA: normal, healthy but a little bit distracted with the hands lesion
VS: normal
Describe the lesion
On the right forearm there is a lesion extended from the wrist up to the elbow, red
erythematous rash with vesicle formation. There are some scattered yellowish spots
indicating that there is infection in the lesion. My diagnosis is infected contact dermatitis.
What is it?
It is a skin reaction as a result of irritating substance come in contact with the skin. This
substance can be non irritating for other people but in some persons they have
hypersensitivity or sensitive immune response, so they react with this substance abnormally.

Most probably allergy from the cement or concrete.


Because your hands have infection, I’ll give you a course of systemic antibiotic and some
cream called corticosteroid or betamethasone (Elocon).
For face lesion use hydrocostisone cream.

Concerning your job, I’ll give you a certificate for 3-4 days.
When you have used the antibiotic & cream, in 1 week you’ll be all right.
I’ll refer you to the dermatologist, who will do a patch test.
Several substances will be put on your skin (usually on the back of the body) for 24-48 hours
and then the specialist will see the effect of the substances on your skin. Peel the patch and
see which one is the most reddish, it means more irritant to your skin. Which one irritates
your skin, try to avoid it in future. The specialist will make a grading for the report.

1) First avoid the cause


2) Avoid the substance that is allergic to you
3) Use barrier cream, before you start your work, put the barrier cream on all your hands to
elbow, use long-sleeves glove. Much better to use cotton-lined gloves. Perhaps you’re
allergic to the rubber of the gloves (the latex). Use the cotton gloves then the rubber gloves
on top.

Try to wash your hands after you work.


If you can change your position, it’s much better but better to confirm which substance is
irritant to your skin first.

! 8!
164
DERMATOLOGY

Melanoma
A biopsy report showed a skin lesion is melanoma with depth of involvement of 0.4 mm
thickness with the tumour extended to the lateral margin, no lymph node enlargement. The
picture was handed and the patient is a school teacher who had this mole for many years.
Recently it became itchy thus he came and had the biopsy done. No history or examination
required.

Task: you’re to explain the biopsy result to the patient and advice on management.

Superficially spreading melanoma (SSM)  dark spot, with lighter colour spreading around.
The spot has 2 colours.

Mr Smith, today you came to know the result of your biopsy. Unfortunately that the result
showed melanoma, a type of cancer skin, third commonest in Australia, the most dangerous
one. But the good thing is that the depth of your lesion is very small. You know what it
means? It means that your cancer has not spread and the lymph nodes are not involved. It
carries a good prognosis. The report said the depth is 0.4 mm, it is much less than 0.7 mm
which is considered a risky depth. We already cut your mole but we have to refer you to the
surgeon or dermatologist. He will cut off some of the normal tissue around the lesion because
we have to have a good and correct safety margin around when we excise the lesion.
The safety margin for lesion with depth of 0.4-0.7 mm is 1 cm. We have to have some normal
tissue excised to prevent the lesion from repeating.
If >0.7 mm, the safety margin is around 2-3 cm.
The depth of cut is to reach the subcutaneous tissue  the cut will be a cone-shaped.

He will check the lymph node. If there is lymph node involvement, it has to be cut also.
After referral to the surgeon, after the surgery, he will decide to give you chemotherapy or
not according to your condition.

You have to avoid the sun especially after 9 am until 4 pm. When you go out, use sunscreen
at least with spf 30. What about your work, outdoor or indoor.
Before you drive, put sunblock on your exposed area.
Mr Smith, when you go to the beach, use wide-brimmed hat.

I will follow you up every 3 months for at least 1 or 2 years and after that we will extend the
period up to 10 years. Careful follow-up because melanoma tends to repeat.
My concern now is that if you notice any lumps or bumps or any mole, don’t hesitate to show
me or go to the specialist.
As the lesion is not deep, the outcome (prognosis) is 95-99%.
But there is high incidence for repeating.

! 9!
165
DERMATOLOGY

Squamous Cell Carcinoma


A 30 yo farmer came to your clinic last week due to 5 mm lesion on his R temporal area. You excised
it & send for biopsy. The result showed squamous cell carcinoma with unclear margin. The depth of
the tumour is 0.4 mm, no cervical LN enlargement. No sign of metastasis.

Task: explain to the patient & further management.

Well Peter, we excised your lesion & send it for biopsy last week and now I got the result. I’m sorry
to tell you that I don’t have a good news. The result showed that the lesion is squamous cell
carcinoma, a kind of skin cancer.
Please don’t worry too much, you’re in the early stage. It’s good that you came now & we can find
out the best treatment.
The result also showed that the margin of the tumour is not clear, it means that not all the tumour was
excised last time. I need to refer you to the surgeon to do more excision, so that all the tumour is
excised with adequate margin.
I want to reassure you again that your tumour is in the early stage which means the outcome is very
good. Do you understand?
For the tumour there are 2 options of treatment:
1. Surgery – we need to excise more
2. Radiotherapy

What do you prefer doctor?


For me, I think surgery is better because the tumour is removed totally.
I’m happy that you have no sign of spread of tumour, it’s very good, but the surgeon may decide to do
further tests like CT to make sure that there is no spread. I want to give you some more advice as the
cause of skin cancer is mainly sun exposure, so you need to avoid the sun especially between 10 am
and 4 pm.
Sunbathing and tanning of the skin

But doctor I’m a farmer?


Try as much as possible to avoid the sun.
If you can’t avoid, please wear long sleeves shirt, sunglasses, broad trimmed hat and long trousers to
cover your body from the sun and please wear sunscreen which has at least 30 spf. Try to stay in the
shades. For the sunscreen please apply 30 minutes before you go out, reapply every 2 hours or after
swimming or heavy sweating. Please examine your body from head to toe every month.
For the appearance of similar lesion or any lumps or bumps, if you think you have any abnormality,
please report to your doctor and see a doctor.
See your doctor every 6 months.

Is it melanoma?
It’s not melanoma. Melanoma has the worst outcome among skin cancers but yours is squamous cell
carcinoma. SCC has a very good outcome if it’s in early stage.
The 5 year survival rate is >90.

There are 3 types of skin cancers:


1.Basal Cell Carcinoma
- Age >40 years
- Mainly fair skin people
- Mostly on the mask area
- Related to basal layer of the skin
- Slow growing tumour
- 80% commonest
- No metastasis

! 10!
DERMATOLOGY

- Spread locally destroying the tissue, deep destruction


- Predisposing factor - sun exposure
- Undermined-edge  rodent ulcer
- Types: nodular, ulcerated, pigmented, fibrotic (scar-like), common (pearly edge,
telengiectasia)
- Therapy:
o Surgery: 3 mm margin, depth – cone shaped; Moh’s micrographic surgery
o Radiotherapy: for old frail patient, not suitable for surgery eg in eyelids
o Photodynamic therapy
- Follow up in 6-12 monthly
- General advice

2.Squamous Cell Carcinoma


- No specific age
- Mostly fair skin, can be dark skin
- Mostly sun exposed area
- Growth in keratocytes
- Second most common skin cancer, about 15%
- Spread: direct, lymph node (mostly), blood spread (metastasis)
- Predisposing factor – sun, burn, scar, sinus, actinic keratosis, Bowen’s disease
- Elevated everted edge
- Prognosis: depth < 2 mm metastasis unlikely; > 6 mm metastasis likely
- Pathology: grade 1 (low grade), grade 2 (moderately differentiated), grade 3 (anaplastic)
- Site: lip, ear – highly recurrence; extremities  far worse
- Therapy: surgery (best) size <2cm = 4 mm margin; size >2cm = 1 cm margin
- If lymph node (+)  remove regional lymph node
- Radiotherapy  if patient is not suitable for surgery; for early stage 80% curable
- Chemotherapy  to reduce size of tumour before surgery
- Follow-up depends on the size  3-6-12 monthly + general advice

3.Melanoma
- No specific age, no specific site
- Presentation: changes in moles – size, shape, colour, bleeding, itchiness, satellite lesions
- Least commonest <5%, worst prognosis
- Spread can be direct, lymph node, blood
- Predisposing factor – sun
- Types: lentigo maligna (Hutchinson’s malignant freckles), superficial spreading
(commonest), nodular, acral (extremities)
- Prognosis depends on Breslow thickness and lymph nodes
Breslow (in mm) 5 year survival rate
< 0.76 95
0.76 – 1.5 70-98
1.51 – 4.0 55-85
>4 30-60

- Therapy – surgery
o < 1.5  1 cm margin
o 1.5 – 4  1-2 cm
o > 4  2-3 cm
- Lymph node (+)  block dissection
- Follow-up: 3-6 monthly + general advice

! 11!
DERMATOLOGY

SKIN CANCERS
LESION BCC SCC Melanoma
Site Masked area Sun exposed area Can come with mold with
In the basal cell layer of the skin In epidermis many changes
(deep) (keratinocytes) (melanocytes)
Fair skin
>40 years old Can affect any age Any age

Spread Slow progress Can be direct or lymph, Can go to any place


Direct spread to underlying blood and metastasise Check regional lymph node
structure, destroy the structure Distant spread by blood to
around but no metastasis liver, brain
(limited)
Predisposing Sun exposure Sun exposure Sun exposure
factor Old scars, burns
Premalignant like:
Bowen’s disease,
leucoplachya, actinic
keratosis
Area became firm, then
erythemathous with
scales, ulcerated
Shape Rolling edge inside Ulcerated Lentigo maligna
Nodular Horny Superficial spreading
Cicatricial (scar like), sclerosing Fungated, cauliflower- Nodular
Pigmented like Acral (related to limbs)
Ulcerated
Rodent ulcer
Treatment Surgical excision – get rid of Surgical excision, check If depth <0.7 not vigorous
lesion and sample to confirm; lymph nodes, if enlarged  excision 1 cm around,
Safety margin 1 mm around sentinel lymph node deep to subcutaneous
because the lesion is not biopsy. Margins like tissue, If >0.7  do other
superficial. melanoma. excision to reach up to 2-3
Radiotherapy (if surgical not Radiotherapy if can’t be cm around and to deep
possible if lesion inaccessible or excised. layers, check regional
for old people) 80% effective – Cytotoxic medication lymph node, if involved
if in cartilage will cause (Bleomycin) – cream excised. Check
necrosis. and tablets – decrease histopathology on lymph
Cryotherapy (but cannot take the tumor size gland.
specimen). Chemotherapy

Follow-up 6-12 months Regular check up (6-12 Every 3 months in the first
month) for the first 2 year and FU until 10 years
years
62-65% 5%
First common Second common First dangerous
Prognosis Good Good 95% (5-yr Depends on the depth, if
survival) >0.7 it carries involvement
of lymph node
Ear – SCC
Behind the ear – BC
Recall: SCC – on upper lip; tongue (with lung cancer)
Melanoma

! 12!

S-ar putea să vă placă și